PANCE: Pre-Test

अब Quizwiz के साथ अपने होमवर्क और परीक्षाओं को एस करें!

104. Which of the following describes the triad of features that constitute the female athlete syndrome? A. eating disorder, low bone mineral density, amenorrhea B. eating disorder, dyspareunia, low bone mineral density C. menstrual dysfunction, depression, eating disorder D. menstrual dysfunction, low bone mineral density, hyperthyroidism

104. The answer is A [Ob/Gyn, Endocrinology]. A. The classic triad of the female athlete syndrome is menstrual dysfunction (usually amenorrhea), an eating disorder, and low bone mineral density (osteopenia/ osteoporosis). B. See A. C. See A. D. See A.

101. A 4-year-old girl presents with a 4-day history of limp. She has no history of trauma. She had a viral upper respiratory infection about 1 week ago that resolved without consequence. She is currently afebrile and has no other symptoms. Examination is unremarkable with full range of motion in all joints but a limp favoring the left leg. CBC with differential, ESR, and CRP are within normal limits. Radiographs are negative. What is the most likely cause? A. avascular necrosis of the hip B. slipped capital femoral epiphysis C. septic arthritis of the knee D. transient synovitis

101. The answer is D [MSS/Rheumatology, Pediatrics, Infectious Disease]. A. Avascular necrosis presents with an antalgic gait, especially following activity. Hip motion is typically limited in abduction and internal rotation. There may be wasting of the gluteal muscles and a positive Trendelenburg test. Plain radiographs are the imaging tool of choice for diagnosis. B. Slipped capital femoral epiphysis typically affects adolescents, often those overweight or obese. These children present with pain, a change in gait, and range of motion abnormalities. C. Septic arthritis presents with fever and pain with localizing signs such as swelling, erythema, and warmth of the affected joint. With involvement of joints of the pelvis and lower extremities, limp or refusal to walk is often seen. D. Transient synovitis is most prevalent from 3 to 10 years of age, presenting 1 to 2 weeks following an upper respiratory infection. Children with this condition walk with a painful limp and often complain of pain in the anterior thigh. Treatment is symptomatic

102. Which type of leukemia is associated with the Philadelphia chromosome? A. acute lymphoblastic B. chronic lymphocytic C. acute myelocytic D. chronic myelogenous

102. The answer is D [Hematology, Oncology]. A. See D. B. See D. C. See D. D. The Philadelphia chromosome (BCR/ABL gene) results from reciprocal translocation between the long arms of chromosomes 9 (ABL) and 22 (BCR). It is most frequently seen in chronic granulocytic (myelogenous) anemia.

103. What is the most common type of hernia? A. direct inguinal B. indirect inguinal C. umbilical D. femoral

103. The answer is B [GI/Nutrition, Surgery]. A. Direct hernias are the most common type of acquired hernias. B. The overall majority of hernias are congenital indirect inguinal hernias. C. Umbilical hernias account for less than 3% of all hernias. D. Femoral hernias are more common in women, accounting for about 1/3 of total; they account for about 2% of hernias in men.

105. A 25-year-old inebriated man, who is otherwise healthy, presents to the ED with right upper chest pain after he fell against a coffee table. Chest x-ray shows a small pneumothorax. The patient is admitted for serial chest fi lms and observation. Which of the following is the most appropriate initial therapy for this patient? A. high-fl ow oxygen B. albuterol metered-dose inhaler C. intravenous prednisone D. prophylactic heparin

105. The answer is A [Pulmonology]. A. If the pneumothorax is small but the patient is mildly symptomatic or unlikely to cooperate with followup, admit the patient and administer high-fl ow oxygen. The resulting nitrogen gradient will speed resorption. B. There is no indication for albuterol with a traumatic pneumothorax. C. There is no need for an IV anti-infl ammatory drug in a small traumatic pneumothorax. D. Heparin is contraindicated, especially in the event that the pneumothorax does not spontaneously resolve and the need for chest tube placement arises.

106. A 17-year-old male college student complains of left ear pain. Physical exam reveals tenderness on palpation of the tragus. Otoscopic examination reveals the tympanic membrane to be intact, gray, and mobile without bulging or retraction. Which of the following is the treatment of choice? A. amoxicillin (Amoxil) B. ofl oxacin solution (Floxin otic) C. carbamide solution (Murine, Debrox) D. pseudoephedrine (Sudafed)

106. The answer is B [EENT, Infectious Disease]. A. Amoxicillin is the treatment of choice for acute otitis media but not externa. B. Antibiotic ear drops are the treatment of choice for acute infl ammatory otitis externa. Solutions containing an aminoglycoside (neomycin, polymyxin) or a fl uoroquinolone are fi rst line. A topical corticosteroid is often added. C. Carbamide solutions are indicated for cerumen impaction to soften the cerumen. D. Pseudoephedrine may help to alleviate symptoms of congestion when otitis occurs in the setting of an upper respiratory tract infection and eustachian tube obstruction

107. A 25-year-old female presents with multiple chronic complaints involving joints, gastrointestinal tract, and headaches. You are the fi fth practitioner the patient has seen in the last 6 months; she brings her medical records to the appointment. The records include the appropriate medical workup for her complaints, and all results are negative. Which of the following is the most likely diagnosis? A. somatization disorder B. major depression C. conversion disorder D. obsessive-compulsive disorder

107. The answer is A [Psychiatry/Behavioral Medicine]. A. This patient meets all the criteria for somatization disorder, including age of onset, gender, and negative medical workup. B. Patients with depressive disorders or obsessive- compulsive behavior may present with somatic symptoms, but symptoms of each specifi c disorder will predominate. C. Conversion disorder is a monosymptomatic somatoform disorder. Defi cits mimic a neurologic or medical illness in which the etiology is psychological factors. D. See B.

108. A 38-year-old obese female presents with progressively worsening exertional dyspnea and a chronic cough productive of foul-smelling phlegm. She gives a history of recurrent pulmonary infections throughout her life. Examination reveals crackles heard throughout the lung fi elds and digital clubbing. What is the most likely diagnosis? A. bronchiectasis B. asthma C. emphysema D. chronic bronchitis

108. The answer is A [Pulmonology, Infectious Disease]. A. Bronchiectasis is an obstructive lung disease that causes a chronic productive cough with purulent and foul-smelling sputum. A physical fi nding of persistent crackles over the affected lung region(s) is a diagnostic confi rmation; clubbing confi rms chronic pathology. Congenital or acquired etiologies are known. B. Asthma typically presents with episodic dyspnea associated with wheezing and an intermittent cough. It is not associated with clubbing. C. Emphysema alone presents with cachexia and little or no sputum production. Both emphysema and chronic bronchitis can have decreased expiratory fl ow rates. A patient with emphysema will have hyperresonant percussive areas and decreased breath sounds over areas with emphysematous bullae. Clubbing is rare. D. Chronic bronchitis presents with a chronic productive cough, but the sputum is not usually large in quantity nor foul-smelling. Patients with COPD are usually older than the age of 50 years when they present with the previously mentioned complaints from long-term smoking

109. A 63-year-old male who smokes complains of insidious onset of vague epigastric pain over the past several months. He also complains of anorexia, diarrhea, malaise, and diffi culty sleeping. Labs reveal anemia, hyperglycemia, and elevated alkaline phosphatase and bilirubin. What is the most likely description of the pain? A. cramping quality after eating B. relief with sitting and leaning forward C. radiation to the left shoulder D. referral to the right lower quadrant

109. The answer is B [GI/Nutrition, Surgery, Oncology]. A. Postprandial pain, especially after a fatty meal, more likely indicates pathology of the gallbladder. B. Pain of pancreatic cancer may be lessened by sitting up and leaning forward. This indicates that the lesion has spread beyond the pancreas and is inoperable. C. Pancreatic carcinoma often radiates to the back. Radiation to the right shoulder is common in gallbladder disease. D. Pain that refers or moves to the right lower quadrant most likely indicates appendicitis.

110. A 64-year-old thin white female undergoes DEXA scan. Her T score is _2.3 SD (standard deviations). How frequently should this test be repeated to monitor her status? A. every 1 to 2 years B. every 3 to 5 years C. every 5 to 7 years D. every 7 to 10 years

110. The answer is A [MSS/Rheumatology]. A. The current recommendations for frequency of DEXA scans for postmenopausal women are based on their T score: T score of -1.0 to -1.5: every 5 years T score of -1.5 to -2.0: every 3 to 5 years T score of greater than -2.0: every 1 to 2 years B. See A. C. See A. D. See A.

111. A patient is brought into the emergency room appearing quite ill. He has a fever of 103.2°F, dry skin and oral mucosal membranes, and abdominal distention and tenderness. His medical history is signifi cant for ulcerative colitis. Given his history and physical exam fi ndings, which of the following would be highest on the differential diagnosis? A. ileitis B. mesenteric ischemia C. small bowel obstruction D. toxic megacolon

111. The answer is D [GI/Nutrition]. A. Ileitis or ileocolitis is common in Crohn disease and manifests as nonbloody diarrhea. B. Mesenteric ischemia causes pain that is out of proportion to physical exam fi ndings, which are typically minimal. C. Small bowel obstruction causes abdominal distention and tenderness along with nausea and vomiting, typically of fecal material. A fever is unusual. D. Fulminant colitis and toxic megacolon are complications of severe ulcerative colitis. Patients have a more fulminant course with rapid progression of symptoms over 1 to 2 weeks and signs of severe toxicity. These patients appear quite ill, with fever, prominent hypovolemia, hemorrhage requiring transfusion, and abdominal distention with tenderness. Patients with active ulcerative colitis are at a higher risk of perforation or development of toxic megacolon and must be followed closely.

112. A 32-year-old with chronic asthma presents to the emergency department with an acute exacerbation. As you are completing the initial intake, she stops talking and her lips take on a bluish hue. Lungs are silent. What should the next course of action be in this case? A. Give dexamethasone IM STAT. B. Give oxygen with a nonrebreather mask. C. Prepare to intubate. D. Start nebulized albuterol.

112. The answer is C [Pulmonology]. A. Once the patient is stabilized respiratory wise, dexamethasone IM or IV is appropriate. B. Oxygen via mask or nasal canula cannot readily enter the airways as they are completely obstructed—only intubation will be of value in this case. C. The patient is in status asthmaticus and must be intubated immediately. D. Nebulized albuterol will be helpful, but fi rst the patient must be intubated.

113. A 30-year-old male complains of insidious yet progressive malaise, fever, weight loss, and dyspnea, which do not improve after three visits to primary care. He has been treated with a course of macrolide antibiotics and supportive measures. Lungs are clear. Chest radiography reveals bilateral hilar and right paratracheal adenopathy. Erythrocyte sedimentation rate is elevated; WBC count is low. What is the recommended treatment? A. corticosteroid B. cyclosporin C. minocycline D. thalidomide

113. The answer is A [Pulmonology]. A. Corticosteroids are considered fi rst-line therapy for the treatment and suppression of sarcoidosis of the lungs. B. Cyclosporin has been tried and is useful in a number of cases, but steroids are still considered the initial therapy. C. Minocycline is most useful in cases of cutaneous sarcoid disease. D. Thalidomide has been proven useful but has a number of side effects and has limited availability in the United States.

114. A 52-year-old obese Caucasian male with a history of chronic gastroesophageal refl ux presents with increasing dysphagia. He describes pain on swallowing and occasional regurgitation. He continues to smoke one pack of cigarettes daily, as he has for the past 30 or more years. He also enjoys a 4-oz glass of whiskey three times weekly. On examination, his weight is down 10 lb from a visit 1 month ago. What is the most likely diagnosis? A. Barrett esophagus B. esophageal cancer C. gastric adenocarcinoma D. infectious esophagitis

114. The answer is B [GI/Nutrition, Oncology]. A. Barrett esophagus is more common in obese white men; its incidence increases with age and in patients with persistent GERD. Progressive dysphagia is a sign of more severe disease. Barrett is present in about 10% of patients with chronic GERD. Weight loss is not common. B. Progressive dysphagia and weight loss of short duration are the initial symptoms in the vast majority of patients with esophageal cancer. Dysphagia initially occurs with solid foods and gradually progresses to include semisolids and liquids. By the time these symptoms develop, the disease is usually incurable, because diffi culty in swallowing does not occur until greater than 60% of the esophageal circumference is infi ltrated with cancer. C. Gastric cancers, when superfi cial, usually produce no symptoms. As the tumor becomes more extensive, patients may complain of insidious upper abdominal discomfort. Anorexia, often with slight nausea, is common but is not the usual presenting complaint. Weight loss may eventually be observed, and nausea and vomiting are particularly prominent with tumors of the pylorus. Dysphagia and early satiety may be the major symptoms. D. Patients with infectious esophagitis may complain of an acute onset of chest pain, odynophagia, and dysphagia. Weight loss is not associated.

115. A 38-year-old female G4P3003 at 27 weeks' gestation has a low hemoglobin and hematocrit and a serum ferritin of 10 _g/L. She admits to palpitations and dyspnea on exertion and greater fatigability than she remembers from prior pregnancies. What additional fi nding is most likely in this patient? A. paresthesias B. pica C. pruritus D. purpura

115. The answer is B [Hematology, Obstetrics/Gynecology]. A. Paresthesias are common in anemia due to vitamin B12 defi ciency. B. Pica is fairly common in iron defi ciency anemia. Ice chips, chalk, or specifi c foodstuffs are the common cravings. C. Pruritus is common in renal failure. D. Purpura typically signifi es a clotting defi ciency.

116. A 32-year-old female who is 6 weeks postpartum complains of double vision that appears midmorning and worsens throughout the day. She has been easily fatigued and has had diffi culty swallowing. Examination reveals ptosis and limited EOM movement. Pupillary refl exes and deep tendon refl exes are normal. Which of the following tests would best confi rm the suspected diagnosis? A. acetylcholine receptor antibodies assay B. cerebrospinal analysis for elevated protein C. muscle tissue biopsy D. plasma electrophoresis

116. The answer is A [Neurology]. A. Up to 90% of patients with myasthenia gravis will demonstrate elevated acetylcholine antibodies in their circulation. Onset of symptoms after pregnancy or a stressful event is not unusual. Response to short- acting acetylcholine is also used to confi rm the diagnosis. B. Evaluation of cerebrospinal fl uid, muscle tissue biopsy, or plasma electrophoresis will not be helpful in the diagnosis of myasthenia gravis. C. See B. D. See B.

117. A 72-year-old female presents with fatigue, dyspnea, and angina. Physical exam reveals pale conjunctiva, a grade II pulmonary fl ow murmur, glossitis, and decreased vibratory sensation in the lower extremities. CBC shows macrocytic indices and pancytopenia with macro-ovalocytes and hypersegmented neutrophils. Reticulocyte count is less than 2%. What is the most likely diagnosis? A. vitamin B12 defi ciency B. defect in hemoglobin synthesis C. folate defi ciency D. iron defi ciency

117. The answer is A [Hematology]. A. The clinical picture and macro-ovalocytes and hypersegmented neutrophils suggest vitamin B12 defi ciency. B. A defect in hemoglobin synthesis is consistent with thalassemia, which is a microcytic anemia; peripheral smear would show acanthocytes and target cells. C. Folate defi ciency also causes a macrocytosis and hypersegmented neutrophils but not peripheral neuropathy. D. Iron defi ciency can also cause a pulmonary fl ow murmur, but it is associated with a microcytic anemia.

118. As a general rule, sutures in the face and neck should be removed in how many days? A. 1 to 2 days B. 4 to 5 days C. 7 to 8 days D. 10 to 14 days

118. The answer is B [Dermatology, Surgery]. A. See B. B. Facial skin sutures should not be left in longer than 4 to 5 days. C. See B. D. See B.

119. A 32-year-old male presents to the emergency department complaining of unilateral scrotal swelling with pain radiating to the ipsilateral groin. Examination reveals exquisite left testicular tenderness, scrotal edema, and erythema of the overlying skin. Urinalysis is normal. What is the most likely diagnosis? A. Fournier gangrene B. hydrocele C. orchitis D. prostatitis

119. The answer is C [GU/Renal, Infectious Disease]. A. Fournier gangrene is a bacterial infection of the skin resulting from an infected abrasion or wound. Severe pain, crepitus, and a darkened and malodorous area are characteristic. B. A hydrocele is a fl uid-fi lled sac around the testicle, which causes edema of the scrotum. Many are congenital but it may be acquired through infl ammation or injury. Hydroceles are typically painless. C. Unilateral painful testicular swelling associated with edema and infl ammatory changes is typical for orchitis. It more commonly occurs in younger patients. Causes include mumps virus or enteroviruses. Doppler ultrasonography or nuclear scintigraphy will differentiate orchitis from testicular torsion. D. Prostatitis causes pain which is worsened with urination or ejaculation. Exam reveals perineal pain.

120. What is the most common pathogen implicated in acute osteomyelitis after total joint replacement? A. Enterococcus B. Pseudomonas C. Streptococcus D. Staphylococcus

120. The answer is D [MSS/Rheumatology, Surgery, Infectious Disease]. A. See D for explanation. B. Pseudomonas aeruginosa is frequently the causative organism in puncture wounds. C. See D for explanation. D. The most common pathogen causing postoperative osteomyelitis is Staphylococcus aureus. Other causes include Enterobacter spp. or Streptococcus spp.

121. A 25-year-old male presents with fatigue and weakness. Physical exam reveals temperature of 99.4°F, purpura, pallor, and traces of gingival bleeding. There is no hepatosplenomegaly. A CBC shows normochromic normocytic anemia, granulocytopenia, and thrombocytopenia. What is the most likely diagnosis? A. sideroblastic anemia B. aplastic anemia C. acute leukemia D. hemolytic anemia

121. The answer is B [Hematology]. A. Sideroblastic anemia would not present with purpura, gingival bleeding, or thrombocytopenia. It has no symptoms outside that of anemia. B. This is the classic presentation of aplastic anemia: a pancytopenia due to bone marrow failure. Patients experiencing idiopathic aplastic anemia are at high risk to develop blood neoplasia. C. Acute leukemia has similar symptoms but also typically presents with splenomegaly. D. A patient with hemolytic anemia would have splenomegaly but not purpura, pallor, or gingival bleeding.

177. The crystals of pseudogout are composed of what substance? A. calcium-containing salts B. monosodium urate monohydrate C. calcium pyrophosphate D. calcium oxalate

177. The answer is C [MSS/Rheumatology]. A. Calcium-containing salts are specifi c for chondrocalcinosis. B. Gout is composed of monosodium urate monohydrate crystals. C. Pseudogout is composed of calcium pyrophosphate crystals. D. Calcium oxalate is the composition of one of the major types of urinary stones.

122. A 70-year-old female reports recurrent episodes of dyspnea, heaviness in the chest, and weakness for 3 days. Cardiac markers are elevated and she is admitted. Which of the following tests would be best to evaluate for reinfarction during her hospitalization? A. CK-MB B. LDH C. troponin I D. troponin T

122. The answer is A [Cardiovascular, Geriatrics]. A. CK-MB typically returns to normal within 24 to 48 hours of an infarction; any further elevation is indicative of reinfarction. B. LDH is nonspecifi c and, therefore, nonspecifi c for recurrent injury. C. Troponin I remains elevated 5 to 10 days following infarction. D. Troponin T remains elevated 5 to 14 days following infarction.

123. What is the most common cause of obstruction of the small intestine in adults? A. hernia B. adhesion C. neoplasm D. gallstones

123. The answer is B [GI/Nutrition, Surgery]. A. Hernias cause small bowel obstruction approximately 10% of the time. B. Adhesions cause 60% of all mechanical small bowel obstructions in adults. C. Neoplasms are responsible for approximately 20% of all small bowel obstructions. D. Gallstone ileus is a rare cause of mechanical obstruction of the small bowel and occurs more frequently in females with an average age of 70 years.

124. A 35-year-old female presents with fever, petechiae, and headache. A companion reports exhibiting two episodes of confusion. Labs reveal marked anemia, thrombocytopenia, and reticulocytosis. PT and PTT are normal; LDH, BUN, and creatinine are elevated. Peripheral smear shows fragmented RBCs with schistocytes and helmet cells. What is the most likely diagnosis? A. disseminated intravascular coagulation (DIC) B. immune thrombocytopenic purpura (ITP) C. thrombotic thrombocytopenic purpura (TTP) D. von Willebrand disease

124. The answer is C [Hematology]. A. DIC will present with prolonged PT and PTT and increased fi brin degradation products. B. ITP is an autoimmune disorder against the platelets and usually occurs after an upper respiratory infection or in association with connective tissue disease. Clinically, it causes mucosal or skin bleeding. Peripheral smear is normal. C. The clinical picture of petechiae, renal involvement, and microangiopathic anemia with schistocytes and helmet cells is indicative of TTP. Neurologic abnormalities range from mild disorientation to severe focal defi cits, seizures, coma, or death. Hemolytic anemia and elevated LDH strongly suggest TTP. D. von Willebrand disease presents with normal PTT and a normal smear. It is a platelet adhesion problem due to a lack of von Willebrand factor. Prolonged bleeding time is typical. Patients are typically asymptomatic unless they undergo major surgery or trauma.

125. A 48-year-old female presents for annual gynecologic examination. External inspection reveals several areas of low, irregularly outlined, fl at-topped, white, well-defi ned plaques on the perineum and labia majora. The patient states she has had no symptoms and previous clinicians have never mentioned anything abnormal to her. What is the recommended management at this time? A. endometrial biopsy B. Pap smear C. Schiller test D. vulvar biopsy

125. The answer is D [Ob/Gyn, Dermatology]. A. An endometrial biopsy evaluates the uterine endometrium, not the vulva. B. Pap smears evaluate the cervix, not the vulva. C. The Schiller test evaluates the cervix after an abnormal Pap smear. Iodine staining highlights areas of rapid cell turnover. D. A vulvar lesion, especially one with a change in color, needs to be biopsied in order to make an accurate diagnosis. This must be differentiated from chronic vulvar dermatitis.

126. A 32-year-old homemaker complains of anterior shoulder and arm pain which worsens when lifting her child or pushing large items. Examination reveals tenderness most pronounced 3 inches below the anterior acromion. Which of the following is also likely to be found on examination of this patient? A. inability to abduct the shoulder greater than 30 degrees B. pain with forearm supination against resistance C. pain with forearm extension resistance D. tingling with percussion of the anterior wrist

126. The answer is B [MSS/Rheumatology]. A. Inability to abduct the shoulder indicates nerve damage. B. Palpation of the tendon within the bicipital groove reproduces the intense pain of bicipital tendonitis. Forearm supination, one of the main actions of the biceps, will also reduce pain, especially when resistance is applied. C. Pain with resistance against forearm extension is associated with medial epicondylitis. D. Tinel sign is indicative of carpal tunnel syndrome, a disorder of the wrist

127. A 58-year-old female is brought to the emergency room for recurrent episodes of severe right upper quadrant pain that lasts for hours. Ultrasonography has failed to show any abnormality on previous evaluations. She has a fever of 103.5°F and appears jaundiced. She is morbidly obese at 295 lb. Which of the following would be the most appropriate next step? A. abdominal plain fi lm B. colonoscopy C. endoscopic retrograde cholangiopancreatography (ERCP) D. HIDA scan

127. The answer is C [GI/Nutrition, Surgery]. A. Plain x-rays have low yield because most gallstones are radiolucent. B. This patient is experiencing upper GI symptoms. C. The combination of pain, fever (and chills), and jaundice represents Charcot triad. Abdominal ultra sonography, helical CT, and MR cholangiography are accurate in demonstrating common duct stones. Ultrasonography may lead to false-negative results in patients who are morbidly obese. ERCP provides the most direct and accurate means of determining the cause, location, and extent of obstruction. If a stone or the presence of cholangitis is suspected, ERCP is the procedure of choice because it permits sphincterotomy with stone extraction or stent placement. D. See C.

128. A local nursing home has reported that six patients have developed high fever, chills, headache, dry cough, and myalgia. Several other patients have not received the annual infl uenza vaccine due to religious preference. What is the recommended treatment for the unimmunized residents? A. Administer a single dose of amantadine followed by rimantadine for 5 days. B. Administer oral dose of oseltamivir daily for 7 days. C. Increase oral intake and supplement with high doses of vitamin C. D. Isolate the residents because no effective drugs are available.

128. The answer is B [Pulmonology, Infectious Disease]. A. Amantadine and rimantadine are no longer recommended as the mainstay of treatment or prophylaxis against infl uenza due to the high rates of resistance. B. Oseltamivir or zanamivir is recommended as both treatment and prophylaxis in high-risk groups in times of outbreaks. C. Fluids are recommended but are not curative. The value of vitamin C in prevention of the common cold is debatable; there is no evidence of its role in infl uenza. D. The residents have likely been exposed; isolation is no longer warranted.

129. A 28-year-old female aerobics instructor complains of intermittent chest pain for the past 2 months. The pain is sharp, lasts several hours, and has no apparent relationship to meals, activity, or stress. Physical exam is remarkable only for auscultation of a midsystolic click. Which of the following is the most likely diagnosis? A. esophageal refl ux B. angina pectoris C. mitral valve prolapse D. costochondritis

129. The answer is C [Cardiovascular]. A. Esophageal refl ux may present with pain similar to that of cardiac origin but is not associated with a midsystolic click. B. Angina pectoris is generally brought on by increased activity and relieved by rest. C. A midsystolic click and atypical chest pain in a young woman are characteristic of mitral valve prolapse. D. Costochondritis presents with chest pain and point tenderness on the chest wall.

130. A 32-year-old male presents with complaints of recurrent, episodic vertigo lasting up to 8 hours per episode for 6 months. The vertigo is associated with unilateral, low-frequency hearing loss and tinnitus. Caloric testing reveals impairment of thermally induced nystagmus on the involved side. What is the most likely diagnosis? A. labyrinthitis B. multiple sclerosis C. acoustic neuroma D. Ménière syndrome

130. The answer is D [EENT]. A. Labyrinthitis causes a continuous vertigo; patient may complain of hearing loss and tinnitus, which typically last days to a week and are associated with a recent upper respiratory infection. B. Multiple sclerosis is more common in women and may present with episodic vertigo, unilateral hearing loss, and chronic imbalance. C. Acoustic neuroma causes central audio-vestibular symptoms, progressive hearing loss, and speech deterioration. D. Ménière syndrome is a disorder of the endolymphatic compartment with episodic vertigo, unilateral lowfrequency sensorineural hearing loss, and tinnitus.

131. A 25-year-old female undergoes echocardiography for nonspecifi c chest pain and palpitations. Results indicate a fl oppy mitral valve. What fi nding was most likely present on physical exam? A. fi xed split S2 B. midsystolic click C. late diastolic rumble D. early systolic ejection sound

131. The answer is B [Cardiovascular]. A. A fi xed split S2 is common in atrial septal defect. B. A midsystolic click is the hallmark of prolapse of the mitral valve. The click may be followed by a mid to late systolic murmur, indicating mitral valve regurgitation. C. A late diastolic murmur is heard in mitral or tricuspid stenosis. D. Early systolic ejection sounds are common in stenosis of the aortic (louder at the apex) or pulmonic (loudest at left sternal border) valves.

132. A 63-year-old male presents with exacerbation of exertional dyspnea and peripheral edema. He has a long history of poorly controlled hypertension and a 50 or more pack-year history of smoking cigarettes. He has been hospitalized several times in the past with similar symptoms. Where is the apical pulse most likely located in this patient? A. along the left axillary border B. high in the epigastric region C. in the right second intercostal space D. under the left nipple

132. The answer is A [Pulmonology]. A. Chronic heart failure leads to cardiomegaly, which displaces the apical impulse laterally. B. In patients with emphysema, hyperinfl ated lungs may cause the apical impulse to be displaced to the upper epigastric region. C. The right second interspace overlies the aorta, which is not usually affected in chronic heart failure. D. Under the left nipple is too vague an area, as the nipple may vary in its location depending on breast size.

133. A 55-year-old smoker presents with a chronic cough. Chest x-ray reveals a 2-cm solitary perihilar nodule. Which of the following would provide the most useful information? A. bronchoscopy with biopsy B. computed tomography C. magnetic resonance D. sputum cytology

133. The answer is A [Pulmonology, Oncology]. A. Biopsy is needed to evaluate a pulmonary nodule for malignancy, unless the lesion is calcifi ed or has been stable for years. New lesions greater than 0.5 cm carry a higher rate of malignancy, especially in a smoker. Perihilar nodes are also accessible by bronchoscopy. Alternatively, a transthoracic fi ne-needle aspiration could be done to obtain tissue. B. Computed tomography is helpful to assess extent of the mass and will detect calcifi cation within the mass, but it will not defi nitively differentiate benign from malignant nodules. C. MRI is an excellent imaging study to depict the characteristics of nodule size, shape, margins, exact location, and presence of satellite lesions. However, it cannot defi nitively depict cell type or malignancy. D. Sputum cytology is rarely diagnostic in extrabronchial pulmonary nodules.

134. What is the normal respiratory rate in the newborn? A. 20 to 30/min B. 30 to 60/min C. 50 to 70/min D. 60 to 90/min

134. The answer is B [Pulmonology, Pediatrics]. A. See B. B. The normal respiratory rate in the newborn is 30 to 60 bpm. C. See B. D. See B.

135. A 26-year-old female presents with a rash consisting of hypopigmented macules and papules with fi ne scales located on the lower back and abdomen. Which of the following laboratory fi ndings is consistent with the most likely diagnosis? A. elevated white blood cell count with eosinophilia B. multinucleated giant cells C. large blunt hyphae with budding spores D. grapelike clusters of Gram-positive cocci on microscopy

135. The answer is C [Dermatology, Infectious Disease]. A. Tinea versicolor is a superfi cial skin infection that should not cause an elevated white blood cell count. B. Multinucleated giant cells on a Tzanck smear preparation of a skin lesion are seen with herpes and varicella zoster skin infections. C. Tinea versicolor is a yeast infection caused by Malassezia furfur. A scraping of the lesion placed on a slide with 10% KOH should reveal the classic large hyphae with budding spores ("spaghetti and meatballs") on microscopic examination. D. Gram-positive cocci in grapelike clusters on a Gram stain of a skin lesion scraping is typical of impetigo caused by Staphylococcus aureus.

136. A 44-year-old overweight female presents with acute sharp epigastric pain 2 hours after a large meal. She vomited once. Which of the following examination fi ndings will most likely be found on this patient? A. arrested inspiration with deep palpation of the right upper quadrant B. nontender palpable gallbladder on palpation of the right upper quadrant C. referred pain to the left lower quadrant with right lower quadrant palpation D. elevated jugular venous pressure on compression of right upper quadrant

136. The answer is A [GI/Nutrition, Surgery]. A. Right upper quadrant abdominal tenderness with inhibition of inspiration on palpation constitutes a Murphy sign, indicative of cholecystitis. B. A painless palpable gallbladder in a jaundiced patient suggests malignancy. This sign is often referred to as Courvoisier law or sign. C. Referred pain from RLQ to LLQ indicates peritonitis. D. Positive hepatojugular refl ux indicates portal hypertension.

137. A 22-year-old with sickle cell anemia presents to the emergency department with acute pain and swelling of the lower tibia. He is febrile and lethargic and reluctant to move the leg. CT scan shows cortical bone destruction. Which of the following antibiotic regimens is indicated? A. ceftriaxone 1 g IV daily for 48 hours B. ciprofl oxacin 400 mg IV three times a day for 6 weeks C. itraconazole 200 mg po twice a day for 6 months D. tetracycline 250 mg po three times a day for 3 months

137. The answer is B [MSS/Rheumatology, Infectious Disease, Hematology]. A. Ceftriaxone IV is indicated for gonococcal arthritis until improvement is noted, then exchanged for oral antibiotics. B. Staphylococcus aureus is a common cause of osteomyelitis in patients with sickle cell anemia. Suspected cases should be treated with a fl uoroquinolone for 4 to 6 weeks. It may be combined with rifampin. Salmonella is also a possible cause but less likely than Staphylococcus aureus. Treatment is the same. C. Itraconazole is the treatment of choice for bone and joint coccidioidomycosis. D. Tetracycline is not indicated in the treatment of osteomyelitis caused by Staphylococcus aureus. Treatment is the same.

138. A 24-year-old presents complaining of malodorous, frothy, thin, gray vaginal discharge. Clue cells are present on wet mount. What is the best treatment? A. miconazole vaginal suppository for one dose B. miconazole cream for 7 days C. metronidazole po for one dose D. metronidazole po for 7 days

138. The answer is D [Reproductive, Infectious Disease]. A. Miconazole vaginal suppository or cream is used to treat vaginal candidiasis, not bacterial vaginosis. B. See A. C. See D. D. Metronidazole is the treatment of choice for bacterial vaginosis, but the recommended regimen is 500 mg po twice a day, for 7 days, not the single 2-g oral dosing. The single dosing is more appropriate for Trichomonas vaginalis.

139. A 35-year-old overweight female presents to the clinic complaining of tender infl ammatory nodules and abscesses in her axillae and anogenital area. The lesions have waxed and waned over the past few years but have become more painful and bothersome in the past month. Some of the larger lesions are draining a purulent material. What is the most likely diagnosis? A. chronic chafi ng B. contact dermatitis C. drug reaction D. hidradenitis suppurativa

139. The answer is D [Dermatology, Infectious Disease]. A. Chafi ng is a result of chronic friction; common causes include obesity, tight clothing, and exercise. It presents with erythema and edema and, at times, excoriations. B. Contact dermatitis typically manifests as vesicles that weep and crust. C. A drug reaction can take many forms. It typically occurs within days of exposure. D. Hidradenitis suppurativa (acne inversa) is a disease of the apocrine glands (axilla, anogenital, and scalp). It affects females between puberty and menopause more often than males. Predisposing factors include obesity, history of acne, apocrine duct obstruction, and bacterial infection. There appears to be a genetic tendency. Abscesses tend to recur. Old scars and sinus tracts may be found.

140. A 4-year-old is brought to the offi ce by his mother because the day care teachers noticed he is unusually restless at school. The mother also noticed that he has not been sleeping well lately and has started wetting the bed at night. The child is alert and cooperative but scratches his buttocks while you are interviewing. Cellophane tape applied to the perianal area reveals football-shaped ova under the microscope. What is the medication of choice? A. fl uconazole (Difl ucan) B. itraconazole (Sporanox) C. mebendazole (Vermox) D. metronidazole (Flagyl)

140. The answer is C [Infectious Disease, GI/Nutrition, Pediatrics]. A. Fluconazole and itraconazole are antifungal medications. B. See A. C. Mebendazole, albendazole, or pyrantel is the treatment of choice for pinworm infections. Enterobius vermicularis is commonly diagnosed by identifying the eggs on cellophane tape applied to the perianal area. D. Effective against many other protozoans and bacteria, metronidazole is not a recommended treatment for pinworms.

141. A 14-year-old girl presents for a health maintenance visit. Her mother is concerned that her daughter has not begun to menstruate. The mother's menarche was at age 13 years. Medical history and complete physical exam are normal with breast development and pubic hair (Tanner 3). She began pubertal changes at the age of 12.5 years. What is the most appropriate course of action? A. chromosomal analysis B. laboratory evaluation including TSH and T4 C. reassurance that she likely will begin menstruating within a year D. urinary estriol determination

141. The answer is C [Endocrinology, Pediatrics]. A. Chromosomal analysis or urinary estriol determination may be useful in the evaluation of delayed or precocious puberty; however, this girl is within the normal range. B. Thyroid disease may cause delayed puberty; however, this girl is still within the normal range of development. C. Menarche typically happens 2 to 2.5 years after the onset of pubertal changes. The mean age of menarche is 12.75 years; however, there is a wide variation in this age and range. This girl is within the normal range and has appropriate physical changes and should be observed. D. See A

142. What is the most common site of pathology in Crohn disease? A. the proximal jejunum B. the terminal ileum C. Auerbach plexus D. rectosigmoid junction

142. The answer is B [GI/Nutrition]. A. See B. B. Regional enteritis, Crohn disease, can affect any site along the entire gastrointestinal tract. The terminal ileum is most commonly involved. C. See B. D. See B.

143. A 15-year-old boy presents for examination after being hit in the groin during a football game. Examination reveals the absence of his right testicle, with no masses, transillumination, or surgical scars noted on either side of his scrotum. His left testicle is present, and although tender to palpation, is of appropriate size. Which of the following is your greatest concern? A. hydrocele B. penile cancer C. spermatocele D. testicular cancer

143. The answer is D [GU/Renal]. A. A hydrocele is a nontender swelling of the scrotum due to fl uid accumulation in the tunica vaginalis. It will trans illuminate on examination. B. There are no physical fi ndings to suggest penile cancer which generally presents with a painless nonhealing ulcer on the shaft of the penis. C. A spermatocele is a cystic swelling of the epididymis. It will not be as large as a hydrocele but should transilluminate on examination. D. Testicles generally descend into the scrotum by 12 months of age. A "missing" testicle, with no sign of orchiectomy, could indicate cryptorchidism. Cryptorchidism is a risk factor for testicular cancer. It should also be noted that the descended testicle is also at increased risk of developing cancer.

144. A 36-year-old sustained a low back injury in a pickup basketball game. The pain has been somewhat relieved with overthe- counter topical analgesic patches and aspirin. Today, he presents with fatigue and darkened urine. He appears jaundiced. Labs reveal low hemoglobin and hyperbilirubinemia. Smear reveals Heinz bodies and bite cells. Which of the following recommendations should be given to this patient? A. Avoid known oxidant drugs. B. Obtain platelet transfusions as directed. C. Refrain from contact sports. D. Undergo elective splenectomy as soon as possible.

144. The answer is A [Hematology]. A. This patient likely has glucose-6-phosphate dehydrogenase (G6PD) defi ciency. All other choices are unnecessary as hemolytic episodes are self-limiting and the spleen is not the offending problem. Patients should be warned to avoid drugs, such as aspirin, that cause oxidative stress. B. See A. C. See A. D. See A.

145. A 65-year-old who presents with progressive, bilateral, central visual loss is found to have retinal drusen deposits on funduscopic examination. This fi nding is most consistent with which of the following diagnoses? A. cataract B. open-angle glaucoma C. macular degeneration D. vitreous hemorrhage

145. The answer is C [EENT]. A. Clinically, cataracts prevent the retina from being visualized completely and may cause obscuring of the red refl ex. B. Open-angle glaucoma causes slight cupping of the optic disc or changes in the retinal nerve fi ber layer. C. The precursor to age-related macular degeneration is age-related maculopathy, of which the hallmark is the development of retinal drusen. D. Vitreous hemorrhage is suspected with a sudden loss of vision, fl oaters, or bleeding within the eye.

146. When visual acuity is recorded, what does the numerator indicate? A. number of items in the chart line incorrectly identifi ed B. percentage of items in the chart line incorrectly identifi ed C. distance in feet of the patient from the eye chart D. distance in feet at which a patient with normal vision can read the chart line

146. The answer is C [EENT]. A. The number of items in the chart line incorrectly identifi ed is charted as "20/30-2" in which "-2" indicates that the patient missed two items. B. The percentage of items in the chart line incorrectly identifi ed is not a standard way of recording visual acuity. C. The numerator indicates the distance in feet at which a patient can read the full line in the chart correctly. D. The denominator indicates the distance in feet at which a patient with normal vision can read the full line in the chart correctly.

147. A 12-year-old is brought to the emergency department after an altercation at school. Examination reveals postauricular ecchymosis. What does this fi nding indicate? A. a basilar skull fracture B. a contrecoup head injury C. an epidural hematoma D. a depressed skull fracture

147. The answer is A [Neurology, Surgery]. A. The clinical signs of a basilar skull fracture include periorbital bruising (raccoon eyes), postauricular ecchymosis (Battle sign), and leakage of CNS fl uid from the ear or nose. B. A contrecoup head injury results from acceleration and deceleration trauma. It would not demonstrate Battle sign and is best evaluated with CT. C. An epidural hematoma usually results from laceration of a meningeal vessel. It would not demonstrate Battle sign, and diagnosis is based on CT. D. In a depressed skull fracture, the outer table of the skull on one side is depressed below the inner table of the other side. It is best evaluated with CT.

148. A patient presents with a spontaneous pneumothorax of 10% on the right with associated chest wall pain. There is neither dyspnea nor a history of pulmonary disease. Which of the following is the most appropriate management plan? A. simple aspiration, then outpatient bed rest B. outpatient treatment with bed rest C. thoracotomy tube D. pleurodesis

148. The answer is B [Pulmonology]. A. If surgical measures are used, such as aspiration or thoracotomy, the patient should be admitted. B. Outpatient treatment is adequate if there is less than 15% pneumothorax, no signs of tension pneumothorax, and no underlying lung disease. Air is reabsorbed from a small, spontaneous pneumothorax in a few days. C. A thoracotomy tube is useful for the treatment of large pneumothorax, tension pneumothorax, evidence of dyspnea or cyanosis, or underlying lung diseases. D. Pleurodesis is indicated in recurrent pneumothorax.

149. A 52-year-old postmenopausal woman develops chest pain suggestive of angina. Fasting labs reveal glucose of 92 mg/dL, total cholesterol of 271 mg/dL, LDL of 127 mg/dL, HDL of 31 mg/dL, and triglycerides of 375 mg/dL. An exercise stress test is positive. What is the best therapy for her dyslipidemia? A. atorvastatin (Lipitor) B. cholestyramine (Questran) C. estradiol D. niacin

149. The answer is A [Endocrinology, Cardiovascular]. A. Atorvastatin (Lipitor) will decrease triglycerides and LDL while increasing HDL without the side effects associated with niacin and cholestyramine. B. Cholestyramine (Questran) has insignifi cant effects on the HDL level as well as increased GI distress such as constipation and gas. C. Estradiol is not indicated for the treatment of hyperlipidemia. D. Niacin is able to lower triglycerides and LDL while increasing HDL; however, many people cannot tolerate the side effect of hot fl ashes.

150. A worried mother brings her 3-year-old son to the emergency department with fever of 104°F, sore throat, and refusal to eat. The parents are migrant farm workers who recently emigrated from Mexico. On physical exam, the patient is cyanotic and tachypneic and has obvious signs of respiratory distress, including inspiratory stridor and retractions. He is drooling. What is the fi rst step in the management of this patient? A. direct visualization of the patient's throat using a tongue blade and laryngoscope B. immediate protection of his airway with intubation or tracheostomy C. intravenous administration of penicillin and gentamicin D. radiography of the lateral neck with soft-tissue window

150. The answer is B [EENT, Infectious Disease, Pediatrics]. A. The diagnosis of epiglottitis is usually made on clinical grounds; direct visualization of the infl amed epiglottis should only be performed if acute intubation is available. B. Once the airway is secured, blood and throat cultures should be obtained. Administration of a second- or thirdgeneration cephalosporin is indicated for coverage of Haemophilus infl uenzae. Mass vaccination has led to decreased incidence of H. infl uenzae in developed countries. Other pathogens include group A Streptococcus, Streptococcus pneumoniae, Haemophilus parainfl uenzae, and Staphylococcus aureus. This child, from Mexico, may be unvaccinated. C. Treatment of epiglottitis, once the airway is secured, is second- or third-generation cephalosporins. D. Patients with suspected epiglottitis should be kept still and not transported. Patients without signs of impending respiratory failure can undergo portable lateral neck radiographs in the emergency department. The classic fi nding is a "thumbprint" representing the infl amed epiglottis.

151. A child has a history of brief lapses in consciousness without involuntary movements or autonomic components. The episodes begin and end abruptly. Which of the following type of seizure is this child most likely exhibiting? A. absence B. myoclonic C. simple partial D. atonic

151. The answer is A [Neurology, Pediatrics]. A. Absence seizure is characterized by brief impairment of consciousness with an abrupt beginning and ending. At times, involuntary movements may occur, but they are uncommon and the patient has no recollection and witnesses commonly miss them. B. Myoclonic seizures consist of single or multiple myoclonic jerks. C. Simple partial seizures manifest focal motor symptoms or somatosensory symptoms without loss of consciousness. D. Atonic seizures manifest as drop attacks—sudden loss of motor tone.

152. A 25-year-old female presents with paroxysmal episodes of palpitations and rapid heart rate that occur in no particular pattern. She has no current symptoms, and her ECG shows a sinus rhythm with a rate of 72 bpm. There are no pathologic Q waves and no ST-segment elevation or depression. The PR interval is 0.11 second with a delta wave at the onset of a slurred QRS complex. What is the most likely diagnosis? A. fi rst-degree AV block B. paroxysmal atrial tachycardia (PAT) C. Lown-Ganong-Levine syndrome D. Wolff-Parkinson-White syndrome

152. The answer is D [Cardiovascular]. A. The PR interval in fi rst-degree AV block is greater than 0.20 second. B. Paroxysmal atrial tachycardia (PAT) presents with paroxysmal palpitations and dyspnea. ECG reveals a P wave different from that of sinus rhythm but no other abnormalities. C. Although Lown-Ganong-Levine syndrome can cause supraventricular tachycardia, it is less common and has a short PR interval but no delta wave. D. The hallmark of Wolff-Parkinson-White syndrome is a short PR with a delta wave. It frequently causes palpitations.

153. A patient presents after several episodes of recurrent diarrhea over the past 2 weeks. He describes one to two bulky, foulsmelling stools daily, followed by several days of more frequent watery episodes, then with several days of apparent resolution. Other than mild fatigue and some mucus passed with the stools, there are no other symptoms. What organism is most likely responsible? A. Giardia lamblia B. Campylobacter jejuni C. Salmonella spp. D. Bacillus cereus

153. The answer is A [GI/Nutrition, Infectious Disease]. A. Giardia infection incubates for 1 to 3 weeks, causes foulsmelling bulky stools with few other symptoms, and may wax and wane over weeks before resolving. It is a parasite acquired through contaminated water. B. Campylobacter jejuni incubates over 2 to 10 days and results in fever and bloody diarrhea. C. Salmonella incubates in 8 to 48 hours causing fever, nausea, and diarrhea, with or without blood, for 3 to 5 days. D. Bacillus cereus incubates in less than 18 hours and causes vomiting initially followed by diarrhea, both of which resolve in 24 hours.

154. A 25-year-old female with a history of chronic alcohol abuse presents with mild hematemesis and melena. Examination reveals a slightly overweight female with erosion of the front teeth and enlarged salivary glands. Bowel sounds are normal; abdomen is soft and nontender. What is the most likely cause of the bleeding? A. erosive esophagitis B. erosive gastritis C. Mallory-Weiss tear D. portal hypertension

154. The answer is C [GI/Nutrition, Surgery]. A. Erosive esophagitis from GERD can be a cause of upper GI bleeding. It can cause severe esophagitis, may be asymptomatic, or present with dyspepsia. This patient's history is more suggestive of Mallory-Weiss tear. B. Erosive gastritis is a superfi cial process and is an unusual cause of signifi cant GI bleeding. More commonly, it presents as a chronic blood loss. C. Mallory-Weiss tears are lacerations of the gastroesophageal junction. Most patients have a history of retching, vomiting, or heavy alcohol use. This patient also shows signs of bulimia, which is characterized by binge-purge behavior. D. Esophageal varices can arise secondary to portal hypertension. Typically, they result in severe upper GI bleeding. The most common cause of portal hypertension is cirrhosis. Alcoholic cirrhosis carries a very high risk of upper GI bleeding, especially if the patient continues to consume alcohol.

155. An 8-month-old boy is brought to the rural clinic by his mother. He was born at home and has not been seen by a pediatrician. The child is irritable, is not eating well, and easily fatigues. He appears jaundiced. Fingerstick hemoglobin is 7.2 g/dL. Additional labs reveal an elevated reticulocyte count and a peripheral smear showing 20% nucleated RBCs/HPF and 10% sickled cells. Which of the following tests would be most appropriate to order next to confi rm the diagnosis? A. IgG autoantibodies B. hemoglobin electrophoresis C. Coombs test D. osmotic fragility

155. The answer is B [Hematology, Pediatrics]. A. IgG autoantibodies are useful in the diagnosis of autoimmune hemolytic anemia, which usually has spherocytes in the peripheral blood smear. B. In sickle cell anemia, the peripheral blood smear usually shows nucleated RBCs; greater than 5% sickled hemoglobin is highly suggestive of the disease. Confi rmation is by hemoglobin electrophoresis to quantify the percentage of hemoglobin S. C. A Coombs test assesses for autoantibodies that attach to RBCs and cause destruction resulting in an auto immune hemolytic anemia. D. Osmotic fragility indicates the presence of spherocytes and is useful in diagnosing spherocytosis but not sickle cell disease.

156. A 19-year-old male presents to the emergency department due to acute onset of fatigue, vomiting, diarrhea, and anorexia. He has postural dizziness, fever, and low blood pressure. Ten days ago, he had an acute nonbloody gastroenteritis that started abruptly and cleared after 48 hours. A chemistry panel shows decreased sodium, increased potassium, and a glucose of 50 mg/dL. What is the most likely diagnosis? A. adrenal insuffi ciency B. type 1 diabetes mellitus C. myxedema D. pheochromocytoma

156. The answer is A [Endocrinology]. A. Patients with adrenal insuffi ciency present with weight loss, fatigue, diarrhea, vomiting, anorexia, and salt cravings. They may have postural dizziness and pigmentation of buccal mucosa, palms, face, and extensor surface. In addition, hyponatremia, hyperkalemia, and hypoglycemia may be present. Adrenal insuffi ciency is considered a medical emergency. B. Patients with diabetes mellitus have nausea, vomiting, and hyperkalemia and would not have hypoglycemia. C. Patients with myxedema have weight gain and fatigue and may appear puffy. They may have hyponatremia and hypoglycemia. D. Patients with pheochromocytoma complain of fatigue, headache, inappropriate sweating, and may have orthostatic hypotension

157. A young man was hospitalized for several weeks following a motor vehicle accident. He was on mechanical ventilation for several days. Three weeks after discharge, he presents with shortness of breath and cough with an inability to clear secretions. Examination reveals coarse breath sounds throughout both lung fi elds. He is afebrile. What is the most likely cause? A. atelectasis B. pneumonia C. tracheal stenosis D. vocal cord paralysis

157. The answer is C [Pulmonology]. A. Atelectasis occurs in the immediate postoperative or postprocedure period. B. Primary symptoms of pneumonia include fever, cough, chest pain, and dyspnea. C. Tracheal stenosis develops secondary to trauma, especially endotracheal intubation. Patients develop progressive respiratory distress weeks or months after the trauma. D. Vocal cord paralysis, if caused by the trauma of intubation, is apparent almost immediately after extubation

158. An HIV-positive patient with a CD4 count of 174 cells/_L presents with a 4-week history of fever, shortness of breath which worsens with exertion, and a nonproductive cough. Chest x-ray reveals bilateral interstitial infi ltrates. What is the most likely diagnosis? A. histoplasmosis B. pneumococcal pneumonia C. pneumocystis pneumonia D. tuberculosis

158. The answer is C [Infectious Disease, Pulmonary]. A. Histoplasmosis is a fungal infection that mainly infects the lung in immunocompromised hosts. Chest x-ray may be normal or show diffuse changes. Diagnosis is by fungal culture. CD4 count is generally less than 100/L. B. Pneumococcal pneumonia is most commonly characterized by lobar consolidation, although it may also show diffuse infi ltrates. C. Pneumocystis pneumonia is most common in patients with cellular and humoral immune defects. The risk of pneumocystis pneumonia among HIV-infected patients rises markedly as circulating CD4 counts fall below 200 cells/L. Classic fi ndings on chest radiography in active disease consist of bilateral diffuse infi ltrates beginning in the perihilar regions. Prophylactic therapy with trimethoprim- sulfamethoxazole is warranted with CD4 count below 200 cells/L. D. Tuberculosis is characterized by granulomatous and cavitary lesions in the lung.

159. An otherwise healthy 70-year-old presents with a painful vesicular eruption spread throughout the left mid lower back. She has been taking 600 mg of ibuprofen every 6 hours with minimal relief. Which of the following would best confi rm the suspected diagnosis? A. Complete a Tzanck smear. B. Measure IgG for varicella. C. Obtain a fungal culture. D. Perform PCR of skin scraping.

159. The answer is D [Dermatology, Infectious Disease]. A. Tzanck smear is commonly done with suspected herpetic outbreaks, but it is less sensitive than a PCR. B. Immunoglobulin titers are not helpful in zoster. C. Fungal rash is not painful. D. Herpes zoster is typically a clinical diagnosis. If testing is deemed necessary, a polymerase chain reaction (PCR) or direct immunofl uorescence are the tests of choice.

160. A 14-year-old girl complains of itchy, watery eyes and clear rhinorrhea that has continued 3 days since completing an overnight spring camping trip. Examination reveals hyperemia with clear, stringy discharge. The papillae on the upper tarsal conjunctivae are enlarged. Which of the following is most important to include in patient management? A. saline irrigation B. cool compresses C. soft eye patch D. frequent hand washing

160. The answer is B [EENT, Infectious Disease, Pediatrics]. A. Irrigation with saline is recommended along with antibacterial drops in cases of purulent conjunctivitis as in bacterial infection. B. Cool compresses several times per day offers supportive therapy for symptomatic relief of allergic eye disease. Additional treatment includes use of any number of topical histamine H1 receptor antagonists. C. A soft eye patch is recommended for up to 24 hours in the treatment of corneal abrasions to protect the cornea until healing begins. Prolonged use of a patch will retard healing. D. Frequent hand washing is a general health recommendation for all but is most warranted in infectious causes of conjunctivitis, especially viral, to stop the cycle of reinoculation.

161. A 52-year-old presents for evaluation of decreased visual acuity, which he describes as a general blurring which has progressed over the past few months. Further questioning reveals increased diffi culty driving at night due to glaring lights. Which of the following pieces of historical information would lead to a consideration of cataracts as the cause of the visual changes? A. history of rubella at 5 years of age B. contact lens use for 25 years C. family history of senile cataracts D. inhaled corticosteroid use for asthma

161. The answer is D [EENT, Surgery]. A. Congenital rubella is associated with cataracts. B. Contact lens use has not been associated with an increased risk of cataract formation. C. Senile cataracts occur commonly after the age of 60 years; a family history confers no signifi cant increased risk over baseline. D. Risk factors for development of cataracts include congenital rubella, trauma, systemic disease such as diabetes, systemic or inhaled corticosteroid use, uveitis, and excessive sun exposure.

162. A 28-year-old presents with acute swelling of the cheek, which worsens with meals. Examination reveals an erythematous and edematous tender parotid duct; purulent material is expressed with massage. What is the most appropriate next step? A. fi ne-needle biopsy B. IV antibiotics C. lithotripsy D. supportive care

162. The answer is B [EENT, Infectious Disease]. A. Fine-needle biopsy is indicated in a patient with an asymptomatic salivary gland mass. B. Treatment of sialadenitis includes IV antibiotics (i.e., nafcillin) and increasing salivary fl ow (hydration, warm compresses, sialagogues, gland massage). C. Lithotripsy has been successful in the treatment of sialolithiasis. D. Supportive care alone is insuffi cient in sialadenitis.

163. Which of the following histories best describes a presentation consistent with multiple sclerosis? A. a 35-year-old male with an acute 4-day history of bilateral leg weakness beginning in the feet and now extending as far as the thighs B. a 60-year-old female with a 2-month history of slowly increasing dysphagia, weakness in the extremities, ptosis, and diplopia C. a 32-year-old female with a several month history of recurrent, brief episodes of extremity weakness and tingling, diplopia, and vertigo D. a 50-year-old male with a 3-month history of weakness, stiffness, and loss of muscle mass in the arms and legs

163. The answer is C [Neurology]. A. Guillain-Barré syndrome presents with acute onset and progression of symmetrical weakness usually beginning in the feet and progressing upward to eventually involve the whole body. B. Myasthenia gravis occurs at all ages; is insidious in onset; and causes weakness of ocular, facial, limb, and respiratory muscles. C. Multiple sclerosis is a disease with primary onset between 30 and 50 years of age characterized by recurrent episodes of sensory abnormalities, blurred vision, and weakness with or without spasticity. It is more common in females. D. Amyotrophic lateral sclerosis (Lou Gehrig disease) is an upper and lower motor neuron disorder with weakness, stiffness, and wasting but minimal sensory complaints

182. A 74-year-old with moderate to severe COPD presents for routine checkup. ECG reveals a heart rate of 120 bpm, three distinct P waves, and variable PP intervals. What is the most likely arrhythmia? A. atrial fi brillation B. atrial fl utter C. multifocal atrial tachycardia (MAT) D. wandering atrial pacemaker (WAP)

182. The answer is C [Cardiovascular]. A. Atrial fi brillation has no discernible P waves. B. Atrial fl utter has distinctive fl utter waves in place of P waves. C. MAT fi ts all three of the stated criteria. It is almost exclusively seen in patients with severe COPD. D. WAP is similar, but the rate must be less than 100 bpm.

164. A 23-year-old female who smokes 1/2 pack per day complains of a bluish lacy rash that appears on her lower extremities when exposed to the cold. The rash disappears once the area is rewarmed. What is the most likely etiology of this condition? A. atheromatous process of the superfi cial arteries B. infl ammatory process affecting small and medium arterioles C. vasomotor instability of dermal blood vessels D. venous spasm throughout the saphenous system

164. The answer is C [Cardiovascular, Dermatology]. A. Atherosclerosis results in peripheral arterial disease which manifests as intermittent claudication. B. Infl ammation of the small and medium arterioles causes thromboangiitis obliterans (Buerger disease). Clinically, it manifests with pain and ulcerations. C. Livedo reticularis is caused by vasomotor instability. It can be a benign phenomenon but may be an indication of vascular disease, in particular anti-phospholipid antibody syndrome. D. Venous spasm is the underlying cause of Raynaus phenomenon, which causes a white-blue-red (spasm, cyanosis, erythema and rewarming) pattern in the hands. It is also exacerbated by exposure to the cold.

165. Which of the following indicates viral replication and infectivity for a patient with chronic hepatitis B? A. HBsAg (_); anti-HBs (_) B. HBsAg (_); HBeAg (_) C. HBcAg (_); anti-HBe (_) D. anti-HBs (_); anti-HBe (_)

165. The answer is B [GI/Nutrition, Infectious Disease]. A. HBsAg (-) and anti-HBs (+) indicates normal response to vaccination (immunity). B. HBsAg (+) and HBeAg (+) indicates chronic hepatitis B with active viral replication. C. HBcAg alone does not appear in serum. It may be measured in liver tissue. D. Anti-HBs (+) and anti-HBe (-) indicates either chronic hepatitis B with heterotypic anti-HBs or recovery from hepatitis B

166. A patient is admitted to the hospital with an idiopathic spontaneous pneumothorax successfully treated with tube thoracostomy. The patient is interested to know if he will experience a recurrence of this disorder. What is the most accurate response? A. Recurrence occurs in up to 30% of patients and is treated by obliterating the pleural space. B. Recurrent spontaneous pneumothorax will heal more quickly with subsequent occurrences. C. He is more likely to develop a tension pneumothorax with subsequent occurrences. D. There is no data to support recurrence rates in idiopathic spontaneous pneumothorax.

166. The answer is A [Pulmonology, Surgery]. A. Spontaneous pneumothorax recurs in 30% of patients usually within 2 years. The majority are ipsilateral. Treatment in recurrent episodes is either administration of a noxious agent into the pleural space to obliterate the pleural space or partial pleurectomy with oversewing of apical blebs or abrasion of the pleural surface. B. There is no evidence to support that a recurrent spontaneous pneumothorax will heal more quickly. A patient experiencing a recurrent pneumothorax has a 70% to 80% chance of experiencing yet another event. C. A tension pneumothorax is more commonly seen in patients receiving mechanical ventilation or as a result of penetrating trauma. D. See A.

167. A 13-year-old has been coughing for 10 days. It began as a dry cough but has now become productive of small amounts of white sputum. He also complains of headache, sore throat, and a pressure feeling in his ears. Examination reveals injected pharynx, no cervical adenopathy, and scattered rales. What is the treatment of choice? A. ampicillin B. gentamicin sulfate C. erythromycin D. supportive therapy only

167. The answer is C [Pulmonology, Infectious Disease]. A. b-Lactam antibiotics such as ampicillin act by inhibiting cell wall synthesis. Mycoplasma pneumoniae is resistant to b-lactam antibiotics due to the lack of a cell wall. B. Aminoglycosides do not have a high degree of activity against Mycoplasma pneumoniae and must be given parenterally. They have signifi cant associated toxicities. C. Macrolides have highly specifi c activity against Mycoplasma pneumoniae. Although newer macrolides offer some advantages, erythromycin is effective and less expensive. D. This child likely has mycoplasmal pneumonia and would benefi t from treatment with a macrolide.

168. A 47-year-old male presents with worsening irritation of his hands that started as dryness and progressed to chapping and erythema and now has fi ssuring. He works as a janitor in the local hospital cleaning the operating rooms. His symptoms become less severe on his days off. What advice should be offered? A. Wash and dry hands well after exposure to cleaning agents. B. Switch jobs as the problem will not otherwise resolve. C. Use emollient creams several times per day. D. Wear appropriate protective clothing and equipment.

168. The answer is D [Dermatology]. A. Exposure to cleaning agents must be avoided or eliminated. B. Healing usually occurs within 2 weeks of removal of noxious stimuli; in more chronic cases, 6 weeks or longer may be required. In occupation irritant contact dermatitis, only one-third of individuals have complete remission and others may require allocation to another job. Atopic individuals have a worse prognosis. C. Emollient creams may offer some relief, but avoidance is key to management. D. Individuals exposed to irritant chemicals should wear protective clothing and equipment regardless of sensitivity.

169. A 23-year-old female is beginning treatment with isotretinoin (Accutane) for her cystic acne. What is the recommended frequency of serum pregnancy testing during treatment? A. one test prior to starting and one test monthly B. one test prior to starting and two tests monthly C. two tests prior to starting and one test monthly D. two tests prior to starting and two tests monthly

169. The answer is C [Dermatology]. A. Recommendations call for two serum pregnancy tests prior to starting treatment, not one. B. See C. C. Isotretinoin is highly teratogenic and therefore is absolutely contraindicated during pregnancy. Standard recommendations call for two serum pregnancy tests prior to initiating treatment and then one monthly for the duration of treatment. D. See C.

170. A 45-year-old female presents for evaluation of pain in both hands. Examination reveals symmetric distal interphalangeal joint involvement. The nails are noted to be ridged with associated pitting and onycholysis. Erythrocyte sedimentation rate (ESR) and serum uric acid are elevated. What is the most likely diagnosis? A. psoriatic arthritis B. reactive arthritis C. rheumatoid arthritis D. systemic lupus erythematosus

170. The answer is A [MSS/Rheumatology, Dermatology]. A. Psoriasis is characterized by silvery plaques on an erythematous base. Patients may develop joint disease. Nail pitting is common. Arthritis precedes or occurs simultaneously with skin disease in 20% of cases. B. Reiter syndrome (reactive arthritis) occurs after dysenteric or sexually transmitted disease. It is characterized by oligoarthritis, conjunctivitis, urethritis, and mucocutaneous lesions. C. Rheumatoid arthritis presents with stiffness and symmetric involvement of small joints. Subcutaneous nodules, vasculitis, and other extra-articular manifestations are common but not nail pitting. D. Systemic lupus erythematosus is characterized by fever, malaise, weight loss, and skin changes.

171. Which of the following ECG fi ndings is the most reliable indicator of myocardial ischemia during an exercise stress test? A. J-point elevation of the ST segment greater than 1 mm B. J-point depression of the ST segment greater than 1 mm C. a decrease in R-wave amplitude D. transient left-axis deviation

171. The answer is B [Cardiovascular]. A. Elevation of the ST segment is suggestive of an infarct. B. Depression of the ST segment greater than 1 mm is most reliable for ischemia. C. Changes in R wave height or axis deviation is not an indicator of myocardial ischemia. They are associated with cardiomegaly or ventricular strain. D. See C.

172. At the age of 1 year, a child's weight should be how much compared to birth weight? A. twice his or her birth weight B. three times his or her birth weight C. four times his or her birth weight D. fi ve times his or her birth weight

172. The answer is B [GI/Nutrition, Pediatrics]. A. See B. B. A child should double his or her birth weight by 6 months and triple it by 1 year of age. C. See B. D. See B.

173. After 3 days of amoxicillin, a patient discontinued treatment for a sinus infection. Five days later, he is brought to the emergency department due to headaches of several days' duration, decreased sensorium, and new-onset seizure activity. Examination reveals focal neurologic signs and a temperature of 100.4°F. Pertinent laboratory values include elevated ESR and PMN leukocytosis. Which of the following is the most likely diagnosis? A. brain tumor B. brain abscess C. cerebral infarction D. encephalitis

173. The answer is B [Neurology, Infectious Disease]. A. Brain tumors are usually not associated with recent infections or leukocytosis. B. A brain abscess usually presents with headache, focal neurologic defi cits, and seizures. Decreased sensorium, drowsiness, and confusion may also be present. Approximately 50% of patients have low-grade fever. Laboratory studies may show leukocytosis with a predominance of polymorphic nucleocytes and an elevated sedimentation rate. C. Cerebral infarction presents with acute focal motor and sensory losses. D. Encephalitis presents with fever, malaise, stupor, nausea and vomiting accompanied by stiff neck, meningeal signs, and possible convulsions. Focal neurologic signs are rarely seen.

174. The morning after an eating and drinking binge, a 39-yearold obese male presents with exquisite pain in the left ankle. Examination reveals a red, hot, swollen medial malleolus. He is afebrile. Which of the following is the treatment of choice? A. allopurinol (Zyloprim) B. indomethacin (Indocin) C. propoxyphene (Darvon) D. colchicine

174. The answer is B [MSS/Rheumatology]. A. Serum uric acid-lowering drugs are indicated for maintenance to reduce episodes. It may be helpful in acute arthritis not controlled by colchicine. B. NSAIDs are the treatment of choice for acute gout. Indomethacin and naproxen are acceptable choices. C. Propoxyphene is not an effective agent for acute gout. D. Colchicine is effective for acute gout but is less favored than NSAIDs due to side effects.

175. A 72-year-old with adenocarcinoma of the lung presents with headache, dizziness, visual loss, stupor, and near syncope. Which of the following is most likely on physical exam? A. bradycardia B. rhinophyma C. scattered rhonchi and wheeze D. swelling of the face and neck

175. The answer is D [Pulmonology, Oncology]. A. Patients with superior vena cava syndrome are typically tachycardic. B. Rhinophyma, soft tissue, and sebaceous hyperplasia of the nose are common in acne rosacea. C. Adenocarcinoma causes localized rhonchi and, rarely, wheeze. D. This patient has developed superior vena cava syndrome secondary to obstruction from the mediastinal mass. Swelling of the face and neck are characteristic as well as headache, dizziness, visual loss, stupor, and syncope. The tumor is likely located in the superior mediastinum.

176. A 27-year-old female presents with progressive unilateral loss of vision over the last 36 hours. She complains of mild pain in the eye, which worsens when performing extraocular movements. Examination reveals loss of color vision and a relative afferent pupillary defect. What is the most likely diagnosis? A. optic neuritis B. central retinal artery occlusion C. amaurosis fugax D. uveitis

176. The answer is A [EENT, Neurology]. A. Optic neuritis has a mild to profound visual loss that develops over hours to a few days with visual fi eld defects that are varied and common; extraocular movements often exacerbate the variable pain. Loss of color vision and a relative afferent pupillary defect are found on examination. The optic nerve is infl amed in about onethird of cases. Optic neuritis is strongly correlated with demyelinating diseases. B. Central retinal artery occlusion causes painless, total or near total "black-out" vision. Exam reveals retinal opacity distal to the occlusion. C. Amaurosis fugax causes a painless, monocular, transient visual loss lasting seconds to 30 minutes. D. Uveitis presents with blurred vision, deep aching pain, photophobia, and varying erythema

178. A 35-year-old female presents with wrist pain. Four days ago, she fell on an outstretched hand while rollerblading. She did not seek care immediately because there was no swelling or bruising. Now, she complains of lateral wrist pain that is most pronounced when gripping glasses or the car steering wheel. She has pain with ulnar deviation and tenderness over the snuffbox. Radiographs are normal. What is the most appropriate management at this time? A. elastic wrap with ice and analgesics B. immediate referral to orthopaedic surgery C. thumb spica cast with repeat radiographs in 2 weeks D. volar wrist splint with ice and analgesics

178. The answer is C [MSS/Rheumatology]. A. Suspected scaphoid fractures require immobilization. B. If an obvious fracture is evident on radiograph, it may be treated operatively with good outcomes and less need for immobilization. C. A scaphoid fracture results from a fall on an outstretched hand with forced dorsifl exion. Disruption of blood fl ow to the scaphoid bone may occur, resulting in avascular necrosis. Initial radiographs may be negative; accepted treatment of suspected cases is to apply a thumb spica cast and repeat fi lms after 2 weeks. D. A volar wrist splint will not provide the proper immobilization.

179. A patient has been diagnosed with schizophrenia that is resistant to conventional medication. He is started on clozapine. Which of the following laboratory values must be monitored weekly? A. white blood cells B. red blood cells C. platelets D. absolute lymphocytes

179. The answer is A [Psychiatry/Behavioral Medicine]. A. Agranulocytosis occurs in 1% to 2% of patients treated with clozapine. Risk is higher in those of Ashkenazi Jewish ancestry. Weekly white blood cell counts should be strictly monitored for the fi rst 6 months of treatment; if white blood cell count is stable, monitoring can be done less frequently after that. Weekly monitoring must resume when the drug is discontinued as well. B. Red blood cells are not affected. C. Platelets are not affected. D. PMNs, not lymphocytes, are affected.

180. A 42-year-old white female presents for well-woman examination. She has a BMI of 28 kg/m2. What additional risk factor would indicate a need for screening for type 2 diabetes mellitus at this time? A. blood pressure of 142/94 mm Hg B. chronic resistant dermatophytosis C. HDL cholesterol 52 mg/dL D. previous delivery of a small-for-gestational-age baby at 38 weeks

180. The answer is A [Endocrinology, Cardiovascular]. A. The American Diabetes Association and the U.S. Preventive Services Task Force recommend fasting blood glucose screening for all individuals older than the age of 45 years with blood pressure greater than 135/80 mm Hg. Screening in younger adults with BMI greater than 25 should be done in the presence of one additional risk factor. Risk factors include elevated blood pressure, family history of type 2 diabetes, non-Caucasian race, history of large for gestational age birth, low HDL, PCOS or acanthosis nigricans, history of vascular disease, and a history of impaired fasting glucose or impaired glucose tolerance. B. See A. C. See A. D. See A.

181. A 42-year-old female presents with intermittent joint pain that causes her to miss work at times. This joint pain began about a year ago involving primarily the joints in her hands, wrists, and feet. She is also concerned about her progressively worsening fatigue, muscle aches, and feelings of depression. Physical exam reveals tender, edematous bilateral wrists; painless oral ulcers; and erythematous maculopapular lesions on her face. What is the most likely diagnosis? A. systemic sclerosis B. osteoarthritis C. rheumatoid arthritis D. systemic lupus erythematosus

181. The answer is D [MSS/Rheumatology]. A. Scleroderma (systemic sclerosis) manifests as diffuse thickening of the skin with telangiectasias and pigment changes. Systemic symptoms include dysphagia, gastrointestinal dysmotility, and pulmonary fi brosis. B. Osteoarthritis typically is insidious in onset. Constitutional symptoms are unlikely. C. Rheumatoid arthritis (RA) is seen primarily as a subacute, symmetric polyarthritis. The joint manifestations are usually more persistent in RA than reported by this patient. D. Systemic lupus erythematosus (SLE) has various presentations. Characteristically, patients present initially with one or two symptoms such as fatigue, myalgias, and arthritis and later develop additional features of SLE. Maculopapular "butterfl y" rash is characteristic, although it manifests in only half of affected individuals.

183. A 40-year-old "weekend" athlete presents with 3 weeks of worsening hip pain. Examination reveals tenderness at the greater trochanteric region at the lateral hip which exacerbates with external rotation of the hip. Which of the following is the best initial therapy? A. aspiration B. NSAIDs C. antibiotics D. injectable steroids

183. The answer is B [MSS/Rheumatology]. A. Aspiration is indicated if infection is the suspected cause. B. NSAIDs are the fi rst line of therapy for trochanteric bursitis caused by trauma or overuse, as is likely the cause in this case. C. The presentation of bursitis due to infection includes acute onset of pain, fever, swelling, and redness. This is most common in the olecranon or prepatellar bursa. It is not consistent with this chronic presentation. D. Injectable steroids may be indicated after failure of NSAID therapy.

184. A 26-year-old male presents with four episodes of diarrhea per day, rectal bleeding, tenesmus, passage of mucus, and crampy abdominal pain for 1 week. He has experienced intermittent episodes of bloody diarrhea over the past 6 months. What is the most likely diagnosis? A. Crohn disease B. diverticulitis C. infectious diarrhea D. ulcerative colitis

184. The answer is D [GI/Nutrition]. A. The most common presentation of Crohn disease is a chronic history of recurrent episodes of right lower quadrant pain, diarrhea, and weight loss. The extent of diarrhea and presence of blood depends on the location of the pathology. B. Diverticulitis can be confused with Crohn disease clinically and radiographically. Both diseases cause fever, abdominal pain, and tender abdominal mass. C. Infections of the small intestine and colon can mimic Crohn or ulcerative colitis. They may be bacterial, fungal, viral, or protozoal in origin. Campylobacter colitis can mimic the endoscopic appearance of severe ulcerative colitis and cause a relapse of established ulcerative colitis. Salmonella can cause watery or bloody diarrhea, nausea, and vomiting. Shigellosis causes watery diarrhea, abdominal pain, and fever followed by rectal tenesmus and by the passage of blood and mucus per rectum. All three are usually self-limited. D. This patient presents with several symptoms of ulcerative colitis. Although ulcerative colitis can present acutely, symptoms usually have been present for weeks to months. Occasionally, diarrhea and bleeding are so intermittent and mild that the patient does not seek medical attention. Tenesmus indicates rectal involvement which is always present in ulcerative colitis.

185. A 23-year-old female presents with increasing cough for 3 weeks. The cough is worse at night and occurs in "waves" lasting 5 to 10 minutes and causing gasping inhalations. Posttussive vomiting has occurred on several occasions. The cough was preceded by mild cold symptoms. She completed all immunizations needed to begin public school and has no known drug allergies. What is the treatment of choice for her condition? A. erythromycin B. oral glucocorticoids C. supportive care D. trimethoprim-sulfamethoxazole

185. The answer is A [Pulmonary, Infectious Disease, Pediatrics]. A. Macrolide antibiotics are the drugs of choice for treatment of pertussis. The incidence of pertussis has grown, prompting a recommendation to consider booster vaccines every 10 years. B. Oral glucocorticoids have been advocated by some but have not proven effective in pertussis. C. Although a quiet environment can decrease the stimulation that can trigger paroxysmal episodes, antibiotics must be given to decrease the spread of pertussis. D. Trimethoprim-sulfamethoxazole is recommended as an alternative for individuals allergic to macrolides.

186. A 5-year-old is brought to the clinic by his mother due to productive cough, sore throat, and mild fever for the past week. Sputum Gram stain and smear is inconclusive. Chest x-ray shows a subsegmental infi ltrate. PCR is positive for Chlamydia. What is the recommended treatment? A. erythromycin B. penicillin C. tetracycline D. trimethoprim-sulfamethoxazole

186. The answer is A [Pulmonology, Infectious Disease]. A. A macrolide (erythromycin) or a tetracycline (doxycycline) is the treatment of choice for chlamydial pneumonia. B. Penicillin is effective against Pneumococcus but is less effective against Chlamydia. C. Tetracycline is effective but contraindicated in children secondary to staining of teeth and bones. D. Chlamydia is not susceptible to sulfa drugs.

187. A patient presents with concerns of thickening of fi ngers, oily skin, and coarsening voice, which has progressed over the past 1 to 2 years. Further questioning reveals shrinking testicles with loss of libido as well. Which of the following imaging tests is recommended? A. cerebral angiography B. CT of the head C. MRI of the head D. radiographs of the skull

187. The answer is C [Endocrinology, Neurology]. A. Acromegaly is not a vascular disorder. B. MRI is generally superior to CT scanning. C. The history is consistent with acromegaly which is caused by a pituitary tumor in 90% of cases. MRI is the preferred imaging modality. D. Radiography may show an enlarged sella and thickened skull, but this is not as diagnostic as an MRI.

188. After his best friend had a myocardial infarction, a 49-yearold male comes in for medical care for the fi rst time in many years. He is 72 inches tall and weighs 210 lb. His diet consists of a moderate amount of fat; he has never smoked, drinks alcohol only on rare occasions, and plays basketball or touch football on weekends. Blood pressure is 138/86 mm Hg. He thinks it would be easiest to embark on a program of more regular physical activity as a fi rst step to lifestyle modifi - cation. How much is his systolic blood pressure likely to drop if he is able to walk briskly for 30 minutes on most days of the week? A. 2 to 4 mm Hg B. 4 to 9 mm Hg C. 5 to 20 mm Hg D. 8 to 14 mm Hg

188. The answer is B [Cardiovascular]. A. Moderation of alcohol consumption to no more than two drinks per day results in a reduction of systolic blood pressure of 2 to 4 mm Hg. B. Engaging in regular aerobic exercise for 30 minutes most days of the week results in a reduction of systolic blood pressure of 4 to 9 mm Hg. C. Weight loss results in a reduction of systolic blood pressure of 5 to 20 mm Hg per 10 kg lost. D. Adopting a DASH diet results in a reduction of systolic blood pressure of 8 to 14 mm Hg.

189. A 63-year-old male presents with sudden onset of left-sided facial droop and inability to fully close his left eye or raise his left eyebrow. He states that his face feels stiff. What other complaint is commonly present in patients with this disorder? A. facial pruritus B. nostril fl aring C. periauricular pain D. scalp tenderness

189. The answer is C [Neurology]. A. Facial pruritus is not a common presenting symptom of Bell palsy. B. Nostril fl aring is associated with many pulmonary disorders but is not associated with Bell palsy. C. Patients with Bell palsy will often complain of pain about the ear, which precedes or accompanies the onset of facial weakness. D. Scalp tenderness is not commonly associated with Bell palsy, although it is a classic symptom of giant cell (temporal) arteritis.

190. A 35-year-old G1P0 presents to labor and delivery at 38 weeks complaining of headache and right upper quadrant pain. Blood pressure is 160/110 mm Hg; cervix is 1 cm dilated, 50% effaced, and at _2 station. Urine dipstick shows _3 protein and negative for ketones and leukocytes. Fetal heart tracing is reactive and reassuring. Which of the following is the best intervention at this time? A. cesarean section B. induction of labor C. more labs are necessary to determine management D. send home on bed rest

190. The answer is B [Ob/Gyn]. A. A cesarean section would be performed if induction of labor fails or if mom or fetus becomes unstable at any point. B. This patient has severe preeclampsia defi ned by elevated blood pressure and proteinuria during pregnancy. It most commonly occurs in primiparous women who are close to term (but can occur any time after 20 weeks' gestation). Preeclampsia can progress to eclampsia (seizures) and death so it is managed aggressively. The best management is delivery of the fetus. Because both mom and fetus are stable, induction of labor is warranted. C. Although more labs are indicated, they are not necessary in making the diagnosis, and waiting for their results could delay delivery of the fetus. D. Only women with mild preeclampsia with a " stable home situation" can be sent home on bed rest. This patient has severe preeclampsia, which always requires hospitalization.

191. A 68-year-old female presents after a syncopal episode that lasted less than 1 minute. She states she felt nauseous prior to losing consciousness. She has no signifi cant past medical history. Resting blood pressure is 132/84 mm Hg. ECG is normal. Which of the following diagnostic tests is indicated at this time? A. basal metabolic temperature B. electrophysiologic testing C. exercise stress testing D. tilt-table testing

191. The answer is D [Cardiovascular]. A. Basal body temperature is useful in assessing gynecologic fertility issues, not syncope. B. Electrophysiologic testing may be needed, but it is invasive and expensive; it should be reserved for a later date. C. Stress testing is useful to determine cardiac dysfunction especially in a patient with ischemia. She has no signs of ischemia (chest pain). D. The tilt-table test and event recorder placement are useful in diagnosing patients (especially middle-aged and older) with syncope due to vasovagal events, bradycardia, or hypotension. It should be done prior to invasive studies.

192. A 40-year-old professional comes to see you because his business partner is ready to dissolve their partnership due to his behavior. The patient is unmarried and lives alone. At work, he is very rigid and perfectionist. He can multitask easily, has great attention to detail, works very long hours, and expects others to do the same. He is often displeased with the work of others and has diffi culty delegating when he should. He has had a steady stream of assistants because it is so diffi cult to work for him. He almost never takes a vacation. This patient most likely has which personality disorder? A. avoidant B. narcissistic C. obsessive-compulsive D. paranoid

192. The answer is C [Psychiatry/Behavioral Medicine]. A. Avoidant personality disorder is characterized by social inhibition, feelings of inadequacy, and hypersensitivity to negative criticism. B. Narcissistic personality disorder is characterized by grandiosity, need for admiration, and lack of empathy. C. Obsessive-compulsive personality disorder is characterized by traits seen in this patient: a pattern of preoccupation with orderliness, perfectionism, and interpersonal control. D. Paranoid personality disorder is characterized by pervasive distrust and suspiciousness of others, including suspecting others of malevolent behavior without evidence, being unable to confi de in others, and bearing grudges.

193. A 64-year-old male presents with worsening pain in his right calf that occurs after walking short distances and resolves with rest. Examination reveals loss of hair and thinned and darkened skin over the lower extremities; distal pulses are diminished. Which of the following diagnostic studies is most appropriate to perform at this time? A. ankle-brachial index B. arteriography C. lymphangiography D. magnetic resonance angiography

193. The answer is A [Cardiovascular]. A. An ankle-brachial index is useful in gauging the degree of arterial insuffi ciency and should be performed prior to any additional imaging studies. B. This man exhibits signs and symptoms of arterial insuffi ciency. Arteriography is invasive and, therefore, not considered the initial diagnostic study of choice. Instead it is used to precisely localize the disease process and to assist in determining the best invasive surgical procedure for treatment, if surgery is planned. C. Lymphatic obstruction is typically associated with prominent, minimally pitting edema, which is not a fi nding in this case. D. Magnetic resonance angiography is less invasive than traditional angiography and provides excellent anatomic defi nition. However, it is indicated only if surgical revascularization is planned.

194. Abduction of the fl exed hip of a 1-month-old elicits a "clunk." What test is this and what does it assess? A. Ortolani sign; developmental dysplasia of the hip B. Lachman test; slipped capital femoral epiphysis C. Galeazzi test; Legg-Calvé-Perthes disease D. Pavlik's sign; Osgood-Schlatter disease

194. The answer is A [MSS/Rheumatology]. A. In the Ortolani maneuver, abduction of the fl exed hip results in the "clunk," which represents the reduction of the dislocated hip back into the acetabulum. Barlow maneuver reproduces slippage (clunk) on adduction. B. Lachman test evaluates anterior cruciate ligament stability. C. The Galeazzi test may detect unilateral hip dislocation in an older infant (3 to 6 months) by observing that one knee is lower than the other when the patient is supine with knees and hips fl exed and feet fl at on the examining table. D. The Pavlik harness is a device sometimes used to treat developmental dysplasia of the hip.

195. Five days following a blood transfusion of 2 units of packed red blood cells, a patient complains of fever, chills, nausea, and myalgias. What is the recommended management? A. mannitol B. an antihistamine C. watchful waiting D. prednisone

195. The answer is C [Hematology]. A. Forced diuresis with mannitol may help prevent renal damage in acute hemolytic reactions to major antibodies (ABO) occurring within minutes to hours of transfusion. B. Antihistamines are appropriate in the treatment of IgEmediated allergic reactions occurring within minutes to hours of transfusion. C. Therapy is rarely necessary for delayed hemolytic reactions to minor antibodies, which occur 3 to 21 days (most commonly 5 to 10 days) after the transfusion. D. Prednisone is indicated in more severe or acute reactions.

196. A 47-year-old male presents to the clinic complaining of severe pain in his lower jaw, fever, and facial swelling. He describes a constant, throbbing pain which prevents him from eating or sleeping because the pain intensifi es if he tries to lie down. The patient's breath exhibits a foul odor and there is an erythematous, fl uctuant, tender mass near the left lower third molar. What is the most likely diagnosis? A. dental abscess B. halitosis C. mumps infection D. sialadenitis

196. The answer is A [EENT, Infectious Disease]. A. Dental caries begin asymptomatically as a destructive process of the hard surface of the tooth. Over time, dental caries extend to the tooth pulp, which can lead to abscess formation. Abscesses are characterized by swelling, pain, and fever. B. Halitosis can be a sign of periodontal disease, oral abscess, xerostomia, esophageal stasis, sinusitis, lung abscess, systemic disease, or poor hygiene. It is not painful. Treatment is directed at the underlying cause. C. The prodrome of mumps includes fever, malaise, myalgia, and anorexia. Parotitis usually develops within the next 24 hours but may be delayed for as long as a week. It is generally bilateral. The submaxillary and sublingual glands are involved less often than the parotid. Swelling of the parotid obliterates the space between the ear lobe and angle of the mandible. The patient typically complains of earache and diffi culty eating or talking. Stensen duct will be red and swollen. D. Acute bacterial sialadenitis typically affects the parotid or submandibular gland. There is swelling and pain which worsens with eating. There is tenderness and erythema of the duct opening, and massage may release purulent material. Ductal obstruction leads to salivary stasis and secondary infection; the most common organism is Staphylococcus aureus.

197. An 8-year-old complains of nonspecifi c abdominal pain, cramps, fl atulence, bloating, and occasional diarrhea. The symptoms seem to be related to the ingestion of dairy products. What is the next step in management? A. 2-week trial of lactose-free diet B. hydrogen breath test C. use of calcium and magnesium supplementation D. use of pancreatic enzyme replacement

197. The answer is A [GI/Nutrition, Pediatrics]. A. This is a typical presentation of a patient with lactose intolerance. A practical approach in clinical practice is to eliminate dairy products for 2 weeks and look for symptom resolution. B. A hydrogen breath test is helpful to diagnose lactose intolerance, but a 2-week trial of lactose-free diet is more acceptable as it required ingestion of a lactose-loaded beverage followed by serial assessment of breath hydrogen. C. After the diagnosis of lactose intolerance is established, calcium supplementation is appropriate to replace the lost dietary calcium. D. Pancreatic enzymes do not break down lactose and thus are not appropriate for lactose intolerance. Lactase is from the brush border of the small intestine. Lactase enzyme replacement (Lactaid) is appropriate.

198. A patient is 12 hours post closed reduction of a tibial fracture and is in a long leg cast. Despite an injection of meperidine (Demerol), the patient complains of unrelieved pain and cannot move his toes. What is the best option for the management of this problem? A. Elevate the leg on an extra pillow. B. Immediately split the cast medially and laterally. C. Explain that this pain is typical of an early fracture and observe for several hours. D. Remove the cast completely.

198. The answer is B [MSS/Rheumatology, Surgery]. A. Elevation should be used while the necessary equipment is obtained to bivalve the cast. B. The treatment of suspected compartment syndrome is to split or bivalve the cast immediately, allowing circulation to the distal aspect of the limb. C. Observation of this serious condition alone could result in irreparable damage to the limb. D. Although this could be done, the original application was required to reduce the fracture. The removal of the cast could cause further injury to the fracture site.

199. A 38-year-old male continues to experience progressive dyspnea, wheezing, cough, and sputum production despite cessation of smoking 5 years ago. His father suffered from the same chronic respiratory problems until his death due to liver failure at age 52 years. The patient is on several bronchodilators including ipratropium, albuterol, and fl uticasone. Which of the following should be recommended to this patient at this time? A. lung transplantation B. oral prednisone C. _1-antitrypsin D. supplemental oxygen

199. The answer is C [Pulmonology]. A. Lung transplantation is an option for end-stage disease involving the lungs. It is considered last resort as organs are not always available and transplantation rejection is common. B. Oral prednisone, oxygen, and pulmonary rehabilitation are all therapies that may prolong life and decrease symptoms but are not curative. C. This patient likely is a1-antitrypsin defi cient. a1- Proteinase inhibitor (Prolastin) is a human protein that helps prevent breakdown of lung tissue, thereby reducing symptoms and prolonging life. D. See B.

200. A 36-year-old with chronic temporomandibular joint disorder presents to the emergency department with acute pain and muscle spasm. What is the treatment of choice? A. diazepam (Valium) B. cyclobenzaprine (Flexeril) C. ketorolac (Toradol) D. amitriptyline (Elavil)

200. The answer is A [MSS/Rheumatology]. A. Using 10 mg of diazepam IV will aid in muscle relaxation and decrease some of the patient's anxiety prior to manual reduction. B. Cyclobenzaprine (Flexeril) is only available in an oral preparation. An oral muscle relaxant will not control the spasm associated with dislocation. C. Ketorolac (Toradol) will relieve the pain but not the muscle spasm. D. Amitriptyline (Elavil) may be used to control chronic pain in TMJ, but it is ineffective for acute dislocation.

201. A 53-year-old male was diagnosed with a UTI and started on a fl uoroquinolone 3 days ago. Today, he presents with fever, chills, low back pain, and perineal pain for the past 6 hours. He has not been able to urinate for that time despite multiple attempts. Examination reveals a boggy and tender prostate. Urinalysis via percutaneous suprapubic catheterization reveals numerous leukocytes. Prostate-specifi c antigen (PSA) is acutely elevated. What is the most appropriate next step? A. computed tomography of pelvis B. continue antibiotics for 7 more days C. prostate massage and cultures D. transrectal ultrasonography

201. The answer is D [GU/Renal, Infectious Disease]. A. Transrectal ultrasonography is preferred over CT to rule out abscess or calculi. B. Treatment failure indicates a need for further workup. C. Massage is contraindicated in acute prostatitis due to the risk of septicemia. D. Patients with prostatitis will present with irritative voiding symptoms. Many will complain of low back pain, perineal pain, or suprapubic pain. Exam should proceed with caution as vigorous massage is contraindicated. An enlarged, boggy, or indurated prostate gland is characteristic. In cases of treatment failure, transrectal ultrasonography of the prostate gland may help to detect prostate calculi or abscess.

202. A 23-year-old male complains of frequency, urgency, and dysuria associated with fever and chills for 2 days. The patient states that he has diffi culty starting his stream. Physical exam reveals a febrile patient with minimal suprapubic tenderness and moderate perineal tenderness. The remainder of the examination is unremarkable. What is the most likely diagnosis? A. acute cystitis B. acute epididymitis C. acute prostatitis D. acute pyelonephritis

202. The answer is C [GU/Renal, Infectious Disease]. A. Acute cystitis is uncommon in males. It presents with irritative voiding symptoms but lacks systemic symptoms and perineal tenderness. B. Acute epididymitis may present with irritative voiding symptoms; however, pain starts in the scrotum and radiates along the spermatic cord or into the fl ank. C. Acute prostatitis presents with irritative voiding symptoms and fever accompanied by perineal, sacral, or suprapubic pain. D. Acute pyelonephritis presents with irritative voiding symptoms associated with fever, fl ank pain, and CVA tenderness.

203. A 65-year-old female has worked as a laundromat attendant for 25 years. She complains of pain in both knees which worsens with activity throughout the day. Radiography reveals osteophytes and joint narrowing. What is the recommended fi rst-line treatment? A. acetaminophen B. COX-2 inhibitor C. intra-articular steroid D. salicylates

203. The answer is A [MSS/Rheumatology, Geriatrics]. A. Acetaminophen is the initial treatment for mild to moderate osteoarthritis. If response is inadequate, NSAIDs should be used. NSAIDs are more effective but also carry higher risk of adverse effects. B. COX-2 inhibitors are effective but should be used judiciously due to the potential cardiac side effects. C. Intra-articular steroids are helpful in knee arthritis but are not considered fi rst-line treatment. D. Salicylates are effective in osteoarthriti

204. A 20-year-old female has a chronic history of recurrent episodes of right lower quadrant pain and diarrhea. Which fi nding on colonoscopy would suggest a diagnosis of Crohn disease rather than ulcerative colitis? A. aphthoid ulcerations and focal crypt abscesses in all layers of the bowel wall B. erythematous mucosa with a fi ne granular surface that looks like sandpaper C. lumpy, bumpy appearance of mucosa due to areas of infl amed but intact mucosa contiguous with ulcerated areas D. mucosal disease limited to the rectum

204. The answer is A [GI/Nutrition]. A. Active Crohn disease is characterized by focal infl ammation and formation of fi stula tracts. It can affect any part of the gastrointestinal tract from the mouth to the anus. B. Sandpaper appearance; lumpy, bumpy mucosa; and disease limited to the rectum are more consistent with ulcerative colitis. C. See B. D. See B.

205. A 22-year-old inner city social worker presents to the offi ce with a 2-week history of fatigue, anorexia, a 10-lb weight loss, a low-grade fever, and intermittent night sweats. He has a nonproductive cough that has recently become somewhat productive. What is the most likely diagnosis? A. infl uenza B. Pneumocystis jiroveci pneumonia C. costochondritis D. tuberculosis

205. The answer is D [Pulmonology, Infectious Disease]. A. Symptoms of infl uenza include myalgias, chills, headache, and nasal congestion. B. Pneumocystis jiroveci pneumonia more commonly presents with chronic dry cough and dyspnea with exertion. Although the incidence has decreased, it remains the most common opportunistic infection in patients with HIV. C. Symptoms of costochondritis include intermittent chest wall pain exacerbated by deep breathing and movement. D. These symptoms represent the classic presentation of a patient with tuberculosis. He is at higher risk due to his occupation.

206. A 10-year-old presents with his parents who are concerned about their son's school diffi culties. He has consistently been unable to stay in his seat and has great trouble focusing on his school work. He also has diffi culty following direction, is easily distracted, and is often forgetful about what he is assigned to do. What is the most appropriate pharmacologic choice for this patient? A. a benzodiazepine B. a methylphenidate preparation C. a sedative hypnotic D. a selective serotonin reuptake inhibitor

206. The answer is B [Psychiatry/Behavioral Medicine, Pediatrics]. A. A benzodiazepine may be used to treat anxiety and related disorders but is inappropriate in this case. B. A methylphenidate preparation, such as Ritalin, is the fi rst-line pharmacologic treatment of attention-defi cit hyperactivity disorder. C. A sedative hypnotic may be used in the treatment of sleep disorders and/or anxiety-related disorders. D. A selective serotonin reuptake inhibitor (SSRI) is used primarily in the treatment of depression and related disorders.

207. A 23-year-old presents with a 2-day history of conjunctival injection, discomfort, and a yellow-green discharge from the eyes. Physical exam reveals bilateral conjunctival injection, clear lungs, and no lymphadenopathy. Which of the following is the most appropriate management? A. Begin topical antihistamines for allergic conjunctivitis. B. Begin warm compresses and advise good hand washing hygiene for viral conjunctivitis. C. Begin topical sulfonamide for bacterial conjunctivitis. D. Begin topical steroids and refer to an ophthalmologist for uveitis.

207. The answer is C [EENT, Infectious Disease]. A. Allergic conjunctivitis is often seasonal and presents with itching; infection; a stringy, white discharge; and no adenopathy. Treatment with a topical antihistamine and avoidance of allergens may be helpful. B. Viral conjunctivitis presents with a clear watery discharge. Preauricular adenopathy is frequently present. Treatment is supportive and good hand washing is advised. C. Bacterial conjunctivitis presents with a purulent discharge; typically, there is no preauricular adenopathy. Treatment is with topical sulfonamide, gentamicin, tobramycin, norfl oxacin, or trimethoprim polymyxin B sulfate. Good hand washing is advised, and contaminated pillows, makeup, and towels should be avoided to prevent reinfection. D. Topical steroids exacerbate bacterial infections. Uveitis is not associated with purulent discharge.

208. A 19-year-old female college student is brought to the offi ce over Christmas break because her mother is very concerned about her despondent mood. She is not going out, not seeing her old friends, and stays in her room alone. You learn that last year, the patient went through a period of time where she was having trouble sleeping and was caught shoplifting. She tells you that she had collected almost 100 key chains from area stores during the months that she was feeling good. She also tells you that shortly before this episode, she had been feeling down for a few months and was relieved when she started to feel more energetic. About a month ago, she started to feel hopeless about everything. She denies any hallucinations and managed to pass all her courses even though she felt badly. She denies any suicidal ideation. Along with psychotherapy, which of the following is appropriate treatment for this patient? A. clonazepam (Klonopin) and carbamazepine (Tegretol) B. divalproex sodium (Depakote) alone C. haloperidol (Haldol) alone D. olanzapine and fl uoxetine combination (Symbyax)

208. The answer is D [Psychiatry/Behavioral Medicine]. A. The treatment of bipolar depression is controversial but should not include a benzodiazepine. B. Divalproex sodium (Depakote) has surpassed the use of lithium in the treatment of acute mania but is not generally used for bipolar depression. C. Haloperidol (Haldol) may be useful in acute mania or psychosis. D. Olanzapine is a mood stabilizer; fl uoxetine is an antidepressant. Together (available as Symbyax), they are very effective in treating bipolar depression.

209. A 34-year-old male presents complaining of a painful mouth sore for 2 days. He denies any alcohol or tobacco use and otherwise feels fi ne. Examination is signifi cant for a 2-mm round ulceration with a yellow-gray center surrounded by a red halo on the right buccal mucosa. What is the most likely diagnosis? A. aphthous ulcer B. glossitis C. herpetic stomatitis D. leukoplakia

209. The answer is A [EENT]. A. Aphthous ulcers are common and relatively straightforward to recognize. They appear as described and are found on nonkeratinized mucosa. They may be solitary or multiple and are typically recurring. They are usually 1 to 2 mm but can be larger. B. Glossitis is infl ammation of the tongue; it appears erythematous and smooth. It is usually painless and may be caused by a variety of etiologies (i.e., nutritional defi ciencies, drug reactions, dehydration, etc.). C. Herpetic stomatitis is usually mild and self-limited in most adults with normal immune function. Patients may complain of initial burning followed by multiple small vesicles that break and form scabs distributed over the lips, tongue, and gums. It is more common in children and is easily transmitted. D. Leukoplakia lesions are fl at and white. They are usually small but can be several centimeters in size. Often, they are secondary to chronic irritation (dentures, tobacco), but 2% to 6% may be either dysplastic or early squamous cell carcinoma. Alcohol and tobacco use are the major risk factors.

210. A 19-year-old male college student presents with an asymptomatic rash extending over his upper trunk, shoulders, and neck. The hypopigmented, annular lesions vary in size from 4 to 5 cm in diameter to larger, confl uent areas. There is no visible scale associated with the lesions. What organism is the most likely cause of his symptoms? A. Candida albicans B. Malassezia furfur C. Staphylococcus aureus D. Trichophyton rubrum

210. The answer is B [Dermatology, Infectious Disease]. A. Tinea versicolor is not caused by Candida. B. The patient has tinea versicolor, which is caused by an overgrowth of the yeast organism Malassezia, a common skin colonizer of all humans. C. Tinea versicolor is not caused by bacteria. D. Trichophyton rubrum is the most common cause of tinea corporis

211. A 17-year-old male with type 1 diabetes mellitus presents with acute pain in the left knee, fever, chills, malaise, and anorexia. He denies trauma. On examination, the knee is warm, swollen, and red. Range of motion is severely limited secondary to pain. Which of the following is the most appropriate initial therapy? A. IV vancomycin B. po dicloxacillin C. IV ceftriaxone D. po ciprofl oxacin

211. The answer is C [MSS/Rheumatology, Infectious Disease]. A. Vancomycin is the drug of choice if the causative agent is methicillin-resistant Staphylococcus aureus (MRSA). Risk factors for MRSA include hospitalization, chronic debilitating illness, and chronic antibiotic therapy. B. Oral dicloxacillin is an appropriate therapy for Staphylococcus aureus skin infections, but it is not suffi cient in the initial management of osteomyelitis. C. Parenteral second- or third-generation cephalosporins, such as ceftriaxone, provide good coverage for Staphylococcus aureus and Neisseria gonorrhea—the two most likely causative agents in this age group. It will also cover Gram-negative enteric rods which may be the cause of nontraumatic septic arthritis in the immunocompromised. Urgent drainage and debridement is also necessary. D. Oral ciprofl oxacin may be used to manage chronic osteomyelitis, but it is not indicated for acute treatment.

212. A 36-year-old male presents with fever, nonproductive cough, and burning chest pain that worsens with inspiration. He has had these symptoms and general malaise off and on for several weeks as well as night sweats and fever. He admits to having several male sexual partners in the past year; he is unaware of his HIV status. Examination reveals a temperature of 99.8°F, respiratory rate of 22, heart rate of 100 bpm, and a loss of 5 lb since his last visit. Chest x-ray shows diffuse interstitial infi ltrates. What is the recommended treatment? A. clindamycin-primaquine B. dapsone-trimethoprim C. trimethoprim-sulfamethoxazole D. intravenous pentamidine

212. The answer is C [Pulmonary, Infectious Disease]. A. Clindamycin-primaquine and dapsone-trimethoprim are alternative treatments for mild to moderate Pneumocystis pneumonia if intolerant to trimethoprimsulfamethoxazole (TMP-SMX). B. See A. C. TMP-SMX is the standard for both treatment and prophylaxis of Pneumocystis pneumonia. Most commonly, the CXR shows bilateral infi ltrates but can also be focal, nodular, cystic, or cavitary. D. Intravenous pentamidine is the treatment of choice for severe disease if the patient is unable to tolerate TMPSMX.

213. A 28-year-old was found unconscious on his back. It was unknown how long he had been down. A Foley catheter returns dark brown urine. Urinalysis reveals 4_ hemoglobin and 4_ protein. Microscopy reveals no RBCs; however, there are many granular casts. Serum creatinine is 3.5 mg/dL and K_ is 6.2 mEq/L. What is the most likely diagnosis? A. acute interstitial nephritis B. acute tubular necrosis C. postrenal azotemia D. prerenal azotemia

213. The answer is B [GU/Renal]. A. Acute interstitial nephritis may occur following exposure to a toxic agent, usually an antibiotic, resulting in WBC casts or eosinophilic granular casts in the urine. B. The patient has acute tubular necrosis (ATN) secondary to myoglobin release from being unconscious on his back for an unknown length of time (rhabdomyolysis). This is confi rmed by the acute tubular epithelial cell (granular) casts, lack of RBCs in the urine with positive hemoglobin on urine dip, and an increase in the serum creatinine. C. Postrenal azotemia results from ureteral/kidney obstruction. Urine output is reduced, yet urine is benign. D. Prerenal azotemia is most often due to hypovolemia

214. A 43-year-old female presents with a tremor, irritability, and nervousness. She states that despite the fact that she has an increased appetite and food intake, she has lost 8 lb over the last 4 weeks. Thyroid studies show TSH of 6.8 _U/ mL (normal 0.34 to 4.25 _U/mL), T4 of 12.9 _g/dL (normal 5.4 to 11.7 _g/dL), and T3 of 4.3 nmol/L (normal 1.2 to 2.1 nmol/L). What is the most likely diagnosis? A. Graves disease B. Hashimoto disease C. pituitary adenoma D. thyroid storm

214. The answer is C [Endocrinology, Surgery]. A. Graves disease typically causes decreased TSH and increased T3 and T4. B. Hashimoto disease typically causes increased TSH and decreased T3 and T4. C. These symptoms are likely caused by a TSH-producing pituitary adenoma. This will produce an elevated T4, T3, and TSH compared to typical causes of hyperthyroidism where TSH is low. D. Thyroid storm is a life-threatening complication of hyperthyroidism. This is often a clinical diagnosis as thyroid studies may be normal.

215. A 3-year-old girl is brought in by her mother who is concerned about a rash that started on her daughter's face 3 days ago and moved downward to her trunk and extremities including the palms and soles. The rash is starting to fade and fl ake from the head and face. Prior to the rash, the child had fever, runny nose, sneezing, and a dry cough. The mother states that the child has not been immunized due to religious beliefs. Examination reveals an erythematous maculopapular rash on the face, trunk, and extremities. There are discrete lesions on the trunk and extremities, whereas lesions of the face and neck appear confl uent and with brownish discoloration and fi ne scaling. What is the most likely diagnosis? A. erythema infectiosum B. measles C. rubella D. scarlet fever

215. The answer is B [Dermatology, Infectious Disease, Pediatrics]. A. Erythema infectiosum (fi fth disease) is caused by the erythrovirus (formerly human parvovirus B19). It is characterized by erythematous, edematous plaques on the cheeks giving the face a "slapped cheek" appearance and a generalized exanthem. Lesions are discrete macules that become confl uent to form a lacy or reticular appearance. The exanthem can be preceded by a prodrome of fever, malaise, headache, and coryza. B. The measles virus is highly contagious. Disease is characterized by fever, malaise, coryza, and cough. Koplik spots (bluish-white spots on the buccal mucosa) appear before the exanthem and are pathognomonic. The maculopapular lesions are erythematous and appear from a head-to-toe distribution; desquamation also occurs in a head-to-toe fashion. C. Rubella infection frequently lacks a prodrome. The exanthem is characterized by erythematous macules and papules that appear on the face fi rst, spread inferiorly within 24 hours, and fade without pigmentary changes or scaling by day 3. It is usually associated with enlargement of the postauricular and posterior cervical lymph nodes. D. Scarlet fever is caused by an exotoxin-producing strain of group A b-hemolytic Streptococcus. It is characterized by a fi ne erythematous rash (sandpaper) that presents on the trunk and face, sparing the area around the mouth. The rash is typically accentuated in the skin folds; palms and soles are typically spared.

216. A person with early Alzheimer disease will have diffi culty with recent memory (word recall) on the Mini-Mental Status Examination. What other task would most likely be diminished as well? A. drawing a clock B. object naming C. reading comprehension D. writing a sentence

216. The answer is A [Neurology, Geriatrics]. A. Persons in the early stages of Alzheimer disease will typically have diffi culties with memory and visuospatial abilities. The "mini-cog" test consists of a clock-drawing test and a three-item word recall. B. Object naming, reading comprehension, and sentence writing are not generally affected in early Alzheimer disease. C. See B. D. See B.

217. A 28-year-old female at 33 weeks' gestation complains of leaking fl uid from her vagina since she woke up this morning. Examination with a sterile speculum reveals a collection of fl uid in the posterior fornix. Which of the following tests would best confi rm the presence of amniotic fl uid? A. Check for ferning under a microscope. B. Look for clue cells on microscopic examination. C. Perform a guaiac test on the fl uid. D. Test the fl uid with nitrazine paper to see if it turns red.

217. The answer is A [Ob/Gyn]. A. Ferning is seen when amniotic fluid is air-dried on a microscope slide. B. Clue cells indicate the presence of bacterial vaginosis. C. The guaiac test detects blood. D. Nitrazine paper turns blue in the presence of amniotic fluid.

218. A 27-year-old injection drug user presents with fever, chills, and a new murmur. Examination reveals track marks on the forearms and petechiae on the palate and beneath the fi ngernails. Which of the following agents is most likely responsible? A. HACEK organisms B. non-albicans Candida C. Serratia marcescens D. Staphylococcus aureus

218. The answer is D [Cardiovascular, Infectious Disease]. A. HACEK organisms (Haemophilus, Actinobacillus, Cardiobacterium, Eikenella, and Kingella) are more likely to cause endocarditis in native valves. B. Yeasts and fungi tend to cause a subacute endocarditis in native valves or during the early postoperative phase of prosthetic valve placement. Incidence is low. C. Serratia is a rare cause of endocarditis. D. In injection drug users, Staphylococcus aureus accounts for over 60% of endocarditis cases.

219. A 56-year-old male complains of a nonhealing lesion on his left cheek. Examination reveals a 6-mm pearly papule with surface telangiectasias and a central erosion. What is the most appropriate next step in management? A. curettage B. electrodesiccation C. excision D. shave biopsy

219. The answer is D [Dermatology, Oncology]. A. Curettage is not indicated in the initial evaluation of suspected basal cell carcinoma. B. Electrodessication is not indicated in the initial evaluation of suspected basal cell carcinoma. C. All lesions suspicious for basal cell carcinoma should undergo biopsy before excision is attempted. D. All lesions suspicious for basal cell carcinoma should undergo either shave or punch biopsy prior to initiating curative treatment.

220. A lesion involving which of the following will result in decreased sensation along the palmar aspect of the right thumb, second and third fi ngers, and half of the fourth fi nger? A. radial nerve B. ulnar nerve C. median nerve D. brachial plexus

220. The answer is C [Neurology]. A. The radial nerve supplies the dorsal hand. B. The ulnar nerve supplies the fourth and fi fth digits. C. The median nerve supplies the palmar aspect of the right thumb, second and third digits, and half of the fourth digit. D. Brachial plexus injuries cause pain and weakness of the shoulder and sensory disturbances of the lateral arm (C5 and C6)

221. A 79-year-old male with a history of mild COPD and hypertension controlled with hydrochlorothiazide presents with dyspnea. He states it began 2 days ago on exertion and has progressed over the last 12 hours to dyspnea at rest. Physical exam reveals an S3 and 2_ pitting edema to the ankles. Which of the following is also likely to be present? A. an ejection click B. the presence of a thrill C. jugular vein distention D. a fourth heart sound (S4)

221. The answer is C [Cardiovascular]. A. An ejection click is indicative of mitral valve prolapse or aortic stenosis. B. Thrills often accompany loud, harsh, or rumbling murmurs such as those of aortic stenosis, patent ductus arteriosus, ventricular septal defect, and mitral stenosis. C. Distention of the jugular veins indicates congestion due to right ventricular failure. The most common cause of right ventricular failure is left ventricular failure. D. An S4 occurs when there is increased resistance to ventricular fi lling during atrial contraction. The causes of an S4 include hypertensive heart disease, coronary artery disease, aortic stenosis, and cardiomyopathy.

222. A 15-year-old presents with fever, malaise, and sore throat with diffi culty swallowing. Physical exam reveals enlarged anterior and posterior cervical nodes, palatal petechiae, and severely enlarged "kissing" tonsils without exudate. What is the recommended treatment? A. amoxicillin B. acyclovir C. interferon-alpha D. corticosteroids

222. The answer is D [EENT, Infectious Disease, Pediatrics]. A. Penicillins are effective against bacterial infection. They are not specifi cally therapeutic in reducing tonsillar enlargement that is secondary to mononucleosis. Many patients with mononucleosis who are treated with ampicillin or amoxicillin will develop a diffuse rash. B. Antiviral compounds will reduce viral shedding but will not affect symptoms or duration of mononucleosis. C. Interferon-alpha is not helpful in treating mononucleosis. D. Corticosteroids are advised specifi cally for reducing lymphoid enlargement threatening to compromise the airway in patients with mononucleosis. A maximum of 10 days with tapering dose is advised.

223. A 60-year-old male with a history of moderately severe chronic obstructive pulmonary disease has recently weaned off a long course of corticosteroids. He presents now with headaches that are worse with straining. He also complains of double vision and blurriness. What is most likely to be found on physical exam? A. carotid bruit B. papilledema C. positive Romberg sign D. resting tremor

223. The answer is B [Neurology]. A. A carotid bruit is associated with transient ischemic attacks such as amaurosis fugax. B. Pseudotumor cerebri (benign intracranial hypertension) may develop after stopping corticosteroids, as a complication of otitis or mastoiditis, with endocrine abnormalities, or with certain medications. Papilledema and blind spots are seen on physical exam. C. A positive Romberg sign indicates disorders of the dorsal column or sensory pathways. D. A resting tremor may develop with medication withdrawals or with chronic neurologic disorders such as Parkinson.

224. A 35-year-old female comes to the offi ce complaining of acutely infl amed and painful left eye. Her symptoms began 2 days ago. There is some visual blurring associated with her symptoms. The patient wears contact lenses. Examination reveals diffuse infl ammation of the left conjunctiva; fl uorescein staining reveals a shallow, dendritic ulcer in the center of the cornea. What is the recommended management? A. trifl uridine (Viroptic) B. moxifl oxacin (Vigamox) C. olopatadine (Patanol) D. prednisolone (Pred Forte)

224. The answer is A [EENT, Infectious Disease]. A. Herpes keratitis in an immunocompetent host typically resolves in 2 to 3 weeks. Antiviral treatment is used to prevent stromal damage and scarring. Referral to ophthalmology is warranted. B. Moxifl oxacin (Vigamox) is a topical antibiotic used to treat susceptible bacterial ocular infections. C. Olopatadine (Patanol) is a topical antihistamine/mast cell stabilizer used to treat symptoms of allergic conjunctivitis. It is not contraindicated in this patient with herpes keratitis, although it will not improve her condition. D. Prednisolone (Pred Forte) is a topical steroid that is indicated if the herpes infection results in stromal damage or excessive scarring. All topical steroids should be used with extreme caution and with ophthalmologist supervision due to the risk of corneal melting, perforation, prolonged infection, and induction of glaucoma.

225. What is the hallmark electrocardiographic fi nding in Mobitz type I (Wenckebach) atrioventricular block? A. accordion-appearing QRS morphology B. progressive lengthening of the PR interval C. ventricular rate of less than 50 bpm D. widened QRS complexes

225. The answer is B [Cardiovascular]. A. Accordion-appearing QRS morphology is seen in torsades de pointes. B. Mobitz type I (Wenckebach) second-degree AV block is characterized by progressive lengthening of the PR interval and shortening of the RR interval before a blocked beat. C. Ventricular rates of less than 50 bpm and wide QRS complexes are often seen in a third-degree (complete) heart block. D. Widened QRS complexes are seen in ventricular arrhythmias and in third-degree (complete) heart block.

226. A 32-year-old female presents to the offi ce complaining of left lower extremity swelling and pain. She denies any history of trauma. Examination reveals a swollen left lower extremity that is 7 cm greater in size than the right at the level of the calf. What is the best next step in the diagnosis of this patient? A. bilateral lower extremity venography B. D-dimer blood assay C. lower extremity venous Doppler studies D. spiral CT of the chest

226. The answer is C [Cardiovascular]. A. Venography will provide a defi nitive diagnosis, but it is an invasive procedure that is rarely necessary in the diagnosis of deep venous thrombosis (DVT). B. D-dimer will likely be elevated; however, it has a high sensitivity but low specifi city. C. The diagnosis of deep vein thrombosis requires clinical suspicion as well as carefully chosen appropriate diagnostic studies. Currently, Doppler ultrasonography is the fi rst step in making the diagnosis. It has a high sensitivity and a high specifi city. D. Spiral CT scan would be recommended if a pulmonary embolus was suspected, if Doppler studies are positive, or if there is a prior history of DVT.

227. A 61-year-old Asian American female presents to the emergency department after falling on her front porch. She has pain in her left hip and is unable to bear weight. She has been previously healthy without signifi cant medical history. Thyroid studies, CBC with differential, urine studies, and blood chemistries are normal. Radiographs of the left hip show a femoral neck fracture. What is the most likely underlying contributor to this patient's fracture? A. hypoparathyroidism B. osteosarcoma C. osteoporosis D. Paget disease

227. The answer is C [MSS/Rheumatology, Surgery]. A. Hypoparathyroidism causes hypocalcemia as a result of decreased intestinal calcium absorption. The diagnosis is made by fi nding a low serum calcium level in a patient with reduced parathyroid concentration. B. Osteosarcoma is a primary malignant tumor presenting with pain in the area of the lesion with a deep, fi rm, mobile mass on palpation. This most commonly presents in children or adolescents or as a complication of Paget disease in adults. C. Osteoporosis is often diagnosed following an acute fracture or with bone mineral densitometry assessment. The World Health Organization (WHO) defi nes osteoporosis as a bone mineral density below 2.5 SD (standard deviations). D. Paget disease is the second most common bone disease. Paget disease is focal and typically asymptomatic, being detected on radiography or through increased phosphatase levels. If patients are symptomatic, they typically present with bone pain.

228. A 64-year-old nursing home resident complains of pain and swelling of the penis. Examination reveals a swollen retracted foreskin with marked edema and mucopurulent drainage. What is the most likely diagnosis? A. paraphimosis B. penile carcinoma C. Peyronie disease D. urethritis

228. The answer is A [GU/Renal, Surgery]. A. Phimosis is an inability to retract the foreskin over the glans. In paraphimosis, the foreskin has been retracted over the glans but has failed to reduce to its normal position. This causes venous congestion, swelling, and eventually the arterial supply can be compromised. Surgical intervention is required if it fails to reduce manually. B. Penile cancer is rare. The most common type is a squamous cell lesion of the skin. Metastasis can be fatal. C. Peyronie disease is a progressive curvature of the penis that causes painful erections and fi brosis. D. Urethritis manifests with irritative voiding symptoms (urgency, frequency, dysuria).

229. A 13-year-old female is brought in for evaluation by her father who claims to hear the child vomiting after eating large volumes of food. The patient repeatedly denies vomiting and "feels fi ne." What physical sign or symptom is most likely present in this patient? A. emaciated physical appearance B. eruption of wisdom teeth C. lymphadenopathy D. petechial hemorrhages of the soft palate

229. The answer is D [Psychiatry/Behavioral Medicine, Pediatrics]. A. The patient has bulimia nervosa which usually presents with normal or above average body weight. Anorexia nervosa, not bulimia nervosa, is associated with below normal body weight. B. Eroded dental enamel from contact with stomach acids is common in bulimia. Eruption of wisdom teeth is not effected. C. Parotid and salivary gland swelling may occur; it should not be confused with enlarged lymph nodes. D. Petechial hemorrhages of the cornea, conjunctivae, soft palate, or face may be noted after self-induced vomiting.

230. A 57-year-old male with diabetes mellitus presents for a physical exam. A slight decline in this patient's visual acuity and the presence of microaneurysms and small hemorrhages are noted. Which of the following is the most appropriate management option at this time? A. Perform visual fi eld testing. B. Prescribe artifi cial teardrops. C. Refer to an ophthalmologist. D. Prescribe glaucoma eye drops.

230. The answer is C [EENT]. A. This patient has diabetic retinopathy. Visual fi elds are not affected, therefore testing is not helpful. B. This patient has diabetic retinopathy and does not have, or complain of, dry eyes. Artifi cial teardrops are not an appropriate treatment for this disorder. C. Nonproliferative retinopathy is the most common cause of legal blindness in type 2 diabetes mellitus. Visual loss develops due to edema, ischemia, or exudates at the macula. D. This patient does not have glaucoma; glaucoma eye drops are not effective in retinopathy.

231. A 26-year-old female with a history of systemic lupus erythematosus (SLE) for 5 years is found to have elevated blood pressure on three separate occasions. Labs reveal anemia and mildly elevated BUN and creatinine. Urinalysis shows proteinuria but no casts. Which of the following pharmacologic agents would be best to treat the hypertension? A. ACE inhibitor B. _-blocker C. calcium channel blocker D. diuretic

231. The answer is A [Cardiovascular, GU/Renal]. A. ACE inhibitors and angiotensin receptor blockers (ARBs) have clinically compelling indications in the treatment of hypertension in patients with any type of kidney disease. They promote dilation of the efferent arterioles and, thus, reduction of intraglomerular pressure. B. See A. C. See A. D. See A.

232. On postoperative day 2 after an exploratory laparotomy under general anesthesia, a patient has a low-grade fever, tachypnea, and scattered rales. The patient is not coughing and denies shortness of breath. What is the most likely cause? A. atelectasis B. pneumonia C. pulmonary embolus D. congestive heart failure

232. The answer is A [Pulmonology, Surgery]. A. Atelectasis occurs in about 25% of patients after abdominal surgery. The effects of mechanical ventilation and postoperative analgesia cause a depression of the respiratory refl exes such as coughing, yawning, and periodic deep breathing that would otherwise expand collapsed alveoli (atelectasis). It typically develops within the fi rst 48 hours after surgery and accounts for the vast majority of postoperative fevers. B. Although pneumonia may occur secondary to atelectasis or contamination of the bronchial tree, it is less common than atelectasis and would more likely be accompanied by fever, cough, and localized rales and rhonchi. C. Pulmonary embolus, although a worrisome complication of major surgery, is less common than atelectasis. Shortness of breath and chest pain are common manifestations. D. This is not a typical presentation of congestive heart failure (CHF). CHF occurs most commonly in patients with underlying heart disease. It presents with shortness of breath, fatigue, and edema. It is not a common complication of abdominal surgery.

233. A 27-year-old presents with acute fever and headache. Examination reveals drowsiness, confusion, and nuchal rigidity. CSF analysis will most likely reveal which of the following? A. decreased glucose B. decreased protein C. elevated lymphocytes D. low opening pressure

233. The answer is A [Neurology, Infectious Disease]. A. CSF in bacterial meningitis would have decreased glucose secondary to consumption by the infecting organisms. B. CSF in patients with bacterial meningitis will have elevated protein. C. Elevated lymphocytes are seen in viral (aseptic) meningitis. The course of viral meningitis is usually subacute. D. There is a markedly increased opening pressure in persons with bacterial meningitis.

234. The Allen test should be performed prior to arterial puncture of which of the following arteries? A. radial B. carotid C. brachial D. femoral

234. The answer is A [Cardiovascular]. A. The Allen test is used to evaluate arterial supply in the upper extremity by assessing the radial and ulnar artery loop. B. See A. C. See A. D. See A.

235. A patient with no known risk factors for colorectal cancer should begin screening at what age? A. 40 years B. 45 years C. 50 years D. 55 years

235. The answer is C [GI/Nutrition, Oncology]. A. See C. B. See C. C. Colorectal screening for those without any known risk factors is recommended to begin at age 50 years. For patients with a family history, screening should begin at age 40 years or 10 years earlier than the age at which the family member was diagnosed. D. See C.

236. A 62-year-old male with history of hypertension and hyperlipidemia presents with left-sided hemiplegia and an associated hemisensory loss along with homonymous hemianopia. What area of the brain has most likely been affected? A. right middle cerebral artery B. right posterior cerebral artery C. left middle cerebral artery D. left posterior cerebral artery

236. The answer is A [Neurology, Cardiovascular]. A. Occlusion of the middle cerebral artery will result in contralateral hemiplegia with hemisensory loss as well as homonymous hemianopia. B. Occlusion of the posterior cerebral artery can result in a contralateral hemisensory defi cit but not hemiplegia. Spontaneous pain and hyperpathia may follow. C. Occlusion of the middle cerebral artery results in contralateral defi cits; therefore, this patient's occlusion cannot be in the left middle cerebral artery. D. Occlusion of the left posterior cerebral artery would result in right-sided deficits.

237. A 24-year-old G2P1001 female at 35 weeks' gestation complains of abdominal pain associated with moderate vaginal bleeding but no leakage of fl uid and no history of trauma. She admits to using cocaine recently. Physical exam reveals blood pressure of 150/90 mm Hg and heart rate of 110 bpm. She has no peripheral edema. The fundus is tender and a moderate amount of dark blood is noted in the vagina. The cervix is 1.0 cm dilated. Fetal heart tones are in the 160 to 170 range. What is the most likely diagnosis? A. abruptio placentae B. placenta accreta C. placenta previa D. preeclampsia

237. The answer is A [Ob/Gyn, Surgery]. A. Abruptio placentae presents as painful vaginal bleeding during the third trimester. Cocaine use is associated with increased risk for abruptio placentae. B. Placenta accreta is a placenta that directly adheres to the myometrium without an intervening decidual layer. C. Placenta previa presents as painless third trimester vaginal bleeding. D. Preeclampsia manifests with the classic diagnostic triad of hypertension, proteinuria, and edema.

238. A G3P2002 presents at 33 weeks' gestation. She has had two 30-second contractions within 10 minutes; cervix is 3 cm dilated. Nitrazine test is negative. She has no vaginal bleeding. What is the most likely diagnosis? A. Braxton-Hicks contractions B. abruptio placentae C. premature rupture of membranes D. preterm labor

238. The answer is D [Ob/Gyn]. A. Braxton-Hicks contractions occur during the last 4 to 8 weeks of pregnancy and are generally painless uterine contractions with gradually increasing frequency. They are not associated with cervical dilation. B. Abruptio placentae presents with painful vaginal bleeding. C. Premature rupture of membranes would result in a positive nitrazine test. D. Preterm labor is distinguished from Braxton-Hicks contractions by changes in cervical dilation in response to the contractions.

239. A 2-year-old boy presents to the emergency department with elbow pain. While playing in the park, his father pulled him up by his arm to catch him from falling off the jungle gym. The child immediately cried out in pain and now his arm is hanging by his side with limited mobility. On physical exam, the child has limited supination and swelling is absent. What is the most likely diagnosis? A. elbow tendonitis B. ganglion cyst C. nursemaid's elbow D. Salter-Harris I fracture

239. The answer is C [MSS/Rheumatology, Pediatrics]. A. Tendonosis (formerly called tendonitis) results from overuse injuries and presents with pain over the epicondyles. B. Ganglion cysts are the most common masses of the hand and wrist. They may present as asymptomatic masses or with aching and a feeling of weakness. They are uncommon in children. C. Nursemaid's elbow is a result of subluxation of the annular ligament. The injury occurs with longitudinal pulling of the elbow, often when a child is lifted by one arm. This is easily reduced by supination of the elbow with pressure on the radial head. D. Salter-Harris type I fractures are nondisplaced fractures involving the growth plate. This type of elbow fracture typically involves blunt trauma.

240. A 46-year-old construction worker complains of pain when moving his right thumb or wrist. Examination reveals thickening and tenderness over the radial stylus. What physical exam technique will best confi rm the suspected diagnosis? A. Lachman test B. Phalen test C. Allen test D. Finkelstein test

240. The answer is D [MSS/Rheumatology]. A. Lachman test evaluates anterior cruciate ligament stability. B. Phalen test evaluates the wrist for carpal tunnel syndrome. C. Allen test evaluates the collateral circulation of the hand. D. In de Quervain tenosynovitis, the abductor pollicis longus and the extensor pollicis brevis tendons become infl amed. Finkelstein maneuver is accomplished by placing the thumb in the palm and enclosing the thumb with the fi ngers. The wrist is then placed in ulnar deviation, stretching the tendons; pain is a positive fi nding.

241. A 63-year-old with a history of chronic atrial fi brillation complains of acute pain and numbness of the left arm. On examination, the limb is cool and pale, and distal pulses are absent. What is the most likely diagnosis? A. acute arterial occlusion B. Buerger disease C. Takayasu arteritis D. venous spasm

241. The answer is A [Cardiovascular, Surgery]. A. Acute arterial occlusion results in paleness, pain, and pulselessness. Almost 90% of arterial emboli arise from the heart. B. Buerger disease (thromboangiitis obliterans) is an infl ammatory thrombotic process of distal arteries that is a result of arteritis. It is found most commonly in young men who smoke. C. Takayasu arteritis is an occlusive polyarteritis with predilection for the aortic arch. D. A venous spasm causes a quick stab of pain that subsides quickly and does not cause the other signs described.

242. A 16-year-old has had asthma since age 7 years. He uses combination fl uticasone and salmeterol (Advair Diskus 100/50) twice per day. He has breakthrough wheezing requiring albuterol less than once per week. He claims his asthma does not interfere with daily activities. Lungs are clear on examination. What is the most appropriate recommendation? A. Perform spirometry every 6 months. B. Maintain a self-administered peak fl ow record. C. Instruct patient to call at onset of symptoms. D. Have patient schedule follow-up visits when needed.

242. The answer is B [Pulmonology, Pediatrics]. A. Spirometry is useful at the time of initial assessment and when the condition becomes refractory to treatment. B. This patient has chronic stable persistent asthma. A peak fl ow record and a medication response are most important in managing asthma. Peak fl ow mea surements are simple and can be self- administered. Any change from baseline should prompt adjustment in management and communication with the clinician. C. It is best that the patient be educated and given a written self-directed action plan based on signs, symptoms, and peak fl ow rates. Signifi cant changes in peak fl ow occur prior to development of symptoms. D. It is best to have regularly scheduled follow-up appointments to discuss symptoms at a minimum of 6-month intervals or more frequently based on status.

243. A 31-year-old female complains of weakness and fatigue progressing over the last 3 months. Laboratory studies reveal a hemoglobin of 10.6 g/dL and a hematocrit of 37%. Which of the following lab results is also most likely present in this patient? A. decreased TIBC B. increased platelet count C. decreased serum ferritin D. increased reticulocyte count

243. The answer is C [Hematology]. A. The TIBC is increased in iron deficiency anemia. B. See C. C. With iron deficiency anemia, depletion of iron stores occurs first, then iron reserves are lost, as indicated by reduced serum ferritin. Erythropoiesis is stimulated, TIBC rises, and serum iron is low. An increased platelet count may accompany iron deficiency anemia but is not specific for diagnosis. D. Sufficient iron is necessary for new RBC formation in bone marrow. Without enough iron, a reticulocytosis cannot occur.

244. A patient presents to the emergency department with constant epigastric pain for 6 hours with nausea and vomiting for 3 hours. The pain is not relieved with the vomiting. Abdomen is tender and bowel sounds are minimal; there are ecchymoses on the fl anks. Which of the following laboratory fi ndings would indicate the poorest prognosis? A. LDH greater than 350 U/L; white blood cell count greater than 12,000/_L; AST over 100 units/L B. glucose greater than 200 mg/dL; white blood cell count greater than 16,000/_L; LDH 400 mg/dL C. white blood cell count greater than 16,000/_L; serum LDH greater than 200 U/L; AST over 100 U/L D. white blood cell count greater than 16,000/_L; AST over 100 U/L; serum calcium less than 10 mEq/L

244. The answer is B [GI/Nutrition]. A. See B. B. Ranson criteria is generally used to assess the severity of acute pancreatitis. When three or more of the following findings are present, a severe course of complicated pancreatitis can be predicted, placing the patient at risk for adverse outcome: age older than 55 years, white blood cell count over 16,000/L, blood glucose level over 200 mg/dL, serum LDH over 350 U/L, and AST greater than 250 U/L. Additionally, development of the following in the fi rst 48 hours indicates a worsening prognosis: hematocrit drop of greater than 10%, BUN rise greater than 5 mg/dL, arterial PO2 of less than 60 mm Hg, serum calcium less than 8 mg/dL, base deficit over 4 mEq/L, and estimated fluid sequestration of greater than 6 L. C. See B. D. See B.

250. A 25-year-old student presents with acute onset (_24 hours) of high fever, chills, myalgias, severe malaise, and nonproductive cough. Rapid antigen testing supports the diagnosis of infl uenza type A. What is the recommended treatment? A. amantadine (Symmetrel) B. rimantadine (Flumadine) C. valacyclovir (Valtrex) D. oseltamivir (Tamifl u)

250. The answer is D [Infectious Disease, Pulmonary]. A. Amantadine and rimantadine are no longer recommended for the treatment of acute infl uenza due to the growth of resistance. B. See A. C. Valacyclovir is appropriate for treatment of herpes infections, but it is not recommended against infl uenza virus. D. Neuraminidase inhibitors (oseltamivir, zanamivir) have become the recommended treatment choices for acute infl uenza although reports of resistance are becoming more common.

245. A 40-year-old female presents for an annual visit. When you ask how she has been, she starts to tell you but then breaks down in tears. She tells you that her sister died a few months ago after a long battle with breast cancer, and since that time, she has been sad and cries whenever she thinks of her sister. She has continued in her job at a local school and goes out with her family and friends. She can enjoy herself as long as she does not think about how much she misses her sister. What is the most likely diagnosis? A. complicated bereavement B. major depression C. posttraumatic stress disorder D. uncomplicated bereavement

245. The answer is D [Psychiatry/Behavioral Medicine]. A. Complicated bereavement may take a number of different forms including chronic grief, which is highlighted by bitterness and idealization of the dead and lasts over an extended period of time. B. Major depression includes sadness, but the onset does not usually correspond to a death. It also usually involves more serious functional impairment than described in the case previously mentioned. C. Posttraumatic stress disorder occurs after experiencing an unnatural or violent death. Grief experienced by these patients often includes fear, horror, and vulnerability. Patients often complain of anxiety. D. This is an example of uncomplicated bereavement. The patient is exhibiting sadness and grief over the loss of her sister but is not functionally impaired.

246. A 43-year-old male complains of several episodes of chest pain. Each episode occurs with exertion and is relieved with rest. He denies cigarette smoking and alcohol abuse. His lipid profi le is normal. What is the most appropriate diagnostic study for evaluating this patient? A. exercise stress test B. myocardial perfusion scintigraphy C. electrocardiography D. echocardiography

246. The answer is A [Cardiovascular]. A. An exercise stress test is noninvasive, inexpensive, and has a 70% sensitivity and 75% specificity in diagnosing angina due to coronary artery disease. B. Scintigraphy is invasive and indicated only if exercise stress testing is inconclusive or to localize areas of ischemia. C. Electrocardiography is normal in 25% of patients with angina. D. Echocardiography combined with dobutamine may be an alternative to exercise stress testing but requires more experience to interpret the results.

247. A 21-year-old male comes to the university health services because of a persistent lump in his neck. It has been present for about 4 months. The lump usually does not bother him, but he has experienced pain episodes intermittently, the most recent was the previous weekend after attending an end-of-term party. What is the most likely diagnosis? A. acute infectious mononucleosis B. acute lymphoblastic leukemia C. Hodgkin lymphoma D. lymphadenitis

247. The answer is C [Hematology]. A. Although this male is in a risk group for mononucleosis, the absence of fever, sore throat, and other findings makes this diagnosis less likely. B. Acute lymphoblastic leukemia may be characterized by lymphatic enlargement, but this man lacks other symptoms suggesting the diagnosis. C. The presence of an enlarged lymph node that is painful after alcohol ingestion suggests Hodgkin lymphoma. D. Lymphadenitis is associated with fever and upper respiratory infection.

248. A positive Myerson sign is most commonly seen in persons with which of the following disorders? A. Bell palsy B. Guillain-Barré syndrome C. multiple sclerosis D. Parkinson disease

248. The answer is D [Neurology]. A. Myerson sign is not seen in persons with Bell palsy, Guillain-Barré syndrome, or multiple sclerosis. B. See D. C. See D. D. Myerson sign is reflexive, sustained eye blinking in response to repetitive tapping just above the nasal bridge between the eyes (also referred to as the glabellar reflex). This sign can frequently be observed in persons with Parkinson disease.

249. A 23-year-old female presents with a history of recurrent lower abdominal pain associated with alternating diarrhea and constipation for the past 4 months. Pain is temporarily relieved with defecation. She has had increased levels of stress at work, which seems to correlate with the appearance of her symptoms. She describes her symptoms as mild but is concerned about their recurring nature. She denies fever or weight loss; she has maintained normal menses. The entire abdomen is mildly tender on examination. What is the initial step in management of this disorder? A. anticholinergic medications B. antidiarrheal medications C. reassurance and dietary/lifestyle changes D. serotonin receptor antagonists

249. The answer is C [GI/Nutrition, Psychiatry]. A. Clinicians have observed that anticholinergic drugs may provide temporary relief for symptoms such as painful cramps related to intestinal spasm, although controlled clinical trials have produced mixed results. B. When diarrhea is severe, especially in the painless diarrhea variant of irritable bowel syndrome (IBS), small doses of loperamide can be prescribed. C. Initial treatment of irritable bowel syndrome (IBS) involves education, reassurance, and dietary/lifestyle changes unless alarm symptoms are present (weight loss, acute onset, fever, nocturnal diarrhea). Patients should be instructed to increase water intake, avoid caffeine, and monitor their diet for triggering foods. Common culprits include legumes, fructose, and lactose. Stress reduction techniques and relaxation may be helpful. Medications may be added if symptoms are more than just mild in nature and cannot be adequately controlled with diet and lifestyle adjustments. D. Serotonin receptor antagonist may be a treatment option in moderate to severe disease.

251. A 9-month-old is brought to the clinic after 3 days of cough, low-grade fever, congestion, and runny nose. She attends day care, and a number of the children are experiencing similar symptoms. Examination fi nds an active child who appears well with a temperature of 100.4°F and a respiratory rate of 20. Clear discharge is present at the nares; lungs are clear bilaterally. What is the best management? A. Initiate a broad-spectrum antibiotic. B. Prescribe an over-the-counter antihistamine and decongestant. C. Recommend nasal saline drops and a cool-mist humidifi er. D. Suggest 1 teaspoon of honey every 4 to 6 hours to control the cough.

251. The answer is C [Pulmonology, Pediatrics, Infectious Disease]. A. Because the child appears well and is in no acute distress, an antibiotic is not warranted. The child should be monitored and return for reevaluation if symptoms progress. B. The American Academy of Pediatrics cautions against the use of over-the-counter cough and cold preparation in children younger than 6 years of age. C. The symptoms suggest a viral URI which a child in day care may experience 8 to 12 times per year. Cool-mist humidifi ers and nasal saline drops provide symptomatic relief. D. Honey should never be given to children younger than 1 year of age due to the risk of botulism.

252. A 9-year-old being treated for acute bacterial ethmoid sinusitis returns to the clinic 2 days later with complaints of pain and swelling in the left eye. Visual acuity is normal. Mental status is intact. Examination reveals periorbital edema and pain on movement of the extraocular muscles. The patient is febrile. What is the most likely diagnosis? A. conjunctivitis B. endophthalmitis C. orbital cellulitis D. uveitis

252. The answer is C [EENT, Infectious Disease, Pediatrics]. A. Conjunctivitis is the most common cause of a red, irritated eye. Infection is limited to the bulbar and palpebral conjunctiva with minimal pain, discharge, foreign body sensation, and sometimes photophobia. Young children are at increased risk. B. Endophthalmitis is the term for any bacterial, viral, fungal, or parasitic infection of the internal structures of the eye. Endophthalmitis can occur through hematogenous spread from a remote site. Highest risk is in chronically ill, diabetic, or immunosuppressed patients. Not all patients have pain, and sometimes, visual loss is the only symptom. C. Orbital cellulitis often rises from contiguous spread of infection from the ethmoid sinus. Patients present with pain, lid erythema, proptosis, conjunctival chemosis, fever, leukocytosis, decreased acuity, and restricted extraocular muscle movement. D. Uveitis is divided into anterior or posterior compartment conditions. Anterior uveitis presents with pain, redness, and variable degrees of visual loss. Frequently associated with systemic disease, posterior uveitis presents as painless visual changes.

253. A 3-year-old child playing in an abandoned shed is bitten by a black widow spider. The mother rushes the child to the emergency department within 20 minutes of the incident. What is the initial treatment? A. intramuscular steroids B. administration of antivenin C. narcotic analgesic and methocarbamol D. pain relief with acetaminophen

253. The answer is C [Dermatology, Pediatrics]. A. The bite of this spider does not cause infl ammatory reaction; therefore, steroids are not indicated. B. Antivenin is derived from horse serum and commonly causes a hypersensitivity reaction; it should be reserved for the very young or elderly if they do not respond to analgesics and methocarbamol. C. The black widow injects a neurotoxin, which causes depletion of acetylcholine at the neuromuscular junction. Initial treatment of a black widow bite in a child consists of administration of strong analgesics, methocarbamol to relax muscles, and possibly calcium gluconate to relieve rigidity, although its effectiveness is controversial. If the patient does not respond to treatment, antivenom may be initiated. D. The child will need to be admitted for observation and treatment. Acetaminophen (Tylenol) for the expec

254. Which of the following women has the greatest risk for developing endometrial hyperplasia? A. a 54-year-old woman with a long history of anovulatory cycles B. a 33-year-old woman with a 10-year history of oral contraceptive use C. a young woman suffering from anorexia nervosa D. an obese woman with normal menses occurring every 30 days

254. The answer is A [Ob/Gyn]. A. A woman with a long history of anovulatory cycles is at high risk for endometrial hyperplasia due to unopposed estrogen causing endometrial proliferation. Progesterone is only produced after ovulation, resulting in secretory endometrium that prevents further endometrial proliferation. B. Oral contraceptives, because they contain progesterone, decrease the risk of endometrial hyperplasia. C. Patients with anorexia nervosa have amenorrhea due to extreme weight loss. They are not producing enough estrogen and thus are not at risk for endometrial hyperplasia despite amenorrhea. D. Although obesity increases the risk of endometrial hyperplasia, it would be uncommon for a woman having normal menses every 30 days to not be ovulating on a regular basis. It is the lack of ovulation ( unopposed estrogen) that increases the risk of endometrial hyperplasia.

255. A 16-year-old presents with fever, sore throat, fatigue, malaise, anorexia, and myalgia. Examination reveals anterior and posterior cervical lymphadenopathy, exudative pharyngitis, and splenomegaly. What is the most likely etiologic agent? A. cytomegalovirus B. Epstein-Barr virus C. rubeola virus D. varicella virus

255. The answer is B [EENT, Infectious Disease]. A. Cytomegalovirus (CMV) and Epstein-Barr virus (EBV) are members of the herpes family. Although many of the symptoms listed earlier can occur with infection by either organism, CMV only rarely causes an exudative pharyngitis or cervical lymphadenopathy. B. EBV is the cause of mononucleosis, a disease spread by salivary secretions. Lymphadenopathy (particularly the posterior chain) occurs in greater than 90% of patients, whereas pharyngitis occurs in about 80% and splenomegaly occurs in half. C. Rubeola (measles virus) and varicella viruses cause febrile exanthems. D. See C.

256. A 22-year-old male awoke with stiffness and pain on the left side of his face. Examination reveals drooping of the lower eyelid and loss of the nasolabial fold. Which nerve is affected? A. abducens B. trigeminal C. facial D. hypoglossal

256. The answer is C [Neurology]. A. See C. B. See C. C. Bell palsy is an idiopathic paresis of the facial nerve secondary to an infl ammatory reaction. It may represent a reactivation of herpes simplex infection, but this has not been proven. D. See C.

261. A 56-year-old sustains a fracture of the left femur from a fall. While awaiting surgery, he suddenly complains of chest pain and severe dyspnea. SaO2 has fallen to 86%. ECG shows sinus tachycardia and nonspecifi c ST- and T-wave changes. Chest x-ray is normal. D-dimer is elevated. What type or description of the chest pain is this patient most likely experiencing? A. burning or gnawing B. pleuritic C. squeezing D. stabbing

261. The answer is B [Cardiovascular, Pulmonology]. A. See B. B. The pain of a pulmonary embolus is most likely pleuritic and increases with respiratory movements. C. See B. D. See B.

257. A 28-year-old male presents to the emergency department with confusion and agitation. He appears intoxicated. He is unable to provide a useful history. Vitals reveal temperature of 39°C, pulse rate of 174 bpm, respiratory rate of 29, and blood pressure of 136/68 mm Hg. He has mild proptosis, is tachycardic with regular rhythm, and has fullness in his neck. He has a fi ne tremor, brisk refl exes, and mild to moderate weakness of all four extremities. While in the emergency department, he vomits three times. Serum electrolytes, kidney function, liver function, thyroid studies, and toxicology screen are all within normal limits. What is the most likely diagnosis? A. Addison disease B. diabetic ketoacidosis C. thyroid storm D. toxic adenoma

257. The answer is C [Endocrinology]. A. Addison disease results in signs and symptoms of adrenal insuffi ciency (headache, lassitude, abdominal pain, fever, confusion, N/V, diarrhea), usually with insidious onset. B. Diabetic ketoacidosis is a life-threatening complication of diabetes. Patients have severe electrolyte imbalances and high serum glucose. C. Thyroid storm is a life-threatening complication of hyperthyroidism. Patients may develop fever, tachycardia, agitation, restlessness, delirium, and even coma. Gastrointestinal symptoms include abdominal pain, vomiting and diarrhea, and dehydration. Patients often have tachycardia with widened pulse pressure, which may progress to atrial fi brillation and even cardiac failure. Thyroid storm is often a clinical diagnosis as thyroid studies may be normal. Therefore, index of suspicion must remain high. Acute management consists of close monitoring, fl uid replacement, cooling, antithyroid drugs, and glucocorticoids. D. Toxic adenoma may present with symptoms of thyroiditis in the presence of solitary thyroid nodules, decreased TSH, and elevated T4.

258. A 41-year-old male presents complaining of easy fatigability. On physical exam, a wide fi xed split S2 and a Grade II/VI systolic murmur is heard over the pulmonic region. Which of the following will be most benefi cial in the diagnostic evaluation? A. Obtain an ECG. B. Obtain a chest x-ray. C. Refer for cardiac catheterization. D. Refer for echocardiography.

258. The answer is D [Cardiovascular]. A. An ECG may show right-axis deviation and incomplete bundle branch block but is not defi nitive for diagnosis of an atrial septal defect, which was manifested classically in this patient. B. A chest x-ray may demonstrate increased pulmonary vascular markings but is not defi nitive for the diagnosis of atrial septal defect. C. Oximetry performed during cardiac catheterization will reveal evidence of a signifi cant increase in oxygen saturation at the atrial level, but direct visualization of the atrial septal defect by echocardiography may eliminate any need for cardiac catheterization. D. Direct visualization of the atrial septal defect by two-dimensional echocardiography and demonstration of a leftto- right shunt through the defect by color-fl ow Doppler confi rms the diagnosis of atrial septal defect.

259. A 14-year-old male is noted to be limping after track practice. Examination reveals swelling and tenderness of the left tibial tuberosity. Radiography is negative for fracture but reveals some overlying soft-tissue swelling and ossifi cation of the tibial tuberosity. What is the most likely diagnosis? A. jumper's knee B. Osgood-Schlatter disease C. osteochondral fracture D. torn medial meniscus

259. The answer is B [MSS/Rheumatology, Pediatrics]. A. Jumper's knee is patellar tendonitis, an overuse injury that presents with pain and stiffness. B. Osgood-Schlatter disease is typically seen in 12- to 14-year-olds. It is a variant of traction apophysitis and occurs when the muscle associated with the inserting tendon becomes tight. Stretching of the quadriceps muscle will reproduce the pain. C. Osteochondral fracture is a tear of the articular cartilage resulting in immediate pain, swelling, and inability to bear weight. D. A torn meniscus results in pain and swelling and a sensation of knee locking.

260. A patient gives a history of vertigo episodes over the last several years. Each episode occurs about 10 seconds after quick movement of the head and lasts about 1 minute. An MRI done a year ago was negative. What is the recommended management? A. Perform maneuvers that provoke vertigo. B. Lie completely still and rest until the episode resolves. C. Continue normal activities as usual. D. Hyperventilate for 1 to 3 minutes when episodes begin.

260. The answer is A [Neurology]. A. Patients with chronic positional vertigo may respond to habituation maneuvers or physical therapy routines. Dix-Hallpike maneuvers can be diagnostic as well as therapeutic. B. Physical activity tends to enhance the CNS ability to compensate for labyrinthine dysfunction. Bed rest is indicated if nausea and vomiting are present. C. Continuance of activities is recommended but will not improve the symptomatology. D. Hyperventilation is useful in eliminating anxiety or emotional states which may cause or accompany the vertigo.

262. A 23-year-old male presents for evaluation of a productive cough that has been present for the past 3 weeks. He has additional complaints of fever and night sweats, weight loss, anorexia, general malaise, and weakness. Sputum Gram stain fails to reveal any organisms. What is the most likely etiologic agent? A. Bordetella pertussis B. Mycoplasma pneumoniae C. Mycobacterium tuberculosis D. Streptococcus pneumoniae

262. The answer is C [Infectious Disease, Pulmonary]. A. Whooping cough produces characteristic paroxysmal coughing spells, which cause gasping inhalations (the whoop). Bordetella pertussis is a Gram-negative coccobacillus. B. Mycoplasma lacks a cell wall and does not stain with Gram staining. Patients may present with cough disproportionate to lung fi ndings. Night sweats and weight loss are rare. C. Mycobacterium tuberculosis is a rod-shaped, non- spore-forming, obligate aerobic bacterium. It is associated with negative Gram staining and positive acid-fast staining. Presumptive diagnosis is made until cultures return. D. Streptococcus pneumoniae would appear as Grampositive cocci in chains.

263. A 27-year-old female complains of worsening dysmenorrhea and dyspareunia for the last 2 years. The pain has become constant and aching and is worse around the time of menses. Which of the following tests would provide the most defi nitive diagnosis? A. abdominal CT scan B. hysteroscopy C. laparoscopy D. pelvic ultrasonography

263. The answer is C [Ob/Gyn, Surgery]. A. CT scan does not adequately identify endometriosis. An MRI may provide more detailed mapping after identifi cation through ultrasound and before defi nitive diagnosis through laparoscopy. B. Hysteroscopy evaluates abnormal vaginal bleeding in the presence of a negative dilation and curettage or to evaluate infertility. C. Diagnostic laparoscopy is the only way to defi nitively diagnose endometriosis because many patients do not have any abnormal fi ndings on physical exam. Visualization is diagnostic. D. Although pelvic ultrasonography may show signs of endometriosis (such as endometrioma), this is the rare exception.

264. A 52-year-old male presents with unilateral left-sided hearing loss that has progressed over 2 months. His wife states that he has diffi culty discriminating words, although he can hear the sounds. He has also experienced some imbalance lately, but he contributes this to trying to do too many things too quickly. Which of the following is the likely pattern of testing on physical exam? A. Rinne air conduction greater than bone conduction; Weber lateralizes to left. B. Rinne air conduction greater than bone conduction; Weber lateralizes to right. C. Rinne bone conduction greater than air conduction; Weber lateralizes to right. D. Rinne bone conduction greater than air conduction; Weber lateralizes to left.

264. The answer is B [EENT]. A. See B. B. This patient likely has an acoustic neuroma. Sensorineural hearing loss will result in air conduction greater than bone conduction and contralateral Weber. C. See D. D. In conductive hearing loss, the Rinne will result in bone conduction greater than air conduction and Weber will lateralize ipsilaterally.

265. A 33-year-old presents complaining of cough. She had 2 days of cold symptoms (scratchy throat, congestion, postnasal drip) 6 days ago, which have cleared. She is left with an irritative cough that comes in spasms throughout the day. Which of the following is the recommended management at this time? A. dextromethorphan B. guaifenesin C. hydrocodone D. phenylephrine

265. The answer is A [Pulmonology, Infectious Disease]. A. Dextromethorphan is available without a prescription, is nonnarcotic, and may be useful in the treatment of patients with persistent coughing secondary to postviral bronchitis. Cough that persists longer than 2 weeks should prompt suspicion of pertussis. B. Guaifenesin is classifi ed as a mucolytic and is purported to be useful in patients with inspissated (dense) mucus. C. Hydrocodone is helpful to patients with paroxysmal cough that keeps them awake at night; it is a narcotic and should be used judiciously. D. Phenylephrine is a sympathomimetic and would be useful in patients whose primary complaint was congestion.

266. In early intestinal obstruction, what are the characteristic initial fi ndings on abdominal auscultation? A. absent bowel sounds B. diminished bowel sounds C. hyperactive bowel sounds D. low-pitched, rumbling bowel sounds

266. The answer is C [GI/Nutrition, Surgery]. A. Obstipation and absent bowel sounds are seen late in intestinal obstruction. B. See C. C. Hyperactive bowel sounds are very common early in the course of intestinal obstruction. They are described as high-pitched with occasional rushes (crescendo- decrescendo). D. See C.

267. Which of the following is the most appropriate test to differentiate chronic sinusitis from a tumor? A. CT without contrast B. sinus radiographs C. MRI with contrast D. sinus endoscopy

267. The answer is C [EENT, Infectious Disease]. A. Limited coronal CT is inexpensive and sensitive in detecting infl ammatory changes and bone destruction, which would raise suspicion for a tumor, but it is not as sensitive as MRI with gadolinium. B. Sinus radiographs are more sensitive than physical exam in diagnosing sinusitis but are not cost-effective. They are not helpful in distinguishing tumor. C. MRI with gadolinium is much more sensitive than CT to differentiate tumor from fl uid, infl ammation, and inspissated mucus. D. Sinus endoscopy is useful in evaluating and treating anatomic blockage of the osteomeatal complex.

268. A 35-year-old skier reports knee pain and instability after a sudden stop. Which test would be best to confi rm the suspected diagnosis? A. Apley B. Lachman C. McMurray D. Thompson

268. The answer is B [MSS/Rheumatology, Surgery]. A. Apley test assesses the meniscus. B. The Lachman test assesses the anterior cruciate ligament which is commonly injured during sudden stress to the knee. C. The McMurray test assesses the meniscus. D. Thompson test assesses the Achilles' tendon.

273. A 55-year-old who drinks alcohol daily and smokes 1 pack per day recently attended a high school reunion at a large hotel. During the reunion, the hotel experienced numerous problems with their air-conditioning system. Three days later, he began to experience nausea, vomiting, headache, fever, and productive cough associated with pleuritic chest pain and dyspnea. Examination reveals temperature of 103.8°F, pulse rate of 64 bpm, and scattered rales and rhonchi. What is the recommended treatment? A. doxycycline B. levofl oxacin C. penicillin D. rifampin

273. The answer is B [Pulmonology, Infectious Disease]. A. Doxycycline is a second-line treatment against Legionella. B. Azithromycin or a fl uoroquinolone are considered fi rstline treatment in Legionella pneumonia. C. Penicillin is not effective against Legionella. D. Rifampin may be added in very severe cases but is not a fi rst-line treatment or initial therapy

269. A 5-year-old girl presents with sore throat, sudden onset fever, and upper respiratory symptoms. On examination, she has mild inspiratory stridor and is leaning forward and drooling. She appears toxic. Vitals reveal a temperature of 102.5°F, respiratory rate of 26, and heart rate of 110 bpm. What is the next step in managing this patient? A. administration of aerosolized racemic epinephrine B. endotracheal intubation C. lateral neck radiography D. oropharyngeal examination using a tongue blade

269. The answer is B [Pulmonary, Infectious Disease, Pediatrics]. A. Aerosolized racemic epinephrine is typically used in the treatment of croup. B. Epiglottitis constitutes a medical emergency in children. Direct fi beroptic laryngoscopy is frequently performed in a controlled environment (e.g., an operating room) in order to visualize and culture the typical edematous "cherry red" epiglottis and to facilitate placement of an endotracheal tube. Immunization against Haemophilus infl uenzae has resulted in a dramatic decrease in incidence of epiglottitis; cases occur in underimmunized or in specifi c Streptococcus spp. pharyngitis. C. Lateral neck radiography and laboratory tests can assist in the diagnosis but may delay the critical securing of the airway and cause the patient to be moved or repositioned more than is necessary, thereby increasing the risk of further airway compromise. However, these tests may be useful in distinguishing croup from epiglottitis. D. Direction visualization in an examination room (e.g., with a tongue blade and indirect laryngoscopy) is not recommended because of the risk of immediate laryngospasm and complete airway obstruction.

270. A 27-year-old female presents complaining of left fl ank pain that has been present for the past 8 hours. She states that for the past couple of days, she has been experiencing frequency and dysuria and today began to experience fever and chills. Examination reveals a fever of 102.0°F with left costovertebral angle tenderness. What is the most likely diagnosis? A. glomerulonephritis B. nephrolithiasis C. pyelonephritis D. urinary tract infection

270. The answer is C [GU/Renal, Infectious Disease]. A. Glomerulonephritis causes an elevation in blood pressure and edema (periorbital, scrotal, perineal). Urinalysis may show hematuria, proteinuria, and red blood cells. B. Nephrolithiasis (kidney or renal stone) causes fl ank pain that radiates to the groin along with hematuria and red cell casts. C. Acute pyelonephritis is an infection involving the renal parenchyma or renal pelvis. Patients experience fever, fl ank pain, shaking chills, and voiding symptoms. Exam reveals costovertebral tenderness. Urinalysis may show white cell casts. D. Urinary tract infection causes voiding symptoms but not costovertebral tenderness. It generally occurs prior to development of pyelonephritis. Urinalysis will not include casts.

271. Which of the following is the best description of erysipelas? A. a group of small, pinpoint, follicular pustules found on the upper outer aspect of the upper arms or anterior thighs that often remain unchanged for years B. localized painful, distinctly demarcated, raised erythema and edema often with streaking and prominent lymphatic involvement C. one pustule or a group of pustules, usually without fever or other systemic symptoms, which may be found on any skin surface D. originates as a small vesicle or pustule that ruptures to expose a red, moist base; a crust of honey-yellow to white-brown fi rmly adheres as the lesion expands

271. The answer is B [Dermatology, Infectious Disease]. A. This describes keratosis pilaris. B. This is the classic presentation of erysipelas. C. This describes folliculitis, most likely secondary to Staphylococcus. D. This describes nonbullous impetigo, most likely secondary to Streptococcus.

272. A patient complains of a strange motor phenomenon involving her left arm. The fi ngers of her left hand begin to jerk spasmodically, which progressively involves the forearm, upper arm, and shoulder, until the whole left upper extremity displays this activity. She does not lose consciousness. This is characteristic of which of the following types of seizure? A. grand mal B. petit mal C. focal motor D. myoclonic

272. The answer is C [Neurology]. A. Grand mal seizures involve tonic-clonic movements bilaterally with sudden loss of consciousness. B. Petit mal, or absence, seizures are characterized by brief lapses in consciousness. C. A common type of focal motor seizure (previously referred to as simple partial seizure) involves motor symptoms that begin distally and progress to involve an entire limb or body. Other types of focal seizures involve sensory or autonomic symptoms or affective disturbances or hallucinations. All focal seizures share the feature of no loss of consciousness. D. Myoclonic seizures consist of single or multiple myoclonic jerks without a loss of consciousness.

274. A 45-year-old with multiple health problems is concerned with an unintentional 10-lb weight gain and extreme fatigue that has progressively worsened over the last 4 months. History and physical exam fi ndings are suspicious for hypothyroidism and are substantiated with the laboratory evaluation. Which of the following currently prescribed medications taken for other disorders should this patient discontinue, if possible? A. fl uoxetine (Prozac) B. hydrochlorothiazide (HydroDIURIL) C. lithium (Lithobid) D. propranolol (Inderal)

274. The answer is C [Endocrinology, Psychiatry]. A. Although fl uoxetine (Prozac) has the potential for inducing weight gain and fatigue with chronic use, this patient's diagnosis of hypothyroidism is more likely the cause of the symptoms and signs found on physical exam. B. Hydrochlorothiazide, a potassium-wasting diuretic, can cause fatigue but is often associated with a transient weight loss. C. Lithium (Eskalith, Lithobid), a medication used for controlling bipolar disorder, is a potential goitrogen and should be discontinued under the careful supervision of a psychiatrist until the exact cause of this patient's hypothyroidism is established. D. Propranolol, a b-blocker, is usually part of the treatment regimen for hyperthyroid disorders to control the sympathetic response elicited by the presence of excessive thyroid hormone. It is not known to cause hypo thyroidism

275. A 48-year-old homeless male presents to the indigent care clinic with abdominal pain and tenderness, anorexia, and nausea. Examination is signifi cant for palpation of the liver border 5 cm below the right inferior costal margin at the midclavicular line. He admits to drinking 1 to 3 bottles of liquor per day. His skin and sclerae are jaundiced. AST, ALT, and GGTP are elevated threefold above normal. Besides abstinence from alcohol, what is the most effective treatment measure for this patient? A. pentoxifylline B. cholestyramine C. immune globulin D. pegylated interferon

275. The answer is A [GI/Nutrition]. A. The clinical presentation of alcoholic liver disease can vary from an asymptomatic hepatomegaly to a rapidly fatal acute illness or end-stage cirrhosis. A recent period of heavy drinking, complaint of anorexia and nausea, and the demonstration of hepatomegaly and jaundice strongly suggest the diagnosis. Abstinence from alcohol is essential. Pentoxifylline reduces the risk of hepatorenal syndrome. B. Cholestyramine is a lipid-lowering agent. C. Immune globulin may be benefi cial in acute viral hepatitis. D. Pegylated interferon is indicated in chronic viral hepatitis.

276. A 41-year-old female with long-standing GERD undergoes endoscopy, which reveals orange, gastric type epithelium present in tongue-shaped lesions extending upward from the distal end of the esophagus. How often and by what method should this patient undergo screening for cancer? A. ambulatory pH monitoring every 1 to 3 years B. ambulatory pH monitoring every 3 to 5 years C. upper endoscopy every 1 to 3 years D. upper endoscopy every 3 to 5 years

276. The answer is D [GI/Nutrition, Oncology]. A. Ambulatory pH monitoring is suited to document acid refl ux. It is useful when the endoscopic fi ndings are normal to evaluate symptomatic patients unresponsive to proton pump therapy or for patients considering antirefl ux surgery. It has no role in screening. B. See A. C. See D. D. Once Barrett esophagus has been identifi ed, screening every 3 to 5 years by upper endoscopy is recommended to look for dysplasia or adenocarcinoma. There is an 11-fold increase in esophageal adenocarcinoma in patient with Barrett esophagus.

277. A 30-year-old female complains of involuntary movements of her hands and head which have developed over the last 6 months. The movements are rhythmic and are worsened with emotional stress and lessened with alcohol intake. What else is likely in this patient's history? A. exposure to high-dose estrogen B. family history of similar complaints C. gait disturbances D. progressive hoarseness

277. The answer is B [Neurology]. A. High-dose estrogen has not been correlated with development of neurologic syndromes. B. Essential tremor is an autosomal dominant disorder. The tremors can begin at any age and characteristically affect the hands, head, or voice. C. The legs are spared in benign essential tremor. D. The voice may be affected by a tremor, but hoarseness does not develop.

278. A 40-year-old female presents for evaluation of a fi ne tremor in both hands, which she says has been present for years but has become more noticeable in the last few years. Her father and sister have the same tremor. Which of the following is the best treatment to recommend to help control the tremor? A. alcohol B. alprazolam C. primidone D. propranolol

278. The answer is D [Neurology]. A. A familial benign tremor will often subside with alcohol, but this is not prudent for chronic treatment. B. Alprazolam is very effective but is addictive. C. Primidone is effective, but the side-effect profi le is great; it should be reserved for those who do not respond to propranolol. D. Propranolol is effective against benign familial tremor. It can be used intermittently or on a continuous basis.

279. A 52-year-old G1P0010 with chronic obesity and irregular menses presents for a checkup. She reports having mild hot fl ushes for 6 months. For the past 14 months, she has had some irregular bleeding, which she characterizes "like a period," which occurs at 2- to 4-week intervals and lasts up to 10 days. She believes that the bleeding has been brought on by stress; she has some diffi culty sleeping and increased emotional lability. Physical exam: height, 5 ft 2 inches; weight, 234 lb; BP, 160/90 mm Hg; and pulse rate, 88 bpm and regular. External genitalia is mildly atrophic. Cervix is parous. A small amount of dark blood is noted in the cervical os. The uterus is about 12-week size, irregular in contour, but diffi cult to delineate due to the patient's body habitus. In what life stage would this patient be considered? A. menarchal B. perimenopausal C. premenopausal D. postmenopausal

279. The answer is B [Ob/Gyn]. A. Menarchal indicates someone who just started menstruating, usually a young teen. B. This patient is considered perimenopausal because she is still having periods, although not regular, and she is also having other menopausal symptoms such as hot fl ashes, trouble sleeping, and increased emotional lability. C. This patient is not considered premenopausal because she is having some signs and symptoms of menopause. D. Because this patient has not gone without a period for at least 12 months (the standard definition of menopause), she is not considered postmenopausal.

280. A 56-year-old complains of eye pain and tearing as well as swelling and redness in the medial portion of his left eye. There is tenderness over the tear sac area with purulent material easily expressed. What is the best treatment for this condition? A. dacryocystorhinostomy B. probing of the nasolacrimal system C. systemic antibiotics D. topical antibiotics

280. The answer is A [EENT]. A. Adults who develop dacryocystitis will respond to systemic antibiotics, but recurrence is the rule. Relief of the obstruction is the only cure; dacryocystorhinostomy is the procedure of choice. B. Only infants with congenital nasolacrimal duct obstruction that does not resolve spontaneously will need to have the nasolacrimal system explored using a special probe. C. See A. D. Chronic dacryocystitis may be kept in check with topical antibiotics until such time as relief of the obstruction can be obtained

281. A 60-year-old awoke with acute unilateral visual loss. He denies pain, headache, and nausea or vomiting. Funduscopic examination reveals vessel dilation, intraretinal hemorrhages, and cotton-wool spots. What is the most likely diagnosis? A. retinal artery occlusion B. hypertensive retinopathy C. macular degeneration D. retinal vein occlusion

281. The answer is D [EENT, Cardiovascular]. A. Funduscopic fi ndings in retinal artery occlusion reveal an ischemic, opaque retina. Patients complain of sudden profound painless visual loss. B. Hypertensive retinopathy may have intraretinal hemorrhages with cotton-wool spots but does not present with acute visual loss. C. Macular degeneration gives a mottled appearance to the macula. Visual loss is generally not acute. D. Venous dilation and tortuosity, hemorrhages, and cottonwool spots with acute painless unilateral visual loss are the expected fi ndings in retinal vein occlusion. Occlusion is most common in older patients with chronic atherosclerotic disease or hyperviscosity.

282. A 29-year-old presents to primary care complaining of increased thirst and increased fl uid intake. He has a history of signifi cant head trauma due to a motor vehicle accident 6 months ago. He is drinking more than 90 oz of fl uid per day and urinates frequently throughout the day and night. Physical exam is normal. Electrolytes are within normal limits. Serum osmolality is normal. BUN and creatinine are low. Random urine studies reveal the following: Urine dipstick: negative for heme, protein, ketones, and glucose Urine specifi c gravity: 1.000 (normal 1.002 to 1.035) Urine pH: 5.1 (normal 5.0 to 9.0) What is the most likely diagnosis? A. diabetes insipidus B. diabetes mellitus C. metabolic syndrome D. pituitary adenoma

282. The answer is A [Endocrinology]. A. Central diabetes insipidus is caused by decreased ADH secretion and subsequent diuresis. This disease has many causes, including head trauma. This patient has increased volume with decreased urine concentration. The next test to be completed would be a desmopressin challenge test. B. Diabetes mellitus type 2 would cause an increased fasting glucose. C. Metabolic syndrome is identifi ed in patients with obesity, lipid abnormalities, hypertension, and insulin resistance. D. A pituitary adenoma generally affects the anterior pituitary gland, most commonly presenting as a prolactinoma. Males typically complain of headache, loss of libido, and visual disturbances.

283. Which of the following is an indication to refer an asymptomatic adolescent for colonoscopy? A. brother who is 25 years old has been diagnosed with ulcerative colitis B. father developed multiple strictures after bowel resection at age 32 years C. family history of colon cancer in maternal uncle at age 36 years D. mother had colorectal adenomatous polyps removed at 19 years of age

283. The answer is D [GI/Nutrition, Surgery]. A. Ulcerative colitis is not known to be familial. B. Strictures after bowel resection are due to the adhesions and scarring, which may carry a familial tendency but do not develop without trauma. C. Screening for colon cancer in individuals with a family history of colorectal cancer (not familial polyposis syndromes) should begin at age 40 years or 10 years younger than the age at diagnosis of the youngest affected relative. D. Familial adenomatous polyposis (FAP) is characterized by the development of hundreds to thousands of colonic adenomatous polyps. Colorectal polyps develop by a mean age of 15 years and cancer at 40 years. First-degree relatives of patients with FAP should undergo genetic screening after age 10 years. If the assay cannot be done or is not informative, family members at risk should undergo yearly sigmoidoscopy beginning at 12 years of age.

302. A 12-year-old complains of sore throat. Examination reveals edema and erythema of pharyngeal mucosa with gray-white exudate, cervical adenopathy, and a temperature of 101.0°F. She is allergic to penicillin. What is the recommended management? A. supportive care only B. erythromycin ethyl succinate (EES) C. amoxicillin-clavulanate potassium (Augmentin) D. cephalexin (Kefl ex)

302. The answer is B [EENT, Infectious Disease]. A. This child meets the Centor criteria for probable streptococcal pharyngitis and should be treated with antibiotics. B. Erythromycin is an effective alternative for patients allergic to penicillin. Penicillin is fi rst-line therapy for streptococcal pharyngitis. C. Augmentin is derived from penicillin and may cause anaphylaxis in a patient with a known allergy to penicillin. D. Cephalosporins have the potential for cross-reactivity in patients with a penicillin allergy.

284. A college student presents for evaluation of lower abdominal pain and increased vaginal discharge with "an embarrassing odor." A swab of mucopurulent cervical discharge is stained, and, when examined, reveals abundant neutrophils and Gram-negative intracellular diplococci. What is the recommended treatment? A. ceftriaxone (Rocephin) and acyclovir (Zovirax) B. ceftriaxone and azithromycin (Zithromax) C. ceftriaxone and ciprofl oxacin (Cipro) D. ceftriaxone and penicillin G

284. The answer is B [Infectious Disease, Ob/Gyn]. A. This regimen will not cover coinfection with chlamydia. Acyclovir is appropriate for treatment of herpes. B. This patient likely has gonorrhea. Appropriate treatment is ceftriaxone or cefi xime. Infected patients should be empirically treated for chlamydia infection as well. Azithromycin and doxycycline are the recommended agents. C. This regimen will not cover coinfection with chlamydia. Fluoroquinolones are not recommended for gonococcal infections due to resistance. D. This regimen will not cover coinfection with chlamydia. Penicillin G is appropriate for treatment of syphilis.

285. A 42-year-old obese female has tried diet and exercise for the past 4 months after a diagnosis of type 2 diabetes. She returns to the clinic for follow-up. Her hemoglobin A1c is 7.0, and her serum creatinine is normal; however, she presents with a 10-lb weight gain since her last visit. BMI is 30.5 kg/ m2. Which of the following treatment options would be most appropriate at this time? A. bedtime insulin (NPH) B. metformin (Glucophage) C. glipizide (Glucotrol) D. continuation of diet and exercise

285. The answer is B [Endocrinology]. A. NPH insulin may be recommended when patients fail oral therapy for type 2 diabetes. B. Metformin along with diet and exercise is recommended for the treatment of obese patients with type 2 diabetes and a normal serum creatinine. C. Sulfonylureas are no longer considered a fi rst-line agent in a patient who is obese and has diabetes. D. The patient has already tried diet and exercise for 4 months. She is still hyperglycemic and continues to put on weight.

286. A 63-year-old male presents with sudden onset of a painful, red left eye. He also reports a throbbing frontal headache, blurred vision with halos around lights, and nausea without vomiting. On examination, there is ciliary fl ush, a normal conjunctiva, and cloudiness of the cornea. The left pupil is mid-dilated and fi xed. What is the most likely diagnosis? A. acute angle-closure glaucoma B. retinal detachment C. subarachnoid hemorrhage D. temporal arteritis

286. The answer is A [EENT, Geriatrics]. A. Acute angle-closure glaucoma presents as a painful red eye. Abrupt onset of symptoms includes pain, injection, corneal edema, and blurred vision. In some patients, ocular symptoms are overshadowed by nausea, vomiting, or headache. B. Retinal detachment presents with fl oaters, fl ashing lights, and scotomata in the peripheral visual fi eld corresponding to the detachment. Risk factors include myopia, previous cataract surgery, prior detachment, advanced age, family history, and trauma. C. Subarachnoid hemorrhage presents as severe headache associated with exertion in 45% of cases. It is often described as the "worst headache of my life." Sudden onset of generalized headache along with neck stiffness and vomiting is common. D. Temporal arteritis occurs almost exclusively in patients older than the age of 50 years and is more common in women than in men. The clinical picture is fever, headache, scalp pain, and claudication of the jaw and tongue.

287. A 46-year-old male has recently been diagnosed with metabolic syndrome. Extensive evaluation revealed no structural heart disease and he currently has no dyspnea on exertion, orthopnea, edema, or paroxysmal nocturnal dyspnea. His father, older brother, and paternal grandfather all died in their early 60s of heart failure. Of the following recommendations for prevention of heart failure, which is most appropriate at this time? A. encouraging regular exercise B. implanting a defi brillator C. initiating a _-blocker D. severely restricting dietary salt

287. The answer is A [Cardiovascular, Endocrinology]. 287. The answer is A [Cardiovascular, Endocrinology]. A. This man has stage A heart failure; that is, he is at risk for failure but without structural heart disease or symptoms of failure. Appropriate measures include encouraging exercise, treating lipid disorders, discouraging alcohol and smoking, and controlling hypertension and metabolic syndrome. B. Implanting a defi brillator is appropriate for persons in stage B or C failure. C. Initiation of a b-blocker is appropriate for certain persons in stage B. D. Severely restricting dietary salt is recommended for persons in stage C.

288. A 62-year-old female complains of pain and stiffness of the shoulders and hips along with malaise, weight loss, and lowgrade fever over the past month. Muscle strength is normal on examination. Passive range of motion is limited in all directions. Erythrocyte sedimentation rate is elevated. What is the best choice for treatment? A. ibuprofen B. prednisone C. indomethacin D. aspirin

288. The answer is B [MSS/Rheumatology, Geriatrics]. A. Ibuprofen and aspirin provide only mild to moderate relief of symptoms. B. Symptoms of polymyalgia rheumatica respond quickly and dramatically to steroid administration. Because of this, dosages can usually be rapidly weaned to low-maintenance levels, minimizing side effects. Treatment may need to be continued for a year. C. Although it is a good anti-infl ammatory agent, indomethacin has signifi cant side effects which limit its usefulness. D. See A.

289. A 63-year-old male complains of acute loss of vision in the right eye that lasted about 2 to 3 minutes and completely resolved. He denies pain, blurriness, and motor defi cits. What diagnostic study would be most helpful at this time? A. carotid Doppler recordings B. 24-hour Holter monitor C. CT scan of head D. carotid angiography

289. The answer is A [Neurology, Cardiovascular]. A. Carotid Doppler recordings are noninvasive and very sensitive to the degree of plaque and stenosis of the carotid arteries. This patient likely has experienced amaurosis fugax. The most likely source is the ipsilateral carotid artery. B. An ECG is warranted to asses for atrial fi brillation, a common cause of cerebral emboli. If ECG is normal and the carotids are clean, a Holter may be done to assess for infrequent arrhythmias. C. This patient has experienced a transient ischemic attack (TIA); a CT scan would be negative unless bleeding or infarction has occurred. D. Carotid angiography is a risky invasive procedure that may result in stroke; it should be reserved for those in whom diagnosis is uncertain

290. A 54-year-old male presents to the emergency department after a syncopal episode. The patient states that this is his third such episode in the past 4 weeks. He also describes episodes of dyspnea on exertion that occur with substernal chest pain. Physical exam reveals a harsh III/VI systolic crescendo- decrescendo murmur heard best at second right intercostal space that radiates to the carotid arteries. It is heard best when the patient is sitting forward. What is the most likely diagnosis? A. aortic stenosis B. mitral stenosis C. pulmonary stenosis D. tricuspid stenosis

290. The answer is A [Cardiovascular, Surgery]. A. Aortic stenosis most commonly occurs due to a congenital unicuspid or bicuspid valve; rheumatic heart disease; or degenerative changes brought on by hypertension, hyperlipidemia, and smoking. Syncope is usually a late fi nding and occurs in association with exertion. A systolic ejection murmur heard best in the aortic area with radiation to the neck is characteristic. The murmur peaks at midsystole. B. Mitral stenosis produces dyspnea with exertion but is associated with an opening snap after S2 and a diastolic murmur best heard in the apex with the patient in the left lateral position. C. Pulmonary stenosis is associated with congenital defects. Dyspnea on exertion and right heart failure are common. There is typically a loud harsh systolic murmur with lift and thrill. It radiates to the left shoulder and neck. An early systolic sound is also common. D. Tricuspid stenosis is associated with right failure, a loud S1, and a diastolic rumble murmur which increases with inspiration

291. During the winter semester, a 20-year-old student complains of abrupt onset of sore throat, fever, malaise, headache, nasal congestion, myalgias, and dry cough. Examination reveals pharyngeal injection and clear lungs. What is the most likely diagnosis? A. bacterial pneumonia B. severe acute respiratory syndrome (SARS) C. mononucleosis D. infl uenza

291. The answer is D [Pulmonology, Infectious Disease]. A. With bacterial pneumonia, the cough more likely will produce purulent sputum. B. SARS causes similar symptoms, but the lungs would not be clear; scattered rales and rhonchi are characteristic. C. Mononucleosis more typically has an insidious onset and lack of congestion. Cough is atypical. Fever is low grade, if present. Fatigue and malaise are the most frequent complaints. D. Infl uenza usually occurs during the winter months and presents with the systemic symptoms described. The abrupt onset of symptoms and fever are hallmarks of infl uenza.

292. Seventy-two hours after total knee replacement, an otherwise healthy 57-year-old male develops tachycardia, hand tremor, and nausea. He becomes very anxious, agitated, and tells the nursing staff "millions of ants are crawling over his skin and sheets." An hour later, he has a new-onset tonic-clonic seizure. Which of the following is the most likely cause of this patient's condition? A. alcohol withdrawal B. acute delirium C. epileptic disorder D. hyperthyroidism

292. The answer is A [Psychiatry/Behavioral Medicine]. A. Often, alcohol dependence is uncovered during hospitalization and presents with seizure, increased heart rate, tactile hallucinations, agitation, and anxiousness along with tremor, nausea, and vomiting. Symptoms generally appear within hours or days after cessation or reduced alcohol intake. Patients may not be forthright about their alcohol use during prehospitalization history. B. Delirium is a syndrome and not a disease, and specifi c causes should be sought. Delirium does not present with seizures unless the cause is alcohol withdrawal. C. Epilepsy can occur from a variety of causes. In this case, the patient had a reactive seizure secondary to alcohol withdrawal. An epileptic disorder has its roots in a genetic disorder or abnormal brain function. D. Many of the signs and symptoms in this case overlap with hyperthyroidism (i.e., nervousness, tremor, tachycardia). However, the tactile hallucinations and seizures are not consistent with hyperthyroidism.

293. A 58-year-old obese female with stable exertional angina presents with an irregular, pruritic, weeping lesion just above the medial malleolus. The surrounding skin is hyperpigmented and slightly edematous. What is the most important intervention to minimize the incidence of similar lesions in the future? A. compression stockings B. daily topical steroids C. lower extremity elevation D. regular aerobic exercise

293. The answer is A [Dermatology, Cardiovascular]. A. Frequent ambulation and wearing compression stockings will reduce edema, improve venous return, and minimize the risk of future leg ulcers from venous insuffi ciency. B. Frequent use of topical steroids can lead to irreversible atrophic skin changes. C. Leg elevation will improve venous circulation, but it is not the most important preventive intervention. D. Regular aerobic exercise is important to promote weight loss but will be less effective than compression stockings as a preventive intervention against venous stasis ulcers.

294. A 4-month-old is brought to the office; mother describes 3 days of clear rhinorrhea and low-grade fever. Today, the child has developed rapid breathing and wheezes. Examination reveals shallow, rapid respirations; retractions; and nasal fl aring. Lungs are clear. What is the cornerstone of management? A. antibiotics B. humidifi ed oxygen C. racemic epinephrine D. glucocorticoids

294. The answer is B [Pulmonology, Infectious Disease, Pediatrics]. A. Routine administration of antibiotics has not proved benefi cial in bronchiolitis; the majority of cases are due to respiratory syncytial virus. B. This child has severe bronchiolitis and meets the criteria for hospitalization. The most important therapy for bronchiolitis is humidifi ed oxygen delivered via mask, hood, or tent. The child may also benefi t from a course of ribavirin. C. Racemic epinephrine and bronchodilators are frequently administered to children with severe bronchiolitis as supportive measures; however, little evidence supports their effi cacy. D. Glucocorticoids have not been shown to alter the course of bronchiolitis.

295. A 6-year-old girl recently immigrated with her parents to the United States from Central America. Last year, she had a severe febrile illness that kept her in bed for several weeks with a "heart problem" that began after she had a bad sore throat. She was treated with aspirin and penicillin. In addition, she had painful joints that improved rapidly once treatment started. Today, the child is well and physical exam is normal. In order to prevent recurrence of this disease, what is the recommendation for this child? A. antibiotic prophylaxis prior to dental procedures B. immediate antibiotic treatment of any episode of pharyngitis C. monthly injections of benzathine penicillin D. no prophylaxis is needed

295. The answer is C [Cardiovascular, Infectious Disease, Pediatrics]. A. Antibiotic prophylaxis prior to dental procedure is appropriate to prevent infectious endocarditis, not rheumatic fever. B. Prophylaxis based on symptoms of pharyngitis is not recommended. Appropriate diagnostic analysis should be followed. C. Monthly injections of benzathine penicillin are recommended in children who had rheumatic fever with carditis. This may be discontinued after 5 years of no recurrent symptoms. D. Prophylaxis is recommended for this child.

296. A patient presents to labor and delivery at 38 weeks stating her contractions have begun. She has no history of prenatal care. She complains that she has had episodes of painless, bright red vaginal bleeding about once per week since the 27th week of her pregnancy. What is the initial step in management? A. Artifi cially rupture her membranes and induce labor. B. Insert an internal monitor and perform fetal scalp blood monitoring. C. Order an ultrasound to determine placental location. D. Perform a vaginal examination to determine cervical effacement and dilation.

296. The answer is C [Ob/Gyn]. A. See C. B. See D. C. Third-trimester painless bleeding should be considered placenta previa until proven otherwise. Abruptio placentae presents with painful third- trimester bleeding. Ultrasonography will determine the location of the placenta and should be done before anything, including a bimanual or speculum examination. D. An internal examination may precipitate hemorrhage.

297. A 17-year-old male presents with pain and swelling of the right testis. He is febrile and nauseous. There is marked swelling of the parotid glands and enlargement of the submaxillary lymph nodes. Which of the following is the most effective treatment? A. systemic steroids B. surgical referral C. scrotal support D. antiviral therapy

297. The answer is C [GU/Renal, Infectious Disease]. A. Systemic steroids have only anecdotal support for use in orchitis. B. Surgery should be avoided in mumps orchitis. C. Scrotal support will provide pain relief while the condition runs its course. D. Antiviral therapy is not recommended for mumps; the risks outweigh any benefi ts. Symptomatic treatment with fl uids, analgesics, and antipyretics are recommended.

298. Which of the following is the most appropriate step in managing a patient with papillary thyroid cancer? A. chemotherapy B. radioactive iodine C. propylthiouracil D. surgical removal

298. The answer is D [Endocrinology, Surgery, Oncology]. A. Chemotherapy is not effective due to resistance by the thyroid carcinoma. B. Radioactive iodine is indicated for the treatment of multinodular goiter. C. Propylthiouracil is indicated for hyperthyroidism. D. Surgery is the treatment of choice for papillary thyroid cancer.

299. A 22-year-old female is complaining of a rash around her mouth. She describes a feeling of mild burning or tension but denies pruritus. Examination reveals papulopustules on erythematous bases; the vermillion border is spared. A culture is negative. What is the recommended management? A. ketoconazole cream B. topical metronidazole C. topical steroids D. warm compresses

299. The answer is B [Dermatology]. A. Ketoconazole is effective against fungal infections. B. Perioral dermatitis most often occurs in young women and is treated with topical metronidazole. Topical erythromycin is also effective. If the lesions do not clear, systemic treatment with minocycline, doxycycline, or tetracycline may be tried. C. Topical steroids are contraindicated as they may cause a fl are of lesions. D. Warm compresses may increase the erythema.

300. A 28-year-old female undergoes cholecystectomy. Preoperative CBC, platelet count, and international normalized ratio (INR) were normal. Excessive bloody drainage is noted 8 hours postoperative. Repeat CBC, platelet count, and INR continue to be normal; PT and PTT are normal, but bleeding time is prolonged. What is the most likely diagnosis? A. factor IX defi ciency B. disseminated intravascular coagulation C. von Willebrand disease D. sepsis

300. The answer is C [Hematology, Surgery]. A. Laboratory parameters in factor IX defi ciency include a prolonged aPTT and abnormal INR. It is an X-linked recessive disorder predominantly affecting males. B. DIC is a syndrome recognized by abnormal INR, PT, aPTT, and abnormal platelet counts. C. von Willebrand disease is an autosomal dominant trait. Mildly affected patients may not exhibit excessive bleeding tendencies until undergoing major surgical procedures or experiencing major physical trauma. Routine preoperative laboratory values are typically normal, and diagnosis is made by obtaining a bleeding time and factor VIII:C and von Willebrand factor levels. D. Manifestations of sepsis include leukocytosis or leukopenia and can precipitate DIC.

301. A 17-year-old male with a family history of hereditary spherocytosis recently experienced his second aplastic crisis. Which of the following interventions is recommended for this patient to prevent future episodes of hemolysis? A. allogeneic bone marrow transplantation B. lifelong supplementation with folic acid C. red blood cell transfusion D. splenectomy

301. The answer is D [Hematology, Pediatrics]. A. Bone marrow transplantation has been found to be helpful in the treatment of paroxysmal nocturnal hemoglobinuria. B. Folic acid supplementation is recommended for patients with risk of hemolysis, but it will not correct the underlying disorder nor prevent aplastic anemia in all cases. C. Red blood cell transfusion can be helpful during the aplastic crisis but will not prevent future episodes of this problem. D. Young patients with repeated aplastic anemia and spherocytosis should be referred for splenectomy. Splenectomy will not correct the underlying membrane defect in this disorder but will prevent hemolysis and thus another aplastic crisis.

303. A 50-year-old patient has a screening intraocular pressure of 20 mm Hg bilaterally. Remainder of examination is normal. Which of the following is the best management for this patient? A. Order a fl uorescein angiography. B. Refer to an ophthalmologist emergently. C. Rescreen in 6 to 12 months. D. Begin topical pilocarpine 1%.

303. The answer is C [EENT]. A. A fl uorescein angiography is used to visualize retinal and choroidal vasculature. B. Intraocular pressure of 20 mm Hg without other risk factors/ signs does not warrant emergent referral to an ophthalmologist. C. Intraocular pressure of 20 mm Hg by tonometry is within normal limits (10 to 21 mm Hg). An examination of the optic disc and visual fi elds is also necessary. D. Normal intraocular pressure does not require pharmacologic therapy. Pilocarpine may cause side effects, rarely including retinal detachment and lens opacity.

63. A patient with chronic renal failure secondary to diabetes mellitus has developed chronic constipation unresponsive to dietary changes and increased fl uids. Which of the following should be avoided in attempts to relieve the constipation? A. docusate sodium (Colace) B. lactulose (Chronulac, others) C. milk of magnesia (MOM) D. psyllium (Metamucil, others)

63. The answer is C [GU/Renal, Endocrinology, GI/Nutrition]. A. Docusate sodium, lactulose, and psyllium do not have any effect in the kidney; therefore, they are safe to use in patients with chronic renal insuffi ciency. B. See A. C. Patients with chronic renal insuffi ciency should avoid magnesium-containing compounds as they will not be able to readily excrete the magnesium. Excess magnesium will result in hypermagnesemia. D. See A.

78. A 52-year-old male with no signifi cant past medical history and no current complaints undergoes screening ECG. The rhythm strip shows multiple fi ve to six beat runs of wide QRS complexes. Which of the following medications should be administered now? A. amiodarone B. atropine C. epinephrine D. verapamil

78. The answer is A [Cardiovascular]. A. Ventricular tachycardia is defi ned as three or more consecutive ventricular premature beats. Amiodarone or lidocaine, given in a slow IV bolus followed by a slow infusion, is the recommended treatment for stable ( nonsustained) ventricular tachycardia. B. Atropine is used to treat bradycardia, not ventricular tachycardia. C. Epinephrine is best used in the treatment of ventricular fi brillation. D. Verapamil is most often used in the treatment of supraventricular tachycardias.

1. A 62-year-old male with a history of stable angina presents with mild chest pain and shortness of breath. Pulse rate is 142 bpm and irregularly irregular, and BP is 90/50 mm Hg with mild orthostatic changes. ECG reveals erratic, disorganized atrial activity between discrete QRS complexes, which also occur in an irregular pattern. What is the immediate treatment of choice? A. Beta-blocker B. digoxin C. electric cardioversion D. pacemaker placement

1. The answer is C [Cardiovascular, Geriatrics]. A. beta-Blocker is excellent for rate control in a stable patient; the chest pain and low blood pressure make this patient unstable. B. Digoxin also provides good rate control but should not be used in an urgent situation. C. Electric cardioversion is the treatment of choice in the urgent situation with an unstable patient with shock and ischemia. A shock with 100 to 200 J administered at the R wave is the initial trial. D. Temporary pacemakers may be needed in bradycardias and sick sinus syndrome.

10. A 24-year-old patient has a 3-year history of GERD symptoms. He has failed multiple pharmacologic treatment regimens, including H2 blockers, proton pump inhibitors, and metoclopramide. What is the next best step? A. 24-hour pH probe B. barium esophagography C. esophagectomy D. upper endoscopy

10. The answer is D [GI/Nutrition]. A. Ambulatory pH monitoring is indicated (1) in patients with a normal endoscopy who have reflux symptoms unresponsive to therapy with a proton pump inhibitor; (2) to document abnormal esophageal acid exposure in a patient being considered for antireflux surgery who has a normal endoscopy; and (3) to detect either abnormal amounts of reflux or an association between reflux episodes and atypical symptoms such as noncardiac chest pain, asthma, chronic cough, laryngitis, and sore throat. B. Barium esophagography has a limited role in the workup of GERD symptoms. It may be helpful to delineate strictures. C. Esophagectomy is an invasive procedure and is not indicated unless dysplastic changes have been documented. D. Upper endoscopy is an excellent study. It allows for visualization of the esophagus, stomach, and duodenum. Biopsies are readily obtained.

100. A woman brings her elderly father to the emergency department due to bizarre motor activity. She states he has severe Alzheimer disease and was begun on haloperidol (Haldol) about 4 weeks ago due to increasing psychotic behavior. Which of the following is the most likely motor activity he is displaying? A. repetitive, rhythmic tongue protrusion and opening and closing of the mouth B. brief, repetitive localized muscle twitches commonly of the face C. occasional, brief sudden jerks of the extremities, especially when falling asleep D. brief, fl orid, irregular movements that interrupt normal motion

100. The answer is A [Neurology, Psychiatry]. A. Tardive dyskinesia often occurs after months or years of psychotropic medications and is characterized by repetitive, rhythmic, stereotypic movements of the face, mouth, tongue, and sometimes, limbs and trunk. It can happen after a short trial or after chronic use of a psychotropic medication. B. A localized muscle twitch is the typical description of a tic. C. Myoclonic movements of this nature frequently occur in normal patients. D. Chorea is manifested by brief, dance-like, irregular movements that interrupt normal motion. It is not associated with medication use.

11. An 8-year-old boy is brought in for evaluation of a chronic rash on his trunk. Examination reveals multiple erythematous, scaling plaques and papules with raised borders and some central clearing. What is the best next step in management? A. Examine scrapings in a 20% KOH solution by direct microscopy. B. Culture scrapings on Sabouraud glucose medium. C. Obtain multiple punch biopsies and submit for pathology. D. Culture scrapings on blood agar.

11. The answer is A [Dermatology, Infectious Disease]. A. In a suspected dermatophyte infection, it is appropriate to collect scales with a scalpel blade or edge of a glass microscope slide, place in a 5% to 20% KOH solution, heat gently, and examine for hyphae or mycelium. B. Culturing on Sabouraud medium may be helpful when direct microscopy results are negative, but the clinical suspicion of dermatophytosis is strong. Culturing may take weeks before results are available. C. Biopsies are not appropriate in typical dermatophyte rashes. D. Culture for Staphylococcus aureus or Streptococcus is not appropriate, as the clinical picture suggests fungal infection.

25. A 54-year-old is brought to the emergency department after losing consciousness. Just prior to the event, he complained of an acute severe headache unlike anything he had ever experienced before. He is now conscious but likely displays which of the following symptoms? A. amnesia B. confusion and irritability C. weakness and lassitude D. parietal scalp tenderness

25. The answer is B [Neurology]. A. Postevent amnesia is unusual in subarachnoid hemorrhage. B. Most patients with SAH regain consciousness but display confusion and irritability. Other patients progress to coma, depending on the size of the hemorrhage. C. Lethargy and lassitude are common after a seizure. D. Parietal scalp tenderness is associated with temporal arteritis.

12. The straight leg raise test is used to evaluate possible disk disease of the lower back. What is considered a positive straight leg raise test? A. pain radiating along the sciatic nerve at 20 degrees of active elevation of the ipsilateral leg B. pain radiating along the sciatic nerve at 60 degrees of active elevation of the ipsilateral leg C. pain radiating along the sciatic nerve at 20 degrees of passive elevation of the ipsilateral leg D. pain radiating along the sciatic nerve at 60 degrees of passive elevation of the ipsilateral leg

12. The answer is D [MSS/Rheumatology, Surgery]. A. See D. B. See D. C. See D. D. A positive straight leg raise test replicates the symptoms of disk disease. It is done by passively elevating the ipsilateral leg 30 to 60 degrees. Further support for the diagnosis can be elicited by lowering the leg 10 degrees below the level of pain and dorsifl exing the foot.

13. A 33-year-old female presents to the emergency department with severe right fl ank pain that radiates to the groin. She is nauseous but afebrile. She is anxious and unable to lie still. Urinalysis shows marked hematuria and no casts. What is the most appropriate next step in the management? A. lithotripsy B. narcotic analgesia C. intravenous fl uids D. helical CT scan

13. The answer is B [Urology/Renal]. A. Extracorporeal shock wave lithotripsy (ESWL) is reserved for stones that do not pass with time. B. Analgesia is the most appropriate treatment. The pain of an obstructing stone is exquisite. Due to the common presence of nausea, parenteral narcotics are commonly required. C. Intravenous fl uids are given during the acute phase, but evidence to support their role in forcing stone passage is lacking. Increased fl uids are a mainstay in preventing recurrence. D. Helical CT scan is the imaging modality of choice in nephrolithiasis. The quality of the study will be improved with analgesia.

14. A 10-year-old presents with ear pain. Examination reveals edema of the external auditory canal producing an anterior and inferior displacement of the auricle with percussion tenderness posteriorly. What is the most likely diagnosis? A. acute otitis externa B. mastoiditis C. otitis media D. malignant otitis extern

14. The answer is B [EENT, Infectious Disease, Pediatrics]. A. Otitis externa presents with tragal tenderness and infl ammation of the ear canal without tenderness over the mastoid. B. This scenario is most consistent with mastoiditis caused by an acute otitis media that was not treated adequately. C. Otitis media presents with a cloudy fl uid-fi lled middle ear with a bulging or retracted membrane. D. Malignant otitis externa is usually caused by Pseudomonas and presents as otitis externa with extension to surrounding anterior tissues with tissue necrosis. It is more common in diabetics or immunocompromised patients.

15. A generally healthy, well-developed 18-year-old female presents to the emergency department with chest pain, which has recurred three times during the past week. The chest pain is accompanied by palpitations and is not brought on by exertion. She explains that she has been very anxious about leaving for her first year of college in a couple of days and feels like she is having a heart attack now. A year ago, she experienced similar pain events while preparing for the college placement examinations. She denies tobacco or illicit drug use and takes no medications or supplements. Which of the following additional symptoms is most likely to be found in this patient? A. bradycardia B. hyperrefl exia C. sensation of breathlessness D. vertigo

15. The answer is C [Psychiatry/Behavioral Medicine]. A. Tachycardia is more common in panic attacks. B. Hyperrefl exia is common in hyperthyroidism. C. Typical presentation of a panic attack includes unexpected, untriggered periods of intense anxiety and fear with associated physiologic changes which include palpitations, sweating, tremulousness, a sensation of breathlessness, chest pain, GI distress, and faintness. D. Patients may experience feelings of impending doom or feelings of faintness during a panic attack but typically not vertigo

16. A 29-year-old multipara presents to a rural clinic. She successfully delivers a baby boy; a continuous, rough, machinery-like murmur is heard on cardiac auscultation of this newborn. What is the drug of choice to administer to the newborn? A. ampicillin B. _-blocker C. heparin D. indomethacin

16. The answer is D [Cardiovascular, Pediatrics, Surgery]. A. Antibiotics are not indicated in the treatment of patent ductus arteriosus. B. beta-Blockers will slow the heart rate and, therefore, may be harmful. C. Anticoagulants are not indicated and may be harmful in the neonate. D. Indomethacin, a prostaglandin inhibitor, is routinely administered to help close a patent ductus arteriosus.

17. Which of the following is the drug of choice in the urgent treatment of a panic attack? A. amitriptyline (Elavil) B. propranolol (Inderal) C. buspirone (BuSpar) D. lorazepam (Ativan)

17. The answer is D [Psychiatry/Behavioral Medicine]. A. An antidepressant may be used in sustained treatment. B. b-Blockers have been used in conjunction with other therapies to control some symptoms. C. Buspirone is indicated for the chronic treatment of panic attack but has no place in the treatment of acute attacks. D. A benzodiazepine, such as sublingual lorazepam is effective as an urgent treatment.

18. A 67-year-old has been admitted to the hospital for community- acquired pneumonia. She is dehydrated, weak, hypotensive, and has a cough productive of thick, green phlegm. Examination reveals coarse lung sounds throughout all lung fi elds. She is started on IV levofl oxacin at admission and develops diarrhea on day 4 of treatment. Which part of her history places her at greatest susceptibility for the development of Clostridium diffi cile infection? A. community-acquired pneumonia B. hospital admission C. history of diverticulosis D. patient's age

18. The answer is B [GI/Nutrition]. A. Antibiotics, not the underlying infections, puts a patient at risk for antibiotic-associated colitis. B. Antibiotic-associated colitis is a significant clinical problem almost always caused by Clostridium difficile infection. Hospitalized patients are most susceptible, especially those who are severely ill or malnourished. C. difficile colitis is the major cause of diarrhea in patients hospitalized for more than 3 days. Characteristically, the diarrhea occurs during the period of antibiotic exposure, is dose related, and resolves spontaneously after discontinuation of the antibiotic. Patients who develop severe diarrhea may benefit from a course of metronidazole, vancomycin, or fidaxomicin. C. Diverticular disease does not predispose a patient to antibiotic-associated colitis. D. Antibiotic-associated colitis can occur at any age.

19. Which of the following pulmonary function test results is consistent with a diagnosis of emphysema-predominant COPD? A. decreased total lung capacity (TLC) B. increased diffusing capacity (DLCO) C. decreased ratio of forced expiratory volume in 1 second to forced vital capacity (FEV1/FVC ratio) D. markedly decreased residual volume (RV)

19. The answer is C [Pulmonology]. A. The TLC is increased in emphysema due to air trapping and a fl attened diaphragm. It is generally normal in bronchitis- predominant COPD. B. The increased TLC in emphysema-predominant COPD leads to a markedly reduced DLCO. It is typically normal in bronchitis-predominant COPD. C. FEV1/FVC ratio is reduced in all types of chronic obstructive pulmonary disease. D. The air trapping and increased TLC in emphysema coupled with a reduced FEV1 results in a markedly increased RV.

2. A 32-year-old male presents with a complaint of conjunctivitis and an monoarthritis involving the left knee. Past medical history is signifi cant for chlamydial urethritis, which was treated with doxycycline 3 weeks ago. What is the most likely diagnosis? A. Behçet syndrome B. psoriatic arthritis C. reactive arthritis D. systemic lupus erythematosus

2. The answer is C [MSS/Rheumatology, Infectious Disease]. A. Behçet syndrome is characterized by painful aphthous ulcers in the mouth. B. Psoriatic arthritis is usually a polyarthritis. Arthritis typically precedes development of skin lesions. Nail pitting is common. C. Reactive arthritis (formerly known as Reiter syndrome) is characterized by urethritis, conjunctivitis, mucocutaneous lesions, and septic arthritis. Cases develop after dysenteric infections or sexually transmitted infections, commonly chlamydia. Antibiotics treat the infection but have no effect on alleviating the symptoms or preventing the development of the disorder. D. Systemic lupus erythematosus presents with a myriad of symptoms including joint and eye manifestations, but fever, malaise, weight loss, and skin changes are the most typical.

20. A man who was found unconscious is brought to the emergency department. He is hypotensive, tachycardic, and febrile. Laboratory fi ndings include an elevated white blood cell count, elevated lactic acid, a low platelet count, and a normal hematocrit. Swan Ganz is placed and reveals decreased preload, decreased afterload, and decreased systemic vascular resistance. What is the most likely diagnosis? A. septic shock B. neurogenic shock C. cardiogenic shock D. hypovolemic shock

20. The answer is A [Infectious Disease]. A. Septic shock is characterized by hypotension, tachycardia, decreased preload, decreased systemic vascular resistance, and, often, fever. Severe sepsis may present with hypothermia. Alkaline phosphatase will be ele vated, whereas platelets and other coagulation factors are often decreased. B. Neurogenic shock is characterized by hypertension, normal or increased pulse, an increased to normal cardiac index, and decreased systemic vascular resistance. C. Cardiogenic shock is characterized by hypotension, tachycardia, a markedly decreased cardiac index, and markedly increased systemic vascular resistance. Laboratory studies would show metabolic acidosis and an increased BUN and creatinine. D. Hypovolemic shock is characterized by hypotension, tachycardia, decreased cardiac index, and increased systemic vascular resistance. Arterial blood gas results would reveal metabolic acidosis. Serum electrolytes and serum lactic acid levels would be abnormal. The hematocrit may be low if blood loss is the cause.

21. A 27-year-old male has a solitary, solid, 1-cm, fi rm nodule in the right side of his thyroid which was picked up on routine examination. Ultrasonography confi rms a single solid mass with microcalcifi cations. What is the best next step? A. MRI of the neck B. excisional biopsy C. fi ne-needle aspiration and biopsy D. observation and reassess in 3 months

21. The answer is C [Endocrinology, Surgery]. A. An MRI is not indicated at this time. It would be part of a metastatic evaluation if the biopsy proves positive for malignancy. B. An excisional biopsy is not an initial procedure for a solitary pulmonary nodule and should be reserved for patients with equivocal needle biopsies. C. A fi ne-needle aspiration biopsy should be obtained for cytological evaluation of the thyroid mass tissue. It is minimally invasive and is cost-effective. D. The possibility of malignancy must be excluded in this suspicious solitary mass.

22. A 50-year-old male gives a history of shooting pain that begins at the side of his mouth and radiates to his ear. Episodes occur several times per day with several days between clusters. What is the recommended treatment? A. aspirin B. acetaminophen C. phenytoin D. carbamazepine

22. The answer is D [Neurology]. A. Aspirin and acetaminophen may treat discomfort but will not treat the underlying condition. B. See A. C. Phenytoin should be tried if carbamazepine is ineffective or not well tolerated. D. Carbamazepine or oxcarbazepine are drugs of choice in trigeminal neuralgia.

23. A 45-year-old woman presents with a 1-week history of sudden episodes of lancinating right facial pain that radiates toward the ear. These episodes last anywhere from seconds to minutes. The pain is triggered by touch or eating. What is the most likely diagnosis? A. sinus infection B. cluster headache C. migraine headache D. trigeminal neuralgia

23. The answer is D [Neurology]. A. Sinus pain is characteristically continuous frontal and maxillary throbbing pain that is exacerbated with head movement. B. Cluster headaches predominately affect middle-aged males and present with unilateral periorbital pain accompanied by one or more of the following: ipsilateral nasal congestion, rhinorrhea, lacrimation, and redness of the eye or Horner syndrome. C. Migraine pain may be unilateral or bilateral, dull or throbbing head pain, often associated with nausea, phonophobia or photophobia, and visual changes. Migraine headaches may last for hours to days. D. Pain in the distribution of the trigeminal nerve is pathognomonic for trigeminal neuralgia. The pain is often triggered by touch, movement, and draft. The patient may try to hold her face still while talking.

24. A 13-year-old male complains of knee pain that gets worse throughout the day. Examination reveals tenderness and swelling over the proximal anterior tibia. Which of the following is also likely to be found in this patient? A. hip and knee pain that is worsened with internal rotation B. warm, swollen, painful knees C. knee pain that is exacerbated by descending stairs D. increased pain when walking on fl at surfaces

24. The answer is C [MSS/Rheumatology, Pediatrics]. A. Hip or knee pain that is worsened with internal rotation is typical of Legg-Calvé-Perthes disease. B. Warmth and swelling of a joint indicate infection or acute infl ammation. C. Activity dependent on quadriceps muscle movement, such as ascending and descending stairs, exacerbates the knee pain experienced in Osgood-Schlatter disease, an infl ammatory process of the tibial tuberosity. D. Walking on fl at surfaces does not exacerbate the pain of Osgood-Schlatter disease.

26. An 18-year-old asymptomatic male presents for college entrance physical exam. His body mass index is 33 kg/m2. Vitals include BP of 156/84 mm Hg, pulse rate of 88 bpm, and respiratory rate of 16. He has a family history of diabetes mellitus, hypertension, and coronary artery disease. Which of the following laboratory studies is indicated at this time? A. C-reactive protein B. fasting aldosterone levels C. random plasma glucose D. hemoglobin A1c

26. The answer is D [Endocrinology]. A. C-reactive protein is not currently recommended as a screening test for future cardiac risk. B. There is no indication for obtaining an aldosterone level in this patient. C. Obese patients should be assessed for medical consequences of their obesity by screening for metabolic syndrome (high-density lipoprotein, triglycerides, blood pressure, fasting plasma glucose, and waist circumference). Additional tests may include thyroid function studies, dexamethasone suppression testing, and insulin level with C-peptide measurements to exclude endocrine causes for his obesity. D. Hemoglobin A1c greater than 6.0 is presently recommended by the American Diabetes Association as diagnostic for diabetes mellitus

27. A 35-year-old male presents after a near-syncopal episode. He describes a 2-week history of palpitations, weakness, and dyspnea on exertion. He has been drinking alcohol daily due to the recent loss of his job. The patient's pulse is irregularly irregular. Which of the following is the most likely diagnosis? A. atrial fi brillation B. paroxysmal supraventricular tachycardia C. ventricular bigeminy D. premature atrial complexes

27. The answer is A [Cardiovascular]. A. Acute alcohol excess and alcohol withdrawal in predisposed individuals may precipitate atrial fibrillation. This syndrome is often termed the "holiday heart." B. Although paroxysmal supraventricular tachycardia (PSVT) may result in palpitations, it typically produces a regular pulse. C. In ventricular bigeminy, every other cardiac depolarization arises from a ventricular ectopic focus. This produces a regular irregularity in the pulse and does not result in the systemic symptoms. D. Premature atrial complexes may cause palpitations and an irregularly irregular pulse but do not produce systemic symptoms.

28. A 61-year-old female complains of insidious, bilateral, progressive muscle weakness of her legs, which has caused inability to climb stairs or rise from a seated position without diffi culty. What is the most likely diagnosis? A. Sjögren syndrome B. polyarteritis nodosa C. polymyalgia rheumatica D. polymyositis

28. The answer is D [MSS/Rheumatology, Geriatrics]. A. Sjögren syndrome commonly presents with dry eyes and mouth. Patients may also have swallowing difficulties and parotid enlargement. B. Patients with polyarteritis nodosa report an insidious onset of fever, malaise, and weight loss. Extremity pain may be a prominent feature. C. Patients with polymyalgia rheumatica complain of pain and stiffness of shoulder and pelvic girdle areas. They also may have fever, malaise, and weight loss. It does not, however, cause weakness. D. Polymyositis is characterized by proximal muscle weakness of insidious onset. The lower extremities are usually affected first with progression to the upper arms and sometimes the neck. Weight loss, fever, malaise, and ocular changes are not associated with this condition.

29. A 4-year-old girl presents for a routine checkup. She is new to the practice. Physical exam reveals a harsh, holosystolic grade III/VI cardiac murmur at the left sternal border with wide radiation and fi xed split S2. There is no change with position or respiration. What is the most likely diagnosis? A. innocent murmur B. aortic regurgitation C. patent ductus arteriosus D. ventricular septal defect

29. The answer is D [Cardiovascular, Pediatrics]. A. Innocent murmurs are usually soft (grade I or II), short in duration, and systolic. If a ventricular septal defect is misdiagnosed as an innocent murmur and not corrected, the sequelae of the increased pulmonary artery pressure from the ventricular septal defect can lead to irreversible pulmonary artery hypertension (Eisenmenger syndrome), resulting in premature death. Split S2 occurs only with inspiration. B. The murmur of aortic regurgitation is a diastolic murmur, not a holosystolic murmur. C. The murmur of a patent ductus arteriosus is continuous, not holosystolic. The murmur accentuates in late systole. D. Small ventricular septal defects typically cause harsh, holosystolic murmurs heard best at the left sternal border. The fixed split S2 has a louder pulmonary component and indicates a septal defect.

3. A 30-year-old female who is 32 weeks pregnant begins to experience tremors, heat intolerance, and irritability along with some fatigue, tachycardia, hypertension, and lower abdominal pain. Labs reveal the following: Hct 33%; Hgb 12.8 g/dL; WBC 14,600/_L am cortisol 42 _g/dL (normal 5 to 20 _g/dL) Total thyroxine 13.1 _g/dL (normal 5 to 12 _g/dL) Total T3 225 ng/dL (normal 70 to 205 ng/dL) TSH 0.4 _U/mL (normal 2 to 10 _U/mL) Which of the following therapies is the treatment of choice? A. amiodarone B. propranolol C. propylthiouracil D. radioactive iodine

3. The answer is C [Endocrinology, Ob/Gyn]. A. Amiodarone can be a cause of hyperthyroidism and is not used for the treatment. B. beta-Blockers may alleviate symptomatology of hyperthyroidism but may cause fetal growth retardation. C. Hyperthyroidism results in low TSH and elevated T3 and thyroxine (free T4). It may cause intrauterine growth retardation, prematurity, or transient thyrotoxicosis in the newborn. Propylthiouracil is the only drug recommended for treatment of hyperthyroidism during pregnancy and lactation. This drug does cross the placenta and, although rare, may result in excess TSH secretion and goiter in the fetus. Therefore, the smallest dose possible should be used. Very little is secreted in breast milk; adverse effects in the fetus have not been demonstrated. D. Radioactive iodine would be harmful to the fetus.

30. Which of the following laboratory results is most consistent with a diagnosis of primary hyperparathyroidism? A. serum calcium 11.5 mg/dL (normal 8.5 to 10.5 mg/dL) B. ionized calcium 3.2 mg/dL (normal 4.6 to 5.3 mg/dL) C. serum magnesium 1.1 mEq/L (normal 1.3 to 2.1 mEq/L) D. serum phosphate 3.0 mg/dL (normal 2.5 to 4.5 mg/dL)

30. The answer is A [Endocrinology]. A. Hypercalcemia (greater than 10.5 mg/dL) is the hallmark of primary hyperparathyroidism. B. Ionized calcium levels are invariably increased in primary hyperparathyroidism, typically greater than 5.4 mg/dL. C. Serum magnesium is not affected in primary hyperparathyroidism. D. Serum phosphate is usually low in primary hyperparathyroidism.

31. A patient presents with numbness of the anterolateral aspect of the right calf. Exam reveals ipsilateral foot drop. She states that she has recently started meditation classes and has been spending long periods of time sitting cross-legged on the fl oor. Which of the following is the most likely diagnosis? A. femoral neuropathy B. meralgia paresthetica C. peroneal nerve contusion D. tarsal tunnel syndrome

31. The answer is C [Neurology]. A. Femoral neuropathy presents with weakness and wasting of the quadriceps muscles. B. Meralgia paresthetica involves the lateral femoral cutaneous nerve and presents with pain or paresthesia around the outer aspect of the thigh. C. Peroneal nerve contusion is often secondary to compression of the nerve at the head and neck of the fibula. It presents with foot drop and lateral paresthesias. D. Tarsal tunnel syndrome involves the small muscles of the plantar aspect of the foot and toes.

32. A 27-year-old male presents to the urgent care facility following a jamming injury to his second right digit while playing volleyball. He is unable to extend the fi nger at the DIP. Radiographs are negative. What is the name of this injury? A. boutonnière deformity B. gamekeeper thumb C. mallet fi nger D. trigger fi nger

32. The answer is C [MSS/Rheumatology, Surgery]. A. Boutonnière deformity occurs at the PIP joint. This may be due to osteoarthritis or trauma to the central slip extensor. B. Gamekeeper thumb results from radial or valgus trauma to the thumb, resulting in injury to the lateral collateral ligament of the first MCP joint. C. Mallet fi nger results from an object striking the end of the finger, resulting in forced flexion. Patients present with pain at the dorsal DIP joint, inability to actively extend the joint, and, often, a characteristic flexion deformity. An associated avulsion fracture may require surgical intervention. D. Trigger finger typically presents with painful locking or snapping of the PIP joint during flexion. This may be a result of acute or chronic repetitive trauma or chronic disease such as rheumatoid arthritis, gout, diabetes mellitus, or amyloidosis.

33. A 25-year-old male presents concerned about a mass that he found on his left testicle. Examination reveals a thickened spermatic cord that enlarges when a Valsalva maneuver is performed. The right testicle is signifi cantly smaller than the left. What is the most likely diagnosis? A. epididymitis B. hydrocele C. inguinal hernia D. varicocele

33. The answer is D [GU/Renal]. A. Epididymitis is characterized by urethral pain with voiding symptoms. Pain radiates typically along the spermatic cord. B. A hydrocele is a fluid-fi lled sac around the testicle that causes edema of the scrotum. Many are congenital but may be acquired through infl ammation or injury. Hydroceles are typically painless. C. Inguinal hernias cause a bulge along the inguinal canal that is most prominent with standing and coughing or straining. D. Varicose veins of the spermatic cord are referred to as a varicocele. Traditionally, they are described as feeling like a "bag of worms" but may have a more subtle presentation. A varicocele is more commonly located on the left and its presence may affect testicular growth on the contralateral side. Maneuvers that increase intra-abdominal pressure will cause the pampiniform plexus to also increase in size, making its palpation easier.

34. A 17-year-old male complains of sudden unilateral left-sided groin pain. Examination reveals a markedly tender left scrotum with high-riding horizontal testicle. Elevation of the testicle does not reduce the pain. What is the most likely diagnosis? A. epididymitis B. orchitis C. prostatitis D. testicular torsion

34. The answer is D [GU/Renal, Surgery]. A. Epididymitis is characterized by urethral pain with voiding symptoms. Pain radiates typically along the spermatic cord and can be alleviated with scrotal ulceration. B. Orchitis is characterized by unilateral painful testicular swelling associated with edema and inflammatory changes. C. Prostatitis causes pain which is worsened with urination or ejaculation. Exam reveals perineal pain. D. Testicular torsion is common in younger males; peak incidence is age 13 years. Severe testicular pain is present; it is not relieved with elevation (negative Prehn sign). The testicle rides high and lies horizontal. The cremasteric reflex is lost on the affected side

35. A 55-year-old with mild osteoarthritis presents with swelling of the right knee. He describes intermittent pain and a feeling of locking or giving way over the past week. Physical exam reveals tenderness at the knee joint line. Which of the following maneuvers is the most reliable and useful in diagnosing the suspected disorder? A. Lachman test B. McMurray test C. Finkelstein test D. straight leg raise test

35. The answer is B [MSS/Rheumatology]. A. Lachman test is typically positive in the presence of an anterior cruciate tear. B. McMurray test is typically positive in the presence of a medial meniscus injury. C. Finkelstein test is typically positive in the presence of de- Quervain tenosynovitis. D. Straight leg raise test is typically positive in the presence of a herniated nucleus pulposus.

36. A 55-year-old male presents with excruciating pain in the right metatarsophalangeal (MTP) joint. On examination, it is swollen and erythematous. The patient reports two prior episodes in the last 2 years. Joint aspiration reveals urate crystals. His past medical history includes hypertension and type 2 diabetes. Which of the following would be most useful in the long-term management of this patient? A. Discontinue his insulin regimen and substitute with non-insulin hypoglycemic agents. B. Place him on an angiotensin-converting enzyme inhibitor. C. Place him on a thiazide diuretic. D. Restrict purines in his diet.

36. The answer is D [MSS/Rheumatology, GI/Nutrition]. A. Insulin does not precipitate gouty attacks. B. ACE inhibitors have no effect on the treatment or prevention of gout and should be part of a diabetic treatment plan. C. Thiazides inhibit renal excretion of uric acid and should be avoided in patients with gout. D. Purine-restricted diets may help reduce the risk of recurrent gouty attacks by decreasing the urate load.

37. A 31-year-old female presents for evaluation of numbness and tingling of her right leg. History reveals similar symptoms 3 months ago in left arm that resolved after a week. Examination reveals full range of motion but weakness of motor strength. Which of the following is the most sensitive test for diagnosing the suspected disorder? A. MRI B. CT scan C. CSF analysis D. evoked potentials

37. The answer is A [Neurology]. A. Multiple foci are best demonstrated by MRI, supporting the clinical diagnosis of multiple sclerosis. B. A CT scan is less sufficient than an MRI and does not add useful diagnostic information. C. CSF analysis may reveal lymphocytosis or increased protein during or immediately after an exacerbation of the disease. Oligoclonal bands are common but not specific. D. Evoked potentials can locate only subclinical lesions of the visual, brain stem, auditory, or somatosensory

38. A patient with a 10-year history of schizophrenia, currently on risperidone (Risperdal), presents because she has been feeling very sad. She states that she does not want to do her usual activities and just sits at home. She has had similar episodes in the past, but they resolved after several weeks. When she fi rst presented 10 years ago, her symptoms included hearing voices that she believed came from God and getting messages from the radio and television. Pharmacologic treatment of this patient should include the addition of what medication? A. atomoxetine (Strattera) B. carbamazepine (Tegretol) C. nefazodone (Serzone) D. olanzapine (Zyprexa)

38. The answer is B [Psychiatry/Behavioral Medicine]. A. Atomoxetine (Strattera) is used in the treatment of attention-deficit disorder. B. Carbamazepine (Tegretol) is a mood stabilizer that has been shown to be effective in the treatment of schizoaffective disorder, particularly the depressive type. C. Nefazodone (Serzone) is indicated for the treatment of major depression. D. Olanzapine (Zyprexa) is a serotonin-dopamine antagonist (SDA) that is used in the treatment of schizophrenia and schizoaffective disorder. This patient is already on risperidone (another SDA), so this drug would not be helpful to treat the affective symptoms that the patient exhibits.

39. A 38-year-old female with a resolving upper respiratory infection develops fever, pleuritic substernal chest pain, and shortness of breath that is worse while lying down and better while sitting. A pericardial friction rub is noted on physical exam. Which of the following would most strongly support the suspected diagnosis? A. positive ASO titer B. cardiomegaly on chest x-ray C. diffuse ST elevation on ECG D. Gram-positive diplococci on sputum smear

39. The answer is C [Cardiovascular, Infectious Disease]. A. A positive ASO titer confirms antibodies to Streptococcus but is not useful in confirming pericarditis. B. The heart may appear enlarged on chest x-ray only if a large pericardial effusion accompanies the pericarditis. This test would not be diagnostic. C. Diffuse ST elevations are characteristic of pericarditis and would help confirm the suspected diagnosis in this patient. As disease progresses, the ST elevations return to baseline followed by T-wave inversion. D. A sputum Gram stain may reveal the presence of bacteria but is not diagnostic of pericarditis.

4. A mother of a 4-year-old is concerned about a bump on the child's wrist. Examination reveals a 1-cm solitary, soft, mobile, nontender lesion on the dorsal wrist. It transilluminates symmetrically. What is the recommended management? A. aspiration B. observation C. immobilization D. cortisone injection

4. The answer is B [MSS/Rheumatology, Pediatrics]. A. Nonoperative treatment in symptomatic patients consists of aspiration and/or cortisone. B. Asymptomatic ganglions are left untreated. Indications for surgical intervention include limitation of motion, pain, weakness, and paresthesias. C. Immobilization is indicated for symptomatic ganglions prior to resection. D. See A.

40. A 25-year-old female with sickle cell trait plans to participate in a marathon in Mexico City. What symptom related to this trait is she most likely to experience? A. abdominal pain B. jaundice C. long bone pain D. painless hematuria

40. The answer is D [Hematology]. A. Persons with sickle cell anemia may experience abdominal pain due to gallstones or splenomegaly. B. Persons with sickle cell anemia may become jaundiced due to chronic hemolytic anemia. C. Persons with sickle cell anemia experience bone pain, primarily in the long bones and back, during crises. D. Persons with sickle trait are usually asymptomatic; however, they may experience episodes of painless hematuria, particularly when under extreme conditions. Individuals are also at risk for rhabdomyolysis during vigorous exercise, especially when at high altitudes.

41. A 46-year-old woman presents with a chief complaint of burning and tingling on the plantar aspect of the right foot. She has a history of recurrent ankle sprains on the same foot. The area has reduced sensitivity to light touch. Tinel sign is present at the posterior tibial nerve adjacent to the medial malleolus. Which of the following diagnostic tests will confi rm the tentative diagnosis? A. Schilling test B. peripheral blood smear C. nerve conduction velocity D. magnetic resonance imaging

41. The answer is C [MSS/Rheumatology, Surgery]. A. Schilling test and peripheral blood smear would aid in a diagnosis of peripheral neuropathy due to megaloblastic anemia. B. See A for explanation. C. Nerve conduction velocity tests or electromyography are diagnostic to confirm tarsal tunnel syndrome. D. Magnetic resonance imaging can be done to document entrapment but would not diagnose tarsal tunnel syndrome.

42. A 14-year-old boy sustains an inversion injury of the left ankle while playing basketball. Swelling and pain were immediate. Examination on the fi eld reveals mild swelling and tenderness with full range of motion and a mild limp with weight bearing. What is the recommended treatment? A. early aggressive range of motion B. heat alternating with ice for the fi rst 24 to 48 hours C. ice with progressive exercise against resistance as tolerated D. return to full use as soon as weight bearing is tolerated

42. The answer is C [MSS/Rheumatology, Pediatrics]. A. Early gentle range of motion and exercise when the pain begins to subside is appropriate. B. Acutely, ice is the treatment of choice. C. The best initial treatment is ice with progressive exercise against resistance as tolerated. Radiographs are warranted if there is localized tenderness over the malleoli, tenderness beyond ligament attachments, or excessive or unrelenting swelling or inability to bear weight. D. Limited workouts begin when vigorous activities such as hopping can be performed.

43. A 45-year-old male smoker is obese, does not exercise, and consumes a high-fat and high-sodium diet. He was recently diagnosed with angina by his health care provider. What lifestyle change will have the best long-term improvement to his risk profi le? A. cessation of smoking B. low-fat and low-sodium diet C. improvement in exercise D. weight loss

43. The answer is A [Cardiovascular]. A. Smoking remains the number one preventable cardiac risk factor; the risk decreases by 50% 1 year after complete smoking cessation. B. See A. C. See A. D. See A.

44. A 40-year-old man has one episode of painless gross hematuria. What is the initial management? A. Tell him to return if the bleeding recurs. B. Send urine sample for urinalysis and culture. C. Schedule him for IVP and cystoscopy. D. Prescribe antibiotics for probable UTI.

44. The answer is B [GU/Renal]. A. Gross hematuria is associated with significant GU disease and should be investigated to rule out malignancy. B. Evaluation of painless hematuria should begin with urinalysis and culture. Results will help guide further workup. C. Cystoscopy is a component of second-line evaluation. It is indicated if bladder neoplasm or BPH is suspected as the cause based on urinalysis findings. IVP has been replaced to CT or MRI to image the upper urinary tract. D. Gross hematuria is rarely the sole manifestation of bacterial urinary tract infection, especially in males.

45. An adult presents with right-sided facial fullness with purulent nasal discharge and fever for 10 days. He states symptoms began as a simple cold but have progressed. He has taken no medications and is allergic to penicillin. Sinuses are tender. Pharynx is edematous with purulent postnasal drip. What is the most appropriate antibiotic treatment? A. amoxicillin-clavulanate B. levofl oxacin C. telithromycin D. trimethoprim-sulfamethoxazole

45. The answer is D [EENT, Infectious Disease]. A. Amoxicillin-clavulanate is an appropriate first-line therapy for acute sinusitis; however, the patient has a penicillin allergy. B. Levofloxacin is an acceptable treatment of sinusitis in patients with a history of recent antibiotic use. C. Telithromycin is considered a second-line treatment for sinusitis. D. Trimethoprim-sulfamethoxazole as first-line therapy is suitable for sinusitis in patients with penicillin allergy. Patients should also be treated with intranasal corticosteroids.

46. A 51-year-old female presents with a 2-day history of colicky right upper quadrant pain as well as nausea and bloating. Examination reveals temperature of 100.4°F and signifi cant pain upon palpation of the right upper quadrant. Laboratory fi ndings include elevated WBC and mild bilirubinemia. What is the most likely diagnosis? A. acute cholecystitis B. acute pancreatitis C. duodenal ulcer D. viral gastroenteritis

46. The answer is A [GI/Nutrition, Surgery]. A. The triad of RUQ tenderness, fever, and leukocytosis is highly suggestive of acute cholecystitis. Additionally, the serum bilirubin may be mildly elevated in one-quarter of patients. It is more common in women older than the age of 40 years and those who are overweight. B. Abdominal pain may vary from a mild and tolerable discomfort to severe, constant, and incapacitating distress. Characteristically, the pain, which is steady and boring in character, is located in the epigastrium and periumbilical region and often radiates to the back as well as to the chest, flanks, and lower abdomen. The diagnosis of acute pancreatitis is usually established by the detection of an increased level of serum amylase. C. Burning epigastric pain exacerbated by fasting and improved with meals is associated with peptic ulcer disease (PUD). D. Illness caused by viruses is characterized by the acute onset of vomiting and/or diarrhea, which may be accompanied by fever, nausea, diffuse abdominal cramps, anorexia, and malaise.

47. A 39-year-old female presents for evaluation of vague constitutional complaints and cold hands. Examination reveals a three-phase color change of her hands when exposed to cold water. The fi ngers of both hands are diffusely swollen; her lips appear thinned. What is the most likely diagnosis? A. polyarteritis nodosa B. polymyalgia rheumatica C. polymyositis D. scleroderma

47. The answer is D [MSS/Rheumatology]. A. Polyarteritis nodosa is characterized by insidious fever, malaise, and weight loss with pain in the extremities. Mononeuritis multiplex is common. B. Polymyalgia rheumatica is characterized by pain and stiffness of the shoulder and pelvic joints. C. Polymyositis presents as gradual progressive weakness of proximal muscles of the upper and lower extremities. D. Scleroderma is characterized by diffuse fibrosis involving the skin and internal organs. Raynaud phenomenon and polyarthralgia are present in 90% of patients. Constitutional symptoms may be due to the presence of a mild anemia and systemic involvement.

48. A 14-year-old female presents to the emergency department with acute abdominal pain which began diffusely and has settled in the right lower quadrant. Examination reveals tenderness, rigidity, and rebound. Which of the following diagnostic tests would provide the highest accuracy rate for the most likely diagnosis? A. abdominal ultrasonography B. CBC with differential C. computed tomography of abdomen D. plain fi lms of the abdomen

48. The answer is C [GI/Nutrition, Surgery]. A. The diagnosis of appendicitis may be established by ultrasound demonstration of an enlarged and thickwalled appendix. Abdominal ultrasonography is most useful, however, to exclude ovarian cysts, ectopic pregnancy, or tubo-ovarian abscess. B. Although moderate leukocytosis of 10,000 to 18,000/L is frequent (with a concomitant left shift), the absence of leukocytosis does not rule out acute appendicitis. C. Several studies have demonstrated the benefit of CT over ultrasonography and plain radiography in the diagnosis of acute appendicitis. The reported positive predictive value of CT is 95% to 97% and the overall accuracy is 90% to 98%. Furthermore, nonvisualization of the appendix on CT is associated with a normal appendix 98% of the time. D. Radiographs are rarely of value in diagnosing acute appendicitis. Consequently, abdominal films are not routinely obtained unless other conditions, such as intestinal obstruction, may be present.

49. A patient who has diabetes and known coronary heart disease has a fasting LDL cholesterol of 175 mg/dL, HDL cholesterol of 35 mg/dL, and a triglyceride level of 255 mg/dL. Which of the following is the fi rst-line choice of pharmacotherapy for this patient's hyperlipidemia? A. nicotinic acid B. HMG-CoA reductase inhibitor C. fibric acid D. bile acid sequestrant

49. The answer is B [Endocrinology, Cardiovascular]. A. Niacin lowers VLDL and LDL cholesterol levels while increasing HDL cholesterol levels. However, niacin may increase glucose levels and possibly insulin resistance. Use of this drug should be with caution because it has a high risk for hepatotoxicity. B. HMG-CoA reductase inhibitors (statins) are effective in lowering triglycerides and LDL cholesterol levels and increasing HDL cholesterol levels. This class of drugs is the best tolerated with the lowest side-effect profile or toxicity. C. Fibric acids lower triglycerides and increase HDL cholesterol but may also increase LDL cholesterol in patients with elevated triglycerides. Patients with poorly controlled glucose levels will often have elevated triglycerides. Fibrates are the drug of choice for patients with a triglyceride level greater than 400 mg/dL. D. Bile acid sequestrants are generally effective at lowering LDL cholesterol. They may increase triglyceride levels; however, they often cause problems with absorption of other drugs or vitamins. Bile acid sequestrants may cause bloating, increased intestinal gas, and constipation.

5. At 1 minute, a newborn displays the following signs: heart rate of 96 bpm; slow, irregular breathing; active motion; grimaces when catheter is inserted into nostril; and a pink body with slightly blue extremities. What is the Apgar score? A. 5 B. 6 C. 7 D. 8

5. The answer is B [Pulmonary, Pediatrics]. A. See B for explanation. B. Apgar score for this newborn is calculated as follows: heart rate (1 point), respiratory effort (1 point), muscle tone (2 points), response to catheter in nostril (1 point), and color of extremities (1 point). The score should be recorded at 1 and 5 minutes after birth. C. See B. D. See B.

50. The presence of which of the following distinguishes eclampsia from preeclampsia? A. hypertension B. proteinuria C. seizure D. thrombocytopenia

50. The answer is C [Ob/Gyn]. A. Preeclampsia and eclampsia both manifest with hypertension, proteinuria, and thrombocytopenia. B. See A. C. When seizure occurs, the patient goes from a diagnosis of preeclampsia to that of eclampsia. D. See A.

51. A 62-year-old male has had increasing shortness of breath over the past several months. He presents today because his feet are so swollen that he cannot wear shoes. He has a long history of hypertension, which has been erratically treated due to unacceptable side effects to most agents. Physical exam reveals elevated jugular venous pressure, rales throughout both lung fi elds, an S3 gallop, and 3_ pitting edema to the midcalf bilaterally. The cardiac apex is laterally displaced. ECG demonstrates sinus tachycardia and a left bundle branch block. What type of cardiomyopathy is most likely? A. dilated B. hypertrophic C. restrictive D. Takotsubo

51. The answer is A [Cardiovascular]. A. Dilated cardiomyopathy accounts for about 90% of cases. It is more common in men and symptoms are those of left or biventricular failure. B. Hypertrophic cardiomyopathy is less common and usually presents with dyspnea, chest pain, and syncope. C. Symptoms of restrictive cardiomyopathy are predominantly related to right-sided failure. Patients often have a history of amyloidosis, radiation exposure, open heart surgery, diabetes, or endomyocardial fibrosis. D. Takotsubo cardiomyopathy occurs after a major stressor that results in a large catecholamine discharge. It is more common in postmenopausal women and presents with angina and dyspnea.

52. A 45-year-old female presents to the emergency department complaining of chest pain. Which of the following descriptions would best suggest a diagnosis of pericarditis rather than myocardial ischemia? A. pain lasting for over an hour B. retrosternal location C. sharp, stabbing pain D. radiation of pain into the neck

52. The answer is C [Cardiovascular]. A. Both pericarditis and myocardial infarction may produce pain that lasts for hours. B. Both pericarditis and myocardial infarction may produce retrosternal or precordial pain. C. Pericardial pain differs in that it is far more likely to be sharp and stabbing (pleuritic), becoming worse with coughing or inspiration. D. Both pericarditis and myocardial infarction may produce pain that radiates into the neck, back, left shoulder, or left arm.

53. A 16-year-old presents with abdominal pain that began periumbilical and has localized to the right lower quadrant. She is nauseous and has no appetite. What is the most likely precipitant of this event? A. fecalith B. neoplastic hyperplasia C. growth of intestinal polyp D. viral infection

53. The answer is A [GI/Nutrition, Surgery]. A. Obstruction due to a fecalith is the most common inciting event in appendicitis. B. Neoplasms, polyps, and viral infection are less common inciting events in appendicitis. C. See B. D. See B.

54. A patient presents with unilateral hearing loss. Weber reveals lateralization to the right ear. Rinne test reveals the following: RIGHT: bone conduction _ 10 seconds, air conduction _ 5 seconds; LEFT: bone conduction _ 5 seconds, air conduction _ 10 seconds. Which of these other physical exam fi ndings is to be expected? A. cerumen impaction in the right ear B. effusion in the left ear C. otitis media in the left ear D. pain on palpation of tragus or mastoid area

54. The answer is A [EENT]. A. Cerumen impaction will block the ear canal. This would result in lateralization of the Weber test to the affected ear and would also cause bone conduction to be greater than air conduction in the same ear. B. An effusion or otitis media could potentially cause a conductive hearing loss in the affected ear. The Weber test will lateralize to the left ear and bone conduction will be longer than air conduction in the left ear. C. See B. D. Otitis externa and mastoiditis are not associated with hearing loss.

55. A previously healthy 37-year-old businessman complains of a 1-week history of fever and abdominal pain. Further questioning reveals a prodrome of decreased appetite, nausea and vomiting, and general malaise. He denies diarrhea. He took a vacation to Mexico 5 weeks ago and business trip to the U.S. West Coast 2 weeks ago. He also ate at a new steak and seafood restaurant in town last week. He is in a heterosexual monogamous relationship of 4 years. Examination reveals a fl at abdomen, normoactive bowel sounds, right upper quadrant tenderness, and negative Murphy sign. What is the most likely diagnosis? A. enterotoxigenic Escherichia coli B. giardiasis C. hepatitis A D. salmonellosis

55. The answer is C [GI/Nutrition, Infectious Disease]. A. Enterotoxigenic Escherichia coli is a cause of traveler's diarrhea. This patient denies diarrhea. B. Giardiasis is a protozoal infection caused by the fl agellate Giardia lamblia. Its cysts are transmitted by food or water and person-to-person contact, including anal-oral sexual contact. Incubation is 1 to 3 weeks but can be longer. Symptoms range from none to acute profuse diarrhea followed by chronic diarrhea. Chronic disease manifests with cramps, nausea, malaise, anorexia, flatulence, and bloating. Fever and vomiting are uncommon with giardiasis. C. Hepatitis A is transmitted via the fecal-oral route. A common source is undercooked shellfish. The incubation period is about 30 days. He could have contracted this during his trip to Mexico. His symptoms are characteristic of HAV infection. D. Infection with salmonella is typically transmitted via contaminated food or water. The incubation period is 5 days to 2 weeks for enteric fever (typhoid type). The patient could have contracted salmonella during this U.S. West Coast trip 2 weeks ago, but his history does not match the prodrome or severity of illness of the typhoid type. He could have contracted the more common form of salmonella gastroenteritis from his recent dining experience, but its incubation period is 8 to 48 hours; the current illness is not acute nor is there diarrhea

56. A 30-year-old obese white female presents with fatigue and generalized weakness for several weeks. Physical exam reveals mucosal pallor and an atrophic tongue. Laboratory data shows a microcytic, hypochromic appearance to the RBCs. Which of the following is the most likely diagnosis? A. pernicious anemia B. iron defi ciency anemia C. folate defi ciency D. b-thalassemia minor

56. The answer is B [Hematology]. A. Pernicious anemia is a macrocytic anemia. B. Iron deficiency anemia presents with fatigue/ weakness; onset is often insidious. Laboratory values and physical presentation described are consistent with iron deficiency anemia. C. Folate deficiency causes macrocytosis. D. b-Thalassemia minor presents with a microcytic, hypochromic anemia; however, physical exam abnormalities are unusual

57. A 4-year-old girl is brought to the clinic by her mother who states that the child has been complaining of progressively worsening ear pain and itchiness over the past week. Examination reveals left tragal tenderness and an edematous and closed canal. Weber lateralizes to the left. What is the most likely diagnosis? A. otitis externa B. otitis media C. otosclerosis D. TM perforation

57. The answer is A [EENT, Pediatrics]. A. Otitis externa is an infection of the external auditory canal secondary to trauma or a consistently moist environment, which favors the growth of fungi or bacteria. It generally presents with canal itching and pain with movement of the ear. If the canal is closed, Weber is expected to lateralize to the side of the blocked canal. B. Otitis media is an infection of the middle ear, which usually has a fairly acute onset and may be accompanied by fever. In this example, the symptoms have progressively worsened over a 7-day period and the canal is closed secondary to edema. Therefore, it would be impossible to visualize the tympanic membrane to ascertain if effusion was present behind it. C. Otosclerosis causes progressive thickening and fibrosis of the bones of the middle ear, resulting in a conductive hearing loss. There is a strong familial tendency. D. Presentation of a tympanic membrane perforation can range from symptomatic to asymptomatic. A perforation can only be diagnosed after visualization of the tympanic membrane.

58. A 78-year-old male with a history of COPD is diagnosed with infl uenza and treated with supportive care and oseltamivir (Tamifl u). Three days later, the fever returns along with increased cough, sputum production, and sharp chest pain. Chest x-ray reveals consolidation in the right lower lobe. Which of the following organisms is a likely etiologic agent? A. Pneumocystis jiroveci (nee carinii) B. Staphylococcus aureus C. Klebsiella pneumoniae D. Mycoplasma pneumoniae

58. The answer is B [Pulmonology, Infectious Disease]. A. Pneumocystis jiroveci pneumonia is found most commonly in patients with immune suppression due to disease or drug therapy. B. Bacterial pneumonia following influenza is most common in the elderly or in patients with chronic pulmonary, cardiac, or metabolic disease. Staphylococcus aureus, Streptococcus pneumoniae, and Haemophilus influenzae are the most frequent pathogens. C. Klebsiella pneumoniae causes pneumonia most commonly in alcohol abusers and people with diabetes mellitus. D. Mycoplasma pneumoniae causes pneumonia most commonly in young adults in ambulatory settings.

59. A 42-year-old female presents for evaluation of asthma symptoms. She reports awakening one to two nights per week due to cough; she uses her _-agonist daily due to shortness of breath. She uses a _-agonist preexercise but has needed it during exercise as well for the past month. She is currently taking a low-dose inhaled corticosteroid daily in addition to the _-agonist. Besides continuing the short-acting _-agonist as rescue treatment and reevaluating in 2 weeks, what is the preferred treatment for this patient? A. a high-potency inhaled corticosteroid B. a low-potency inhaled corticosteroid plus a leukotriene receptor antagonist C. a low-potency inhaled corticosteroid plus a long-acting b-agonist D. a short course of oral steroids plus a medium-potency inhaled corticosteroid

59. The answer is C [Pulmonary]. A. This patient's asthma is classified as moderate persistent, and she requires step 3 treatment as per current guidelines. High-potency inhaled corticosteroids are not indicated. B. Addition of a leukotriene receptor antagonist is an alternative therapy in step 3 treatment, but it is not the preferred treatment option at this time. C. Preferred step 3 treatment includes use of a low-potency inhaled corticosteroid with the addition of a long-acting b-agonist (LABA). A medium-potency inhaled corticosteroid would also be an option. D. A short course of oral steroids with a medium-potency inhaled corticosteroid is not indicated for step 3 treatment.

6. Six weeks ago, a patient began multidrug treatment for pulmonary tuberculosis, which was contracted in an inner city homeless shelter. The patient now complains of diffi culty seeing, dull eye pain exacerbated with eye movement, and loss of color discrimination. Which of the following is the most likely cause of this complaint? A. ethambutol B. rifampin C. isoniazid D. pyrazinamide

6. The answer is A [Pulmonology, Infectious Disease]. A. Optic neuritis is the most common adverse effect of ethambutol. The likelihood of occurrence is increased with higher doses and length of administration. B. Rifampin toxicity is primarily associated with liver dysfunction, rash, and a fl u-like syndrome. C. Isoniazid most commonly produces hepatitis and peripheral neuropathy. Optic neuritis is rare. D. Hyperuricemia and hepatitis are the most common adverse effects of pyrazinamide.

60. A 25-year-old male presents to the clinic complaining of mild dyspnea on exertion. Examination reveals a prominent jugular pulsation and a palpable parasternal lift. There is a harsh systolic murmur best heard at the second and third left intercostal space; it radiates to the left shoulder. An early systolic sound precedes the murmur during expiration. ECG demonstrates right-axis deviation. What is the most likely diagnosis? A. aortic stenosis B. mitral regurgitation C. pulmonic stenosis D. tricuspid regurgitation

60. The answer is C [Cardiovascular]. A. The murmur of aortic stenosis is systolic but best heard on the right; ECG would more likely demonstrate left ventricular hypertrophy (LVH). B. The murmur of mitral regurgitation is systolic but usually loudest at the apex; ECG would most likely demonstrate left-axis deviation or LVH. C. This is the classic description of pulmonic stenosis. Mild forms of this congenital disorder may not be apparent until adulthood when left-sided heart failure develops. D. The murmur of tricuspid regurgitation is systolic, best heard in the third to fi fth intercostal spaces and increases with inspiration. An S3 may be present. ECG may show right-axis deviation. Tricuspid regurgitation is not associated with an ejection click.

61. A 32-year-old male presents for an occupational physical exam. His past medical history is signifi cant for ulcerative colitis; however, he has not had any signifi cant problems in more than 5 years. He describes worsening symptoms of fatigue, pruritus, anorexia, and indigestion over the past 6 months. His wife has commented to him that his skin and eyes appear "yellow" and has questioned him about his alcohol consumption, which he adamantly denies. Labs reveal a signifi cantly elevated alkaline phosphatase level. AST and ALT are only mildly elevated. ERCP fails to show common bile duct obstruction. What is the most likely diagnosis in this patient? A. acute pancreatitis B. chronic cholecystitis C. choledocholithiasis D. primary sclerosing cholangitis

61. The answer is D [GI/Nutrition]. A. Acute pancreatitis presents with acute pain which is lessened with leaning forward or the fetal position, nausea, and vomiting. AST and ALT as well as amylase and lipase are elevated. B. Chronic cholecystitis develops after repeated bouts of acute cholecystitis and manifests with biliary pain, typically epigastric or right upper quadrant. Stones are invariably present. Liver enzymes are not elevated as there is not an obstructive pattern to this disease process. C. Choledocholithiasis would be evident on ERCP. D. Primary sclerosing cholangitis occurs commonly in patients with ulcerative colitis. It is more common in males than females. Pruritus with progressive jaundice is key to the diagnosis. Cholangiography would show fi brosis of the bile ducts with dilations between strictures.

62. A 5-year-old male presents with crusting facial lesions present for 3 days. The mother reports that prior to the development of the facial lesions, her son was scratching at insect bites. Examination reveals confl uent erosions with honey-colored crusts below the left nares and across each cheek. Temperature is 99.8°F. Which of the following is the most appropriate treatment? A. acyclovir B. cephalexin C. doxycycline D. hydrocortisone

62. The answer is B [Dermatology, Infectious Disease, Pediatrics]. A. Acyclovir is an antiviral benefi cial in the treatment of herpes infections. Herpes infections typically present as grouped vesicles that eventually erode and crust. B. Cephalexin provides appropriate coverage for impetigo caused by Staphylococcus and Streptococcus spp. Systemic antibiotics are recommended with widespread or complicated infections or if systemic symptoms, such as fever and malaise, are present. Topical treatment (mupirocin, bacitracin) may be suffi cient for very mild cases that are limited in extent. C. Doxycycline is a reasonable alternative for the treatment of impetigo but is contraindicated for use in children. D. Hydrocortisone is contraindicated in infectious lesions.

64. A 35-year-old female presents with wrist pain. Examination reveals swelling of the proximal interphalangeal (PIP) joints and mild ulnar deviation bilaterally. She mentions that her mother and sister have similar fi ndings. She is currently using NSAIDs with minimal relief. Which of the following is an appropriate treatment regimen at this time? A. Combine glucocorticoids (up to 10 mg/day) with a DMARD such as methotrexate. B. Discontinue NSAIDs and start glucocorticoids at 10 mg/ day, increasing to 40 mg/day. C. Maintain her on NSAIDs and add 20 mg/day of glucocorticoids. D. Substitute acetaminophen for NSAIDs and add glucocorticoids 10 to 20 mg/day.

64. The answer is A [MSS/Rheumatology]. A. In rheumatoid arthritis, glucocorticoids may be used in conjunction with DMARDs only up to 10 mg/day. Once DMARDs are effective, glucocorticoids should be tapered and removed. B. Glucocorticoids provide prompt anti-infl ammatory results but carry high risk of adverse effects and, therefore, should only be used for short periods and at lower doses. C. DMARDs provide greater long-term relief of rheumatoid arthritis symptoms than NSAIDs or acetaminophen. D. See C.

65. It has been established that a 3-year-old has respiratory syncytial virus (RSV) bronchiolitis. He has no past medical history and is eating well despite his cough and congestion. Examination reveals temperature of 100.9°F, pulse rate of 100 bpm, respiratory rate of 22, and SaO2 of 97%. There is a slight wheeze but no stridor; skin turgor is good. What is the best course of action at this time? A. Give 5 days of azithromycin. B. Hospitalize and begin treatment with ribavirin. C. Initiate oxygen and bronchodilators. D. Give supportive therapy and fl uids.

65. The answer is D [Pulmonology, Pediatrics, Infectious Disease]. A. Antibiotics are not useful in the treatment of viral i nfections. B. Ribavirin is useful in severe cases of RSV or when the child has other risk factors such as prematurity or immunocompromise. C. The child is not exhibiting any signs of respiratory distress. His vital signs are stable, there is no stridor, and his oxygen saturation is good. Recent meta-analyses have questioned the role of bronchodilators such that the most recent guidelines from the American Academy of Pediatrics recommends against their use in routine bronchiolitis. D. Supportive therapy including adequate hydration is the recommendation for uncomplicated bronchiolitis.

66. Which of the following medications has the greatest potential to trigger an acute asthma attack? A. narcotics B. chemotherapeutic agents C. aspirin D. ACE inhibitors

66. The answer is C [Pulmonology]. A. Narcotics are associated with respiratory failure. B. Chemotherapeutic agents are associated with pleural effusion. C. NSAIDs and aspirin can precipitate an acute asthma attack. D. ACE inhibitors are associated with a cough.

67. A 28-year-old who is training for a marathon complains of progressively worsening pain in the left foot. Initially, it was relieved with rest but now is persistent. Examination reveals pain on palpation of the fi fth metatarsal. Which of the following is the best test to confi rm the suspected diagnosis? A. plain radiography B. bone scan C. CT scan D. MRI

67. The answer is B [MSS/Rheumatology]. A. Plain fi lms are typically negative early in the course of a stress fracture. Days or weeks may pass before the fracture line or new bone formation becomes visible. B. Bone scan is the most sensitive and defi nitive test for stress fracture. It will show uptake in the area of the stress fracture before anything appears on plain radiography. C. CT scan and MRI are useful in the evaluation of stress fractures, but they offer no benefi t over a bone scan and are more expensive. D. See C.

68. A 28-year-old G3P2002 at 29 weeks' gestation presents complaining of a gush of fl uid from her vagina. Ferning test is positive. She is afebrile, and the fetal heart tracing is reactive and reassuring. Which of the following is the next step in management? A. cesarean section B. corticosteroids C. oxytocin D. send patient home on bedrest

68. The answer is B [Ob/Gyn]. A. This is most likely premature rupture of membranes (PROM). When the pregnancy is preterm and there is no sign of chorioamnionitis, the patient should be admitted to the hospital and monitored for signs of infection. At this time, because the patient is stable, there is no reason to perform a cesarean section. B. Corticosteroids are recommended in PROM patients prior to 32 weeks' gestation to enhance fetal lung maturity. C. The patient is afebrile and stable, so there is no reason to use Pitocin to induce labor at this time. D. Because this patient is preterm, she should be monitored in a hospital setting. Sending her home would not be appropriate.

69. A 43-year-old who is overweight complains of dull achiness of the lower legs whenever standing for prolonged periods of time. Which of the following is most likely to be found on physical exam? A. decreased posterior tibia and dorsalis pedis pulses B. dilated tortuous veins in posterior thigh and leg C. loss of pigment over the thighs and shins D. pitting edema in bilateral ankles

69. The answer is B [Cardiovascular]. A. Decreased pulses are seen in arterial disease. B. Varicose veins are either asymptomatic or produce dull pain with exertion. C. Long-standing varicose veins/venous insuffi ciency leads to hyperpigmentation and thinning of the overlying skin. D. Pitting edema denotes pathology of the deeper veins or excess interstitial fl uid.

7. A patient is brought to the emergency department after falling and hitting his head on the sidewalk. He has been unconscious for 8 minutes. Which of the following is the most important diagnostic test to do at this time? A. skull radiography B. CT scan of the head C. cervical spine radiography D. lumbar puncture

7. The answer is B [Neurology]. A. Plain skull films will not identify intracranial bleeding. B. CT scan will best identify potentially life-threatening intracranial bleeding as well as any fractures or mass effect caused by cerebral edema. C. Although all patients with head injuries must have cervical spine fractures ruled out, neck films may be deferred by stabilizing the neck while more urgent conditions are addressed. D. Lumbar puncture in the presence of increased intracranial pressure may result in transtentorial herniation.

70. A 33-year-old male presents with unilateral periorbital pain with ipsilateral nasal congestion, rhinorrhea, and lacrimation. He gives a history of multiple attacks occurring in groups about three times per year. What is the acute treatment? A. lithium B. methysergide C. oxygen D. viscous lidocaine

70. The answer is C [Neurology]. A. Lithium and methysergide may be effective prophylactically in patients with cluster headache. B. See A. C. Oxygen is the most effective treatment for acute cluster headache. Injectable or nasal triptan may also be effective. D. Viscous lidocaine has been used to treat cluster headache with severe nasal symptoms but is less effective overall than oxygen.

71. After bathing, a 65-year-old male notes intense, whole body pruritus that is unrelieved with a variety of over-the-counter medications. He gives a history of intermittent headaches with a sensation of fullness of his face and head, weakness, fatigue, and dizziness. Physical fi ndings included systolic hypertension, splenomegaly, and facial plethora. Which of the following diagnostic tests is most appropriate at this time? A. abdominal ultrasonography B. arterial blood gas C. carotid Doppler scan D. complete blood count

71. The answer is D [Hematology]. A. Abdominal ultrasound is not helpful in the diagnosis of polycythemia vera. B. An arterial blood gas may help in differentiating polycythemia vera from a secondary erythrocytosis where the arterial O2 saturation may be normal or low. This is not part of the initial diagnostic workup. C. Carotid Doppler scans would typically not be done initially in a patient with polycythemia vera. Patients with untreated polycythemia vera are at a higher risk for serious thrombotic complications and may need carotid Doppler scans during the chronic course of the illness. D. A complete blood count is the fi rst step in diagnosing polycythemia vera. Erythrocytosis, thrombocytosis, and leukocytosis will be found.

72. A mother states that her 9-year-old child has abrupt episodes of diminished attention where he "disconnects" briefl y from conversations yet has no awareness of these lapses. What EEG pattern would you expect to fi nd in this child during a typical episode? A. generalized rapid spiking B. localized disturbances C. normal EEG pattern D. spike-and-wave activity

72. The answer is D [Neurology, Pediatrics]. A. Generalized high-amplitude, rapid spiking on EEG is associated with generalized tonic-clonic (grand mal) seizures. B. Localized EEG disturbances indicate focal lesions. C. A normal EEG pattern would not be seen during an absence seizure. D. This child is experiencing absence (petit mal) seizures. EEG during an event would show symmetric 3-Hz spikeand- wave activity.

73. A 52-year-old male was well until 2 days ago when he experienced a sudden onset of fever, shaking chills, and myalgias. Today, he also complains of headache and abdominal pain but no nausea, vomiting, or diarrhea. He denies stiff neck, photophobia, or altered mental status. He returned home 2 days ago after completing an extended photo safari trip to several countries in Central Africa. What is the most likely diagnosis? A. amebiasis B. dengue fever C. malaria D. toxoplasmosis

73. The answer is C [Infectious Disease, Hematology]. A. Amebiasis causes colitis, which can be mild or quite severe. Extraintestinal manifestations include hepatic abscess (most common) or infection of the lungs, brain, or genitourinary system. B. Dengue fever causes a high fever with a biphasic pattern along with myalgias, arthralgias, headache, backache, sore throat, and malaise. A rash begins as blotches then develops into a maculopapular rash that spares the palms and soles. C. Malaria is a protozoan disease transmitted by the bite of infected Anopheles mosquitoes. Malaria is a very common cause of febrile illness in tropical countries. Symptoms begin as nonspecifi c, then fever develops along with more toxic signs and symptoms. Diagnosis rests on demonstration of the parasite in stained peripheral blood smears. D. Toxoplasmosis causes CNS disease in immunocompromised patients. CT scan will show ring-enhancing lesions.

74. A fair-skinned patient with a long history of biopsy-proven benign actinic keratoses presents with another steadily thickening lesion on her right temple. Despite recommendations to limit sun exposure, the patient plays golf daily, uses no sunscreen, and intends to continue enjoying her retirement. What is the preferred evaluation for this lesion? A. biopsy the lesion B. culture only if ulceration occurs C. continued clinical observation D. monitor for symmetry, color variation, or border irregularity

74. The answer is A [Dermatology, Oncology]. A. Actinic keratoses (AK) are the most common precancerous lesions in humans, and there is no defi - nite way to distinguish between AK and squamous cell carcinoma (SCC) without a biopsy. There is a continuum of clinical signs that makes distinction diffi cult. Increasing thickness, redness, pain, ulceration, and size suggest progression to SCC, but a biopsy is needed to defi nitively differentiate. B. See A. C. See A. D. See A.

75. A 68-year-old female is brought to the clinic by her husband with complaints of severe episodes of vertigo. This is her fourth attack this week. She is nauseous but has not vomited. She also reports fullness in her left ear accompanied by a low-tone blowing tinnitus and sensation of "fullness." Which of the following represents an appropriate treatment for this patient? A. oral steroids on an as-needed basis B. symptomatic care with antihistamines and diuretics C. topical antibiotic drops for 7 days D. vestibular suppressant medication until symptoms Subside

75. The answer is B [EENT, Neurology]. A. Ménière disease is a disorder associated with an increased endolymph within the cochlea and labyrinth. The precise pathogenesis is unknown, but evidence suggests an inability to regulate the volume, fl ow, and composition of endolymph. Oral steroids are of no benefi t, but intratympanic steroid injections performed by an otolaryngologist may provide relief. B. Antihistamines and diuretics (acetazolamide) in addition to a low-salt diet are the mainstays of treatment for Ménière disease. C. The etiology of Ménière disease is not infectious, and there is no role for antimicrobial therapy orally or topically. D. Vestibular suppressant medication is indicated for labyrinthitis or benign positional vertigo, not Ménière disease.

76. A 53-year-old G7P7007 presents with complaints of urinary incontinence that has become progressively worse over the past 2 months. She states that the incontinence is made worse with coughing or sneezing and when going up stairs. She denies any muscle weakness or suprapubic discomfort. The patient states that she has not had a period in 5 months. Physical exam reveals no evidence of vaginal atrophy. Examination reveals good sphincter tone. Which of the following is the most appropriate intervention? A. instruction in pelvic muscle exercises B. intermittent urinary catheterization C. oxybutynin (Ditropan) D. topical estrogen cream

76. The answer is A [GU/Renal]. A. This patient most likely has stress incontinence due to laxity of the pelvic diaphragm as well as a lack of estrogen causing leakage from the bladder when there is an increase in intra-abdominal pressure. Pelvic muscle exercises (Kegel) are most helpful in the treatment of stress incontinence, which is due to relaxation of pelvic musculature. For best results, patients should perform Kegel 50 to 60 times per day in sets of 10 to 20. B. Intermittent urinary catheterization is indicated in the treatment of overfl ow incontinence (detrusor underactivity). C. Urge incontinence (detrusor overactivity) is the most common cause of incontinence in older patients. Treatment involves use of anticholinergic medications, such as oxybutynin, to relax the bladder. D. Topical estrogen cream is not indicated in cases of stress incontinence when there is no evidence of vaginal or urethral atrophy.

79. A 29-year-old female who is 24 weeks pregnant presents complaining of a sudden onset of a severe, intermittent pain originating in the right fl ank and radiating into the right groin. She also complains of nausea and vomiting. Fundal height is consistent with her dates. She exhibits tenderness to palpation along the right fl ank with no rebound. Urinalysis reveals microscopic hematuria. Ultrasonography reveals right hydroureter and hydronephrosis. What is the most likely diagnosis? A. acute appendicitis B. acute cholecystitis C. bladder cancer D. nephrolithiasis

79. The answer is D [GU/Renal]. A. Although an acute abdomen, such as acute appendicitis, is in the differential diagnosis, the absence of fever and peritoneal signs makes this diagnosis less likely. B. The absence of fever as well as presence of pain radiating to the right groin makes cholecystitis unlikely. Confi rmation of hydroureter and hydronephrosis by ultrasonography also make cholecystitis unlikely. C. Although bladder cancer may have associated microscopic hematuria, it presents with painless hematuria or irritative voiding symptoms. D. A sudden onset of severe colicky fl ank pain associated with nausea and vomiting as well as the absence of rebound makes nephrolithiasis the most likely diagnosis. This is further supported by the presence of hematuria and likely also calcium oxalate crystals on urinalysis.

8. A patient presents with epigastric pain described as boring and radiating to the back. It is relieved with leaning forward. What organ is most likely involved? A. gallbladder B. pancreas C. distal small bowel D. duodenum

8. The answer is B [GI/Nutrition]. A. Sharp right upper quadrant pain is a common symptom in patients with gallbladder pathology. B. The major symptom of acute pancreatitis is midepigastric or left upper quadrant pain. It is described most commonly as a constant, boring pain that often radiates to the back as well as the fl anks, chest, or lower abdomen. Relief is often found with leaning forward or lying in the fetal position. C. Pain of mechanical small bowel obstruction is typically periumbilical or more diffuse. D. Burning epigastric pain is the most classic symptom of peptic ulcer disease that most commonly occurs in the duodenum.

80. A known alcoholic is seen in the emergency department. Workup reveals infl amed gingivae; petechiae; rough, dry skin; anemia; and impaired wound healing. This patient is likely defi cient in what vitamin? A. A B. C C. E D. K

80. The answer is B [GI/Nutrition]. A. Vitamin A is an important component in retinal rods and cones and is essential for normal vision; defi ciency causes night blindness. B. Vitamin C is necessary for biosynthesis of collagen, carnitine, bile acids, and norepinephrine; defi ciency results in scurvy. It most commonly develops in alcoholics or elderly due to dietary inadequacy. C. Vitamin E is an antioxidant and free radical scavenger in lipophilic environments; defi ciency results in neuronal degeneration, which manifests as arefl exia and gait disturbances. D. Vitamin K is essential to several circulating procoagulants and anticoagulants as well as proteins in the bone matrix and renal epithelium; defi ciency results in bleeding.

81. A 24-year-old female presents for routine gynecologic examination. History is signifi cant for fi rst birth at age 16 years and second at age 18 years. Since separation from her husband at age 19 years, she has had several sexual partners. Pelvic examination reveals condylomata acuminata on the labia. For what gynecologic neoplasm is this woman at highest risk? A. cervical cancer B. endometrial cancer C. ovarian cancer D. rectal cancer

81. The answer is A [Ob/Gyn, Surgery, Infectious Disease]. A. Human papillomavirus (HPV) types 16, 18, 31, 33, and 35 appear to have the most oncogenic potential. Infection is associated with cervical cancer. B. HPV has not been associated with endometrial or ovarian cancer. C. See B. D. Although anal cancers have been associated with HPV, this patient would be at highest risk for cervical cancer.

82. Which of the following ethnic groups in the United States have the highest incidence of Rh negativity? A. African Americans B. Caucasians C. Indo-Eurasians D. North American Indians

82. The answer is B [Ob/Gyn]. A. African Americans have an 8% incidence of Rh negativity. B. Caucasians have a 15% to 16% incidence. C. Indo-Eurasians have a 2% incidence. D. North American Indians have a 1% incidence.

83. A 13-month-old is brought to the clinic with what her mother describes as intermittent episodes of abdominal pain. The infant cries and pulls her knees up and vomits during each episode. She has had one thick, brown-red, mucus-laden liquid stool. On examination, a sausage-shaped mass is found in the upper abdomen. What is the best treatment option? A. air enema B. Nissen fundoplication C. pyloromyotomy D. surgical resection

83. The answer is A [GI/Nutrition, Pediatrics, Surgery]. A. Air or barium enema not only confi rms the diagnosis but, with appropriate hydrostatic pressure, will also reduce the intussusception in 75% of patients. B. Nissen fundoplication is the most common antirefl ux operation. C. Pyloromyotomy is the treatment of choice for pyloric stenosis. Infants presents with projective vomiting; an olive shaped mass may be palpated. D. Surgical resection is reserved for ischemic or infarcted tissue.

84. A 28-year-old female from Bangladesh is brought to the United States for evaluation and repair of a heart valve problem. Since having a febrile illness with sore throat and skin rash at age 23 years, she has been experiencing increasing exhaustion, dyspnea on exertion, orthopnea, and paroxysmal nocturnal dyspnea. Physical exam reveals an opening snap just after S2 and a low-pitched diastolic murmur heard best at the apex. CXR demonstrates an enlarged left ventricle. What is the most likely diagnosis? A. aortic regurgitation B. mitral stenosis C. pulmonic regurgitation D. tricuspid stenosis

84. The answer is B [Cardiovascular, Infectious Disease]. A. The murmur of aortic regurgitation occurs during diastole, is usually blowing and faint, and is best heard at the left sternal border and the apex. It does not have an opening snap. B. Mitral stenosis is common after rheumatic fever. The murmur and opening snap are as described. C. Pulmonic regurgitation does have a diastolic murmur, but it is best heard at the left second and third intercostal spaces. It is associated with right ventricular enlargement. D. Tricuspid stenosis may have an opening snap as well as a diastolic murmur; however, radiography would demonstrate an enlarged right atrium.

85. A 75-year-old with a history of Parkinson disease presents with minimally pruritic facial lesions present for 1 week. Examination reveals scattered discrete macules approximately 1 cm in size, with an orange-red greasy scale on the cheeks and nasolabial folds. What is the most appropriate treatment? A. benzoyl peroxide gel B. hydrocortisone cream C. metronidazole gel D. mupirocin ointment

85. The answer is B [Dermatology]. A. Acne is characterized by open and closed comedones. It may also present with papular and pustular lesions with cysts and nodules. Benzoyl peroxide gel is an effective topical treatment for mild acne. B. Hydrocortisone cream is a low-dose topical corticosteroid that is appropriate for fi rst-line treatment of facial seborrhea. C. Metronidazole gel is an effective treatment for rosacea. D. Mupirocin ointment is an effective topical treatment for limited impetigo.

86. A 34-year-old previously healthy male presents with pleuritic chest pain relieved by sitting forward. He has a temperature of 102.5°F. Auscultation reveals a biphasic scratching sound heard best in the supine position. ECG reveals diffuse ST-segment elevations. What is the treatment of choice? A. acetaminophen B. broad-spectrum antibiotics C. ibuprofen D. nitroglycerin

86. The answer is C [Cardiovascular]. A. Acetaminophen is not useful in the treatment of infl ammatory pericarditis. B. Antibiotics are not needed in viral pericarditis, which is by far the most common etiology of infl ammatory pericarditis. C. Nonsteroidal anti-infl ammatory drugs are the treatment of choice in infl ammatory pericarditis. D. Nitroglycerin relieves chest pain caused by ischemia, not infl ammation.

87. A 59-year-old male with a 30-pack-year smoking history complains of painless hematuria that has been present for the past 3 weeks. He denies trauma. Urinalysis reveals gross hematuria. Which of the following is the best next step? A. CT of pelvis B. cystoscopy with biopsy C. intravenous pyelogram D. MRI of pelvis

87. The answer is B [GU/Renal, Oncology]. A. CT or MRI is used to assess size of tumor and tumor extension. B. Bladder carcinoma presents with painless hematuria in 85% to 90% of cases. Although bladder cancer can be detected by ultrasonography, CT scan, or MRI, only a cystoscopy with biopsy will establish the diagnosis with certainty. C. Intravenous pyelogram (IVP) may be indicated to assess the upper urinary tract if biopsies prove negative. D. See A.

88. An African American male presents after repeated outbreaks of tender, red, facial pustules that seem to have hairs embedded in them. He changes his razor blade regularly and shaves twice per day to prevent the "fi ve o'clock shadow" look. What type of facial hair does this patient likely have? A. dyed or bleached B. electrolysis treated C. straight and thick D. tightly curled or spiral

88. The answer is D [Dermatology]. A. Hair dye is not implicated in pseudofolliculitis barbae. B. Electrolysis is unrelated to pseudofolliculitis barbae. C. Pseudofolliculitis barbae occurs less frequently in straight-haired individuals. D. Pseudofolliculitis barbae more commonly presents on the cheeks and neck of individuals with tightly curled, spiral hair. It is more common in blacks. Close shaving may induce the whisker to irregularly curve into and out of the follicular wall, leaving a tender, painful red papule or pustule at the point of hair entry. It may also occur in the axillae, pubic region, or legs.

89. A patient has a 4-mm fl at lesion on her posterior lower thigh. Its color is variegated purple and black, and it has well-demarcated margins. What is the preferred evaluation method? A. excisional biopsy B. KOH prep C. punch biopsy D. Wood's lamp assessment

89. The answer is A [Dermatology, Oncology]. A. Any lesion suspected to be malignant melanoma must be biopsied. Excisional biopsy is preferred, especially with small focal lesions. Excision is also the fi rst step in treatment. A lesion does not have to meet all criteria (asymmetry, borders, color, diameter) to be suspicious. B. KOH prep will confi rm dermatophytosis. C. A punch biopsy may be inadequate in suspected melanoma. D. Wood's lamp assessment is not helpful in evaluating lesions suspicious of melanoma. It is helpful in dermatophytosis.

9. A 27-year-old elementary school teacher returns to clinic for reevaluation of a severe sore throat. She was seen earlier in the week, diagnosed with tonsillitis, and prescribed amoxicillin. Today, she reports she is still febrile and has malaise, odynophagia, dysphagia, and otalgia. She speaks with a "hot potato" voice and is drooling. Physical exam reveals tonsillar displacement, palatal edema, and uvular deviation. What additional intervention is most appropriate at this time? A. addition of oral steroids B. change of antibiotic to clindamycin C. emergent ENT referral for abscess drainage D. immediate intubation for airway maintenance

9. The answer is C [EENT, Infectious Disease]. A. Oral steroids are not indicated in the treatment of peritonsillar abscess unless the airway is obstructed. B. Appropriate antibiotic coverage for tonsillitis includes penicillins, macrolides, or clindamycin. For this patient, antibiotic coverage should be continued with the addition of abscess drainage. Change from one class of antibiotics to another is not indicated. C. Needle aspiration or incision and drainage is indicated for peritonsillar abscess. Defi nitive follow-up is required because of the risk of reaccumulation of infection or development of lateral pharyngeal abscess formation. D. Aspiration of bacteria can result in pneumonia, but airway compromise is rare.

90. A 53-year-old male with diabetes type 2 has blood pressure readings in the 125/84 to 134/92 mm Hg range. Urinalysis is negative. He is prescribed an ACE inhibitor. This patient is at risk for which of the following serum chemistry abnormalities? A. hypernatremia B. hyperglycemia C. hyperkalemia D. hypomagnesemia

90. The answer is C [Cardiovascular, GU/Renal]. A. ACE inhibitors have a potential to cause hyponatremia, although this is not encountered clinically. B. ACE inhibitors do not usually alter glycemic control. C. Hyperkalemia is a common side effect of ACE inhibitors due to increased potassium retention. D. Renal insuffi ciency may result in hypermagnesemia.

91. A 42-year-old male presents to the emergency department with edema, hypertension, microproteinuria, and bilateral palpable fl ank masses. He states he was diagnosed with hypertension 2 years ago, which has been resistant to medical management. His father died from complications of chronic kidney disease. Urinalysis is positive for blood and protein. What imaging modality will best confi rm the suspected diagnosis? A. computed tomography B. intravenous pyelography C. magnetic resonance D. ultrasonography

91. The answer is D [GU/Renal, Genetics]. A. See D. B. See D. C. See D. D. Polycystic kidney disease presents frequently with abdom inal or fl ank pain with microscopic or gross hematuria. A family history is present in up to 75% of cases; an autosomal dominant inheritance is known. Whenever patients present with resistant hypertension and a palpable abdominal mass, the diagnosis should be entertained. Renal ultrasonography is the test of choice to confi rm.

92. A 62-year-old female presents to the emergency room profoundly dehydrated after several bouts of watery diarrhea. She describes the diarrhea as "rice water" in color. She believes the shellfi sh she ate yesterday was improperly cooked. The diarrhea and dehydration associated with this disease are primarily the result of which of the following? A. an endotoxin component of the pathogen's cell wall B. an enterotoxin actively secreted by the pathogen C. direct destruction of the GI tissue by the pathogen D. the immune response of the body in eliminating the Pathogen

92. The answer is B [Infectious Disease, GI/Nutrition]. A. See B. B. Cholera is an acute diarrheal disease that can, in a matter of hours, result in profound, rapidly progressive dehydration and death. The characteristic watery diarrhea ("rice water") is due to the action of cholera toxin, a potent protein enterotoxin produced by the organism after it colonizes the small intestine. Local outbreaks in the United States are typically associated with consumption of contaminated, locally harvested shellfi sh. C. See B. D. See B.

93. Which of the following descriptors suggests a diagnosis of paranoid personality disorder? A. dependent, immature, egocentric, and emotionally labile B. perfectionist, indecisive, egocentric, and needs to control C. shy, introverted, withdrawn, and avoids close relationships D. defensive, oversensitive, secretive, and has limited emotional responses

93. The answer is D [Psychiatry/Behavioral Medicine]. A. These descriptors suggest histrionic disorder. B. These descriptors suggest compulsive disorder. C. These descriptors suggest schizoid disorder. D. These descriptors suggest paranoid disorder.

94. A 32-year-old male, a furniture mover by trade, presents with a history of left-sided scrotal pain. He states that 2 days ago, he noticed some soreness in his scrotum. The pain radiated to his left fl ank; he thought he had pulled a muscle. The pain gradually increased in intensity over the ensuing 48 hours. He has developed dysuria and subjective fever. On examination, his temperature is 39°C (102.2°F); there is some scrotal swelling present and a tender, enlarged scrotal mass. What is the most likely diagnosis? A. epididymitis B. hydrocele C. testicular torsion D. sliding inguinal hernia

94. The answer is A [GU/Renal, Infectious Disease]. A. Epididymitis usually has a relatively acute onset, accompanied by fever, dysuria, and an enlargement of the epididymis. Most cases have an infectious etiology and/ or can follow heavy physical strain. Early in the onset of symptoms, the epididymis may be palpated. As the condition or infection progresses, the epididymis becomes indistinguishable from the testis. Often, elevation of the scrotum will help relieve the pain. B. A hydrocele is generally nontender and is not accompanied by fever or dysuria. C. Testicular torsion is a surgical emergency. It generally presents with very acute onset of symptoms accompanied by nausea and vomiting, but no dysuria. Its occurrence is more common in the 10- to 20-year age group. D. Inguinal hernias may be exacerbated by heavy lifting; however, fever and dysuria are not characteristic.

95. In the newborn, failure to pass meconium is a common sign of Hirschsprung disease. Meconium should be passed within what time after birth? A. 12 to 18 hours B. 24 to 48 hours C. 72 hours D. 1 week

95. The answer is B [GI/Nutrition, Pediatrics, Surgery]. A. See B for explanation. B. Hirschsprung disease, the primary absence of parasympathetic ganglion cells in a segment of the colon, interrupts the motility of the intestine. The absence of peristalsis causes feces to accumulate proximal to the defect, leading to intestinal obstruction. Meconium should be passed within 24 to 48 hours. C. See B for explanation. D. See B for explanation.

96. A 62-year-old male with a 10-year history of diabetes mellitus type 2 complains of frequency and hesitancy. He states his urinary stream is decreased, and he often feels as if he has not emptied his bladder. He has had a few episodes of incontinence. Examination reveals an enlarged, smooth prostate. Urinalysis is positive for WBCs and blood; culture is negative. Which of the following measures is most appropriate? A. Refer for bladder biopsy. B. Measure postvoid residual volume. C. Schedule cystometric testing. D. Perform urine cytology studies.

96. The answer is B [GU/Renal, Surgery]. A. A bladder biopsy would not resolve the patient's urinary retention. B. The picture is suggestive of urinary tract obstruction. A bladder catheter should be inserted after voiding to measure postvoid residual volume. A high residual volume is worrisome. Diabetics may have the added insult of neurologic bladder which would also lead to retention. C. Cystometric testing would be indicated if a nonobstructive cause of urinary retention is suspected. D. Urine cytology studies would be indicated if bladder cancer was suspected. Bladder cancer incidence is not higher in diabetes.

97. A 53-year-old male is admitted with a compression fracture of the lumbar vertebra. Radiography reveals punched-out lytic lesions of the vertebral column. What is the most likely diagnosis? A. chondroblastoma B. multiple myeloma C. osteoporosis D. amyloidosis

97. The answer is B [Hematology, Oncology]. A. Chondroblastoma is a rare benign neoplasm that develops in the epiphyses of long bones in young males. B. Punched-out lytic lesions are characteristic of multiple myeloma. They are best visualized using plain radiography. Pathologic fractures are a common initial presentation. C. Osteoporosis is a diffuse demineralizing disorder. D. Amyloidosis is fairly common in patients with multiple myeloma. Both are plasma cell dyscrasias. Extracellular deposits of amyloid occur in various sites, most notably the kidneys.

98. In a patient with elevated blood pressure, which of the following physical exam fi ndings most strongly indicates a need for workup for secondary hypertension? A. S3 gallop B. fl ank bruits C. CVA tenderness D. retinal exudates

98. The answer is B [Cardiovascular, GU/Renal]. A. An S3 gallop is caused by left ventricular systolic dysfunction and suggests heart failure. It may be caused by uncontrolled essential hypertension. B. A fl ank bruit is frequently caused by atherosclerosis of the renal vasculature. This is one of the more common causes of secondary hypertension. C. Renal vascular disease does not cause CVA tenderness. CVA tenderness indicates infl ammation. D. Retinal exudates are manifestations of long-term uncontrolled hypertension.

99. The fi rst stage of labor is considered complete when the cervical os opening measures how many centimeters? A. 4 B. 6 C. 8 D. 10

99. The answer is D [Ob/Gyn]. A. See D. B. See D. C. See D. D. The second stage of labor starts with full cervical dilation: 10 cm.

77. A patient develops an acute onset of shaking chills 10 minutes into a random donor platelet transfusion. Other than a mild fever, vital signs are normal and there is no evidence of rash, urticaria, or respiratory diffi culty. Which of the following would be the best course of action? A. Slow the transfusion and administer acetaminophen and diphenhydramine. B. Slow the infusion and administer dexamethasone (Decadron). C. Stop the transfusion and follow the acute transfusion reaction protocol. D. Stop the transfusion and begin a broad-spectrum intravenous antibiotic.

The answer is A [Hematology]. A. This is a common uncomplicated reaction that occurs in about 30% of platelet transfusions; it is due to the presence of donor white blood cells. B. Dexamethasone administration would be of use for a persistent urticarial (allergic) reaction. C. Acute transfusion reactions are associated with back pain and hematuria. D. Broad-spectrum antibiotics would be useful for treatment of septicemia


संबंधित स्टडी सेट्स

Wordly Wise 3000® Level 8, Lesson 10

View Set

CPIM Part 1 Missed Questions pt. 1

View Set